You are on page 1of 178

K YU

K THI OLYMPIC TON SINH VIN


LN TH 22
Qung Ngi, 7-13/4/2014

HI TON HC
VIT NAM

TRNG I HC
PHM VN NG

TRNG I HC
PHM VN NG

HI TON HC
VIT NAM

K YU
K THI OLYMPIC TON SINH VIN
LN TH 22

BIN TP
Trn Nguyn An
Trng H S phm, H Thi Nguyn
on Trung Cng
Hi Ton hc Vit Nam & Vin Ton hc
Trnh Thanh o
Trng H KHTN, HQG Tp. H Ch Minh
V Nht Huy
Trng H KHTN, HQG H Ni
Lin Vng Lm
Trng H Phm Vn ng
Nguyn Vn Ninh
Trng H S phm, H Thi Nguyn

QUNG NGI, 7-13/4/2014

GII THIU
K thi Olympic Ton dnh cho sinh vin cc trng i hc, cao ng v hc
vin trong c nc din ra ti Trng i hc Phm Vn ng, Qung
Ngi trong khong thi gian mt tun, t 07-13/4/2014. Quyn k yu ny
ch yu dnh tp hp li mt s bi xut ca cc trng tham d k
thi vi mong mun cung cp thm mt ti liu tham kho cho cc sinh vin
quan tm. Do thi gian kh ngn nn ngoi mt s bi c bin tp tng
i k cng, c mt s bi chng ti gi nguyn cch trnh by nh xut,
cng tc bin tp trong trng hp l nh my li, kim tra tnh chnh
xc v ni dung v chnh t.
Quyn k yu chc chn cn rt nhiu li trnh by, chng ti ngh ngi
c lun iu ny.

Nhm bin tp

Mc lc
I

K THI OLYMPIC TON SINH VIN LN TH 22

Mt s thng tin v k thi


1
Thng tin chung . . . . . . . . . . . . . . . . . . . . . . . . .
2
Kt qu . . . . . . . . . . . . . . . . . . . . . . . . . . . . . .

5
5
6

Din vn tng kt - GS. TS. Nguyn Hu D

Din vn tng kt - PGS. TS. NGT. Phm ng Phc

14

II

19

THI

thi chnh thc


1
i s . . . . . . . . . . . . . . . . . . . . . . . . . . . . . . .
2
Gii tch . . . . . . . . . . . . . . . . . . . . . . . . . . . . . .

21
21
23

Cc bi xut: i s
1
Ma trn . . . . . . . . . . . . . . . . .
2
nh thc . . . . . . . . . . . . . . . .
3
H phng trnh tuyn tnh . . . . . .
4
Khng gian vc t v nh x tuyn tnh
5
Gi tr ring v vc t ring . . . . . .
6
a thc . . . . . . . . . . . . . . . . .

.
.
.
.
.
.

25
25
30
34
36
39
39

.
.
.
.
.
.

42
42
45
46
50
55
57

Cc bi xut: Gii tch


1
Dy s . . . . . . .
2
Hm s . . . . . .
3
Php tnh vi phn .
4
Php tnh tch phn
5
Chui s . . . . . .
6
Phng trnh hm

.
.
.
.
.
.

.
.
.
.
.
.

.
.
.
.
.
.

.
.
.
.
.
.

.
.
.
.
.
.

.
.
.
.
.
.
1

.
.
.
.
.
.

.
.
.
.
.
.

.
.
.
.
.
.

.
.
.
.
.
.

.
.
.
.
.
.

.
.
.
.
.
.

.
.
.
.
.
.

.
.
.
.
.
.

.
.
.
.
.
.

.
.
.
.
.
.

.
.
.
.
.
.

.
.
.
.
.
.

.
.
.
.
.
.

.
.
.
.
.
.

.
.
.
.
.
.

.
.
.
.
.
.

.
.
.
.
.
.

.
.
.
.
.
.

.
.
.
.
.
.

.
.
.
.
.
.

.
.
.
.
.
.

.
.
.
.
.
.

.
.
.
.
.
.

.
.
.
.
.
.

.
.
.
.
.
.

.
.
.
.
.
.

.
.
.
.
.
.

.
.
.
.
.
.

.
.
.
.
.
.

III

MC LC

HNG DN GII

59

thi chnh thc


1
i s . . . . . . . . . . . . . . . . . . . . . . . . . . . . . . .
2
Gii tch . . . . . . . . . . . . . . . . . . . . . . . . . . . . . .

61
61
64

Cc bi xut: i s
1
Ma trn . . . . . . . . . . . . . . . . .
2
nh thc . . . . . . . . . . . . . . . .
3
H phng trnh tuyn tnh . . . . . .
4
Khng gian vc t v nh x tuyn tnh
5
Gi tr ring v vc t ring . . . . . .
6
a thc . . . . . . . . . . . . . . . . .

.
.
.
.
.
.

.
.
.
.
.
.

.
.
.
.
.
.

.
.
.
.
.
.

.
.
.
.
.
.

.
.
.
.
.
.

.
.
.
.
.
.

.
.
.
.
.
.

.
.
.
.
.
.

.
.
.
.
.
.

.
.
.
.
.
.

.
.
.
.
.
.

.
.
.
.
.
.

67
67
84
95
100
106
108

Cc bi xut: Gii tch


1
Dy s . . . . . . .
2
Hm s . . . . . .
3
Php tnh vi phn .
4
Php tnh tch phn
5
Chui s . . . . . .
6
Phng trnh hm

.
.
.
.
.
.

.
.
.
.
.
.

.
.
.
.
.
.

.
.
.
.
.
.

.
.
.
.
.
.

.
.
.
.
.
.

.
.
.
.
.
.

.
.
.
.
.
.

.
.
.
.
.
.

.
.
.
.
.
.

.
.
.
.
.
.

.
.
.
.
.
.

.
.
.
.
.
.

115
115
125
130
143
158
165

.
.
.
.
.
.

.
.
.
.
.
.

.
.
.
.
.
.

.
.
.
.
.
.

.
.
.
.
.
.

.
.
.
.
.
.

.
.
.
.
.
.

.
.
.
.
.
.

.
.
.
.
.
.

.
.
.
.
.
.

.
.
.
.
.
.

Phn I
K THI OLYMPIC TON SINH
VIN LN TH 22

MT S THNG TIN V K THI

Trng i hc Phm Vn ng - n cc on tham d k thi. Ngun: H Phm Vn ng

Thng tin chung

K thi Olympic Ton dnh cho sinh vin nm 2014 c t chc t 713/4/2014 ti Trng i hc Phm Vn ng, thnh ph Qung Ngi, tnh
Qung Ngi. K thi l hot ng phi hp hng nm gia Hi Ton hc Vit
Nam cng vi mt c s o to, 2014 l nm th 22 k thi din ra.
c 85 trng i hc, cao ng, hc vin tham d k thi Olympic Ton
hc sinh vin ln th 22. C 719 lt sinh vin d thi hai mn i s v Gii
tch.
Bn cnh hot ng chnh l vic t chc thi, chm thi v trao gii, Ban t
chc k thi phi hp vi Trng i hc Phm Vn ng t chc mt hi
tho v i mi ni dung ging dy ton trng k thut v mt bui
"Giao lu gia cc nh ton hc v sinh vin" ti trng i hc Phm Vn
ng.

6
C quan t chc

B Gio dc v o to

Lin hip cc Hi Khoa hc v K thut Vit Nam

Trung ng Hi Sinh vin Vit Nam

Hi Ton hc Vit Nam

Trng i hc Phm Vn ng

Ban t chc
Trng ban: GS. TS. Nguyn Hu D (Hi Ton hc) v PGS. TS. Phm ng
Phc (Trng H Phm Vn ng).
Ph trng ban: GS. TSKH. Phm Th Long (Hi Ton hc), GS. TSKH.
Phng H Hi (Hi Ton hc), Th.S. Chu Vn Lng (Trng H Phm
Vn ng).
y vin: TS. on Trung Cng (Hi Ton hc), TS. L Cng (Trng H
Bch Khoa H Ni), TS. Nguyn Thanh Hi (Trng H Phm Vn ng).

Kt qu

Vi kt qu thi ca th sinh, Hi ng thi thng nht c cu v mc trao


gii. S lng gii c trao c th nh sau:
a. Mn i s
- Gii nht: 27 gii.
- Gii nh: 42 gii.
- Gii ba: 100 gii.
- Khuyn khch: 30 gii.
b. Mn Gii tch
- Gii nht: 28 gii.
- Gii nh: 44 gii.
- Gii ba: 95 gii.
- Khuyn khch: 27 gii.
c. Gii c bit
Ban t chc k thi quyt nh trao 11 gii c bit cho cc sinh vin
hoc t im cao nht ca mt mn hoc t hai gii nht ca c hai
mn.

Th trng B GD&T Bi Vn Ga v ng Cao Khoa - Ch tch UBND tnh Qung Ngi trao cc gii c bit. Ngun: H Phm Vn ng

DIN VN TNG KT K THI


Olympic Ton Sinh vin Ton quc ln th 22 1
GS. TS. Nguyn Hu D
Ch tch Hi Ton hc Vit Nam
ng Trng ban t chc k thi

Sut chng ng 22 nm qua, cng l lc bt u ca ma h chy bng


Min Trung, Olympic Ton sinh vin ng hnh cng vi cc trng i
hc, cao ng v hc vin v tr thnh s kin quan trng thc y phong
tro hc tp, ging dy Ton ca sinh vin v cc thy, cc c cc trng.
C l cn nhiu tranh lun vi nhau khng nh vai tr ca ton hc trong
chng trnh ging dy, trong nn gio dc ca nc nh cng nh s nh
hng ca n n s pht trin kinh t, x hi, an ninh quc phng ca t
nc. Ch c mt iu khng th ai ph nhn l tt c cc nc tin tin
trn th gii, ton hc c mt v tr c bit quan trng t bc gio dc ph
thng c s n o to tin s. Ton hc cung cp mt cng c mnh m
gip cho cc chuyn gia gii quyt cc vn chuyn mn ca tt c cc
ngnh, ngay c nhng ngnh thuc lnh vc tng chng xa l nh khoa hc
x hi v ngh thut. Ngoi ra, mt vai tr khng km quan trng, nu cha
ni l quan trng hn, l ton hc trang b cho ngi hc mt t duy logic,
t duy sng to cn thit cho cuc sng v ngh nghip sau ny. l cha
ni n nu ai say m ton hc s thy c sc ht mnh m ca n, s
thy l th gii k diu kch thch chng ta khm ph, sng to. Chnh v
vy ngay mt t nc thc dng nh Hoa K, khi bc chn vo i
hc th bt c hc ngnh g, cc sinh vin u phi tri qua cc mn hc v
Ton.
S nghip gio dc v o to ca nc nh ang trong thi k chuyn bin
vi nhng bc ngot mang tnh quyt nh v ang c ton x hi quan
tm. Vic nh hng ng chng trnh ging dy trong cc trng hc,
hc vin l cng vic u tin nng cao cht lng o to. Nhng ai
ngh rng cc trng c th o to ra nhng con ngi c chuyn mn cao
v trch nhim x hi bi mt chng trnh o to theo phng chm gim
thiu vai tr mn ton, thm ch b hn mn ton ra khi chng trnh ging
dy, th c l l nhng ngi khng thc t v nhiu th h sinh vin ra
trng s phi tr gi bi nhng nhm ln tai hi .
1. Trch ni dung bi pht biu ti l tng kt v trao gii Olympic Ton Sinh vin Ton
quc ln th 22, ngy 12/4/2014, ti Tp. Qung Ngi, Tnh Qung Ngi

10
Cng may l xu hng mun b mn ton trong cc chng trnh o to
ch mi xy ra ri rc mt s trng nng v li nhun kinh doanh. Chnh
v th, d gio dc cc bc H v cao ng VN ang c nhng thng
trm th k thi Olympic ton sinh vin vn nhn c s hng ng nhit
tnh ca cc thy cc c v sinh vin. S nhit tnh gip cho k thi
Ton Olympic sinh vin hng nm tr thnh ngy hi ca ng o sinh
vin v gio vin cc trng v cc hc vin.
Mc tiu chnh ca k Olympic Ton sinh vin ng nhin l to ra cuc
hi ng v tranh ti nh cao tr tu ca cc sinh vin yu thch ton ca
cc trng i hc v cao ng chp cnh c m cho nhng em c hoi
bo tr thnh nh ton hc sau ny, l dp cc trng khng nh c
ng cp v thng hiu quc gia v o to ca mnh. Tuy nhin, vt ln
nhng iu cn c nhiu mc ch cao p khc. Cuc thi Olympic s
l ni gp g ca cc thy c tng cng s hiu bit, cng nhau hc
tp kinh nghim ging dy, c bit l ging dy mn Ton. cng l ni
nhng ngi yu thch nghin cu ton hc gp g v trao i nhau, cng l
dp him c cho cc sinh vin cc trng c th giao lu gp g vi cc nh
ton hc nc nh thp mi ngn la am m ton hc v s la chn
ton hc nh l ngh nghip tng lai ca mnh. Nh , k thi Olympic
ton chc chc s gp phn thc y s pht trin ca ca nn Ton hc Vit
nam ni ring cng nn gio dc nc nh ni chung.
K thi Olympic ton hc sinh vin ton quc ln th 22 din ra t ngy 08
n ngy 12 thng 4 nm 2014 ti Trng i hc Phm Vn ng, trn
vng t Qung Ngi giu truyn thng lch s, vn ha v tnh ngi. D
rng mi c 6 nm pht trin nhng Trng c nhng bc trng thnh
ca nhng trng c b dy o to. K thi cng c t Qung Ngi,
mt tnh ang vun ra tm hin i ca quc t m vn gi c nt c
trng ca mt thnh ph Min Trung. Nhng ai n vi k thi ln ny
c dp i qua tng gc ph lng ngm nhn Thnh ph Qung ngi ang
i mi vi s k tha ca nn vn ha Sa Hunh, c bit cc di tch
lch s, danh lam thng cnh M Kh, khu du lch vn ho Thin n, ni C
am, o L Sn..., ni sinh ra cc lnh t kit xut Trng Cng nh,
Phm Vn ng. . . Chng ta cng hiu c tnh ngi vi s gin tan, v
ngt thanh, thm du ca ng phn x Qung.
Ch vi nm ngy ca k thi, chng ta c chng kin s n lc tranh ti
ca nhng sinh vin ti nng yu thch Ton hc k thi Olympic vi quy m
ln. Hn 700 lt thi ca cc sinh vin t 85 trng i hc, cao ng v hc
vin t mi min T quc t hi v tranh ti hai mn i s v Gii tch ton
hc. Cc em cho mi ngi cm nhn c ton hc l iu huyn diu
v chng minh ti nng cng vi lng say m ton hc tuyt vi ca mnh.
S lng ng o cc on v cc sinh vin tham gia k thi minh chng

11
hng hn cho vai tr ca Ton hc trong o to v s quan tm c bit
ca cc trng i hc v cao ng. S thnh cng ca k thi khng ch pht
hin ra nhng nhn ti ton hc ca t nc m chc chn cn gp phn
vo vic nng cao cht lng ging dy, c bit l ging dy mn Ton hc
cc bc gio dc Vit Nam.

Cc thi sinh (tri) v Ban gim kho (phi). Ngun: H Phm Vn ng

Ban gim kho lm vic ht sc cng minh, khn trng, chnh xc. Chng
ta c thi ph hp trnh xp loi c sinh vin. K tranh ti cng
c 11 em t gii nht ng thi c hai mn thi, trong c 2 em t 30
im mn Gii tch. Trng i hc Khoa hc T nhin, HQG H Ni dn
u kt qu thi vi thnh tch 7 gii nht. l kt qu ng t ho m cc
em t c v chng ta tin tng rng nu cc em gi vng c ngn
la am m, cc em s tr thnh nhng nh Ton hc ln trong tng lai.
Hy chng ta by t s ngng m n ti nng ton hc ca cc emnhng ngi sao mi ang xut hin trn bu tri ton hc Vit Nam.
Cuc thi ginh c s quan tm su sc ca B Gio dc v o to,
ca Lin hip hi KH v K thut. Chng ta vui mng vi s c mt ca Th
trng Bi Vn Ga, ngi ang hng ngy ch o cng tc o to i hc
ca cc trng v hc vin.
Chng ta cng tht ng ngng trc s nhit thnh v phng thc t chc
khoa hc ca Trng ng cai. S ch o cn thn n tng chi tit ca Ban
Gim hiu Nh trng gip cho cuc tranh ti thnh cng rt tt p. K
thi cng nhn c s quan tm c bit ca Lnh o tnh Qung Ngi, cc
s ngnh lin quan thc hin cc cng vic gip cho k thi thnh cng. Cng
ng Ton hc Vit Nam xin gi li cm n su sc ti B Gio dc v o
to, Tnh y v y ban nhn dn tnh Qung Ngi v Trng i hc Phm
Vn ng.
Chng ta cng t ho nhn mnh rng s ti tr ln nht m cc trng v
hc vin dnh cho K thi l to iu kin cho cc em sinh vin c lng yu
thch ton hc c n vi ton hc. V th, BTC cm n c bit ban gim
hiu cc trng, ban gim c cc hc vin. Chng ta cng cm n cc thy

12
cc c ng vin, dy d cc em sinh vin gip cc em thp sng ngn
la say m chinh phc nh cao tr tu. Li cm n tip theo chc chn
phi dnh cho cc em sinh vin. Cc em phn u ht mnh chng
minh kh nng sng to Ton hc ca chnh mnh. Chnh cc em l ngun
ng lc chnh m bo cho s thnh cng tt p ca k thi v thp sng
ngn la Olympic Ton sinh vin. Chng ta cng cm phc cc em sinh vin
v kht vng t c nh cao vinh quang. Mong cc em gi mi kht vng
vn ln khi tr v chng ta s l nhng sinh vin xut sc trong hc tp,
sau ny tr thnh nhn ti ca t nc.
Ban T chc Hi thi cng nhn c s c v v tinh thn v s ng h
v vt cht ca cc c quan, n v, cc t chc chnh tr x hi. Chng ti
xin chn thnh cm n i truyn hnh Vit Nam gip chng ti thc
hin chng trnh. Chng ti cng chn thnh cm n s gp sc ca Tp
on Viettel, Tng cng ty ng st Vit Nam Ngn hng Nng nghip qun
Ty H, H Ni; Nh my lc du Dung Qut, Nh my nc khong Thch
Bch, chi nhnh Ngn hng Vit tin Bank, Chi nhnh ngn hang nng nghip
v pht trin nng thn, Chi nhnh ngn hng BIDV, Chi nhnh ngn hang
VietComBank, Nh my Vina Soy ti Qung ngi. S gip ny ca qu c
quan l nim khch l ln gp phn cho Olymlic ln th 22 thnh cng
tt p.

Cc sinh vin c trao gii nht mn Gii tch. Ngun: H Phm Vn ng

Cuc thi tranh ti ca chng ta kt thc nhng ngn la nhit tnh ca k


thi Olympic ln th 22 s mi mi thp sng trong lng nhng ging vin,
sinh vin v ngi dn x Qung. Tuy nhin, cng nh bt c cuc tranh ti
khc, lc kt thc s c nhiu em t ho v nhng thnh tch mnh t c
v l d nhin vn cn mt vi em v l do no m cha th hi lng vi

13
kt qu ca mnh ch v mt cht thiu may mn. D th, cc em hy gi
vng nim tin v chc chn ngy no cc em s c thnh tch cao trong
nhng ln tranh ti tip theo cng nh ton b s nghip ca mnh.
L c Olympic Sinh vin Ton hc c chuyn v cho n v ng cai
nm 2015- Trng i hc Kinh t, i hc Hu nhng nhng tnh cm tt
p m Trng i hc Phm Vn ng v nhn dn Qung Ngi dnh cho
nhng ngi tham d k tranh ti Olympic Ton hc Sinh vin vn cn ng
mi trong tm tr mi ngi.

14

DIN VN TNG KT K THI


Olympic Ton Sinh vin Ton quc ln th 22 1
PGS. TS. NGT. Phm ng Phc
Hiu trng Trng i hc Phm Vn ng
ng Trng ban t chc k thi

Knh tha qu v i biu, qu v khch qu, cc thy c gio v cc em sinh


vin thn mn.
K thi Olympic Ton sinh vin ton quc ln th XXII, nm 2014 c t
chc t ngy 7 thng 4 nm 2014 n ngy 12 thng t nm 2014 ti trng
i hc Phm Vn ng tnh Qung Ngi kt thc v thnh cng tt p.
n tham d k thi nm nay c 85 on t cc trng i hc v cao ng
trn ton quc. C 719 lt cc sinh vin d thi hai mn i s v Gii tch,
c gn 200 thy c gio l trng, ph on v tham gia Ban Gim kho.
Thay mt Ban t chc, cc Thy c gio v sinh vin trng i hc Phm
Vn ng, ti nhit lit cho mng v cm n s c mt ca cc ng ch
lnh o, ca tt c qu v i biu, cc Thy c gio v cc em sinh vin
n d l b mc v trao gii ca k thi.
Knh tha Qu v !
Trng i hc Phm Vn ng hn hnh l n v ng cai t chc k thi
ln ny. Trong mt tun hot ng, k thi din ra ng k hoch:
Trong hai ngy mng 7 v 8 thng t, Ban T chc n tip cc on t
cc Hc vin, cc trng i hc, cao ng t mi min ca T quc v ng
k d thi. Cng tc n tip c trng i hc Phm Vn ng chun b
chu o, trong khng kh thn tnh v nng m. i sinh vin tnh nguyn
ca trng i hc Phm Vn ng h tr cc on d thi, n cc on,
gii thiu ch n, , hng dn ng i. Trong iu kin Tnh Qung Ngi
cn nhiu kh khn, nhng cc on cng sm n nh ch , yn tm
tham gia k thi. Thnh ph Qung Ngi v trng i hc Phm Vn ng
t c dp c n tip mt s lng ln ngi t cc tnh thnh trong c
nc n tp trung trong mt tun ng nh vy. y l ln u nh trng
ng cai t chc mt s kin ln, do , chun b cho k thi c din ra
sun s, an ton, nh trng phi hp vi chnh quyn a phng, cc
s, ngnh lin quan ca tnh, m bo an ninh trt t a bn v khu vc
1. Trch ni dung bi pht biu ti l tng kt v trao gii Olympic Ton Sinh vin Ton
quc ln th 22, ngy 12/4/2014, ti Tp. Qung Ngi, Tnh Qung Ngi

15
thi; m bo v sinh an ton thc phm, m bo cng tc thng tin, in,
nc, y t, . . . , k thi c din ra thnh cng.
Cc hot ng bn l ca k thi cng c chun b v t chc chu o.
Chiu ngy 8 thng t nm 2014, Ban t chc k thi t chc Hi tho
v Phng php ging dy ton cc trng K thut. c gn 200 cc
Thy c gio dy ton ca cc hc vin, trng i hc v cao ng n
tham d. Hi tho trao i, tho lun si ni v chng trnh o to,
bin son sch gio khoa, gio trnh v cc phng php dy ton hin nay
trong cc trng k thut; Hi tho cho thy s cn thit i vi vic bin
son B sch ton cc trng k thut thng nht s dng, cn i mi
chng trnh ging dy ton bc i hc v cao ng ca cc ngnh k
thut cho ph hp hn vi hnh thc o to tn ch v phng php ging
dy mi. Chuyn t phng php truyn th kin thc sang o to k nng,
kh nng t nghin cu cho sinh vin trong iu kin s tit ging ngy cng
thu gn.

Sinh vin Trng i hc Phm Vn ng. Ngun: H Phm Vn ng

Hot ng Giao lu gia cc nh ton hc vi sinh vin cng c ng


o cc em hc sinh, sinh vin ho hng tham gia. Khch mi m giao lu
l cc Nh khoa hc, Gio s ton u ngnh ca c nc v cc Ging vin
ton nhiu kinh nghim ca cc trng i hc, cao ng. y l dp cc
em sinh vin yu thch mn ton c tip cn cc Nh khoa hc, cc Thy
c gio dy ton. c trao i nhng vn chuyn mn, nhng tm t,
kinh nghim trong vic hc ton v nghin cu mn ton. m giao lu
em li khng kh phn khi v nim tin cho cc sinh vin yu thch mn
ton, mang li s ng vin to ln cho cc sinh vin Vit Nam tip tc c
nhng cng hin, thnh cng trong lnh vc ton hc.
Ngy 09 thng t nm 2014, sinh vin ca cc trng chnh thc d thi hai
mn i s v Gii tch. Ban T chc chun b tt cc iu kin v c s
vt cht t chc k thi. Trong nhng ngy ny, thi tit ti Qung Ngi
kh nng nhng cc th sinh khc phc kh khn, tham gia cc bui thi
mt cch y v nghim tc. Cc Thy c gio lm cng tc coi thi th

16
hin tinh thn trch nhim cao, coi thi nghim tc v khch quan. Ban Gim
kho khn trng trin khai vic chm thi, ln kt qu.
Ngy 11 thng t, Ban t chc hp cc trng on thng nht kt
qu k thi. Ban t chc nhit lit chc mng thnh tch ca cc on v nh
gi rt cao tinh thn hc tp ton hc ca tt c cc sinh vin tham d k
thi ny, v cc em u l nhng sinh vin xut sc ca cc trng c c
i, cc em u xng ng t gii. Vi kt qu thi ca th sinh, Hi ng thi
thng nht c cu v kt qu gii c th nh sau: C 55 gii nht (mn
i s c 27 SV, mn gii tch c 28 SV), c 87 gii nh (mn i s c 43 SV,
mn gii tch c 44 SV), c 184 gii ba (mn i s c 99 SV, mn gii tch
c 95 SV), c 57 gii khuyn khch (mn i s c 30 SV, mn gii tch c 27
SV). Trong s cc em t gii nht, c 11 sinh vin t gii c bit - l
nhng sinh vin t th khoa mn i s hoc Gii tch hoc t gii nht
c 2 mn.
Ti ngy 11 thng 4 nm 2014, Trng i hc Phm Vn ng t chc m
giao lu, gp g thn mt cc Thy C gio cng nh sinh vin ca tt c cc
on. m giao lu din ra vui v, thn tnh, li nhiu tnh cm tt
p gia sinh vin cc nh trng, hc vin.
Trong nhng ngy lu li Qung Ngi, c bit l ngy 10 v 11/4, hu
ht cc on c dp c tham quan cc danh thng v di tch lch s ti
Qung Ngi: nh thm nh my lc du Bnh Sn ti Khu kinh t Dung Qut,
thm m c Hunh Thc Khng, Khu chng tch Sn M, bin M Kh; Khu
lu nim Bc Phm Vn ng, Bnh x mang tn anh hng lit s - Bc s
ng Thy Trm, Khu Du lch Sa Hunh,...., c thng thc cc c sn
ca qu hng Ni n, Sng Tr. Qung Ngi chc chn li nhng n
tng p trong lng du khch.
Trong sut thi gian din ra k thi, khng c s c ng tic no xy ra; an
ninh trt t, an ton giao thng, sc khe ca cc th sinh v qu Thy c
gio u c m bo.
Knh tha qu v !
t chc mt k thi ln, quy m ton quc, Hi Ton hc Vit Nam v
trng i hc Phm Vn ng c nhiu c gng phi hp cng tc
cng nh tranh th s gip t nhiu pha.
Ban T chc v trng i hc Phm Vn ng chn thnh cm n B Gio
dc v o to c nhiu quan tm ch o v tng Bng khen cho nh
trng trong cng tc t chc k thi. c bit, s c mt ca GS.TSKH Bi
Vn Ga, Th trng B Gio dc v o to trong bui l b mc v trao gii
hm nay chnh l s khch l, ng vin to ln i vi phong tro dy v hc
ton trong cc hc vin, trng i hc v cao ng ca c nc. Xin trn
trng cm n Th trng.

17
Xin chn thnh cm n lnh o Tnh y v UBND tnh Qung Ngi c s
ch o su st, s gip tn tnh trong vic t chc k thi ti trng i
hc Phm Vn ng, ng vin v tng bng khen cho Hi Ton hc Vit
Nam. Cm n s h tr ca cc s, ngnh v chnh quyn a phng k
thi din ra an ton v t kt qu tt.
Ban t chc trn trng cm n cc c quan, doanh nghip tham gia ti
tr: Nh my lc ha du Bnh Sn - Qung Ngi, Chi nhnh Ngn hng
ViettinBank tnh Qung Ngi, Chi nhnh Ngn hng Nng nghip tnh Qung
Ngi, Chi nhnh Ngn hng u t v pht trin tnh Qung Ngi, Chi nhnh
Ngn hng VietComBank tnh Qung Ngi, Nh my nc khong Thch
Bch - Cty ng Qung Ngi, Nh my sa Vinasoy - Cty ng Qung
Ngi, Nh my Bia Qung Ngi - Cty ng Qung Ngi, Cng ty Qung Co
Thi Nay; Ti H Ni c cc nh ti tr: Tp on Viettel, Tp on Du kh
Vit Nam v Ngn hng Nng nghip qun Ty H, th H Ni. Qua s
quan tm ca cc nh ti tr khch l phong tro hc tp mn Ton ca
sinh vin Vit Nam.
Xin cm n i truyn hnh Vit Nam, Trung tm truyn hnh Vit Nam ti
Thnh ph Nng, i pht thanh v truyn hnh Qung Ngi a tin
trong cc chng trnh thi s v t chc truyn hnh trc tip l khai mc
v b mc K thi. Cm n cc phng vin bo i ti a phng v c nc
n d v a tin v cc hot ng ca k thi din ra trong tun qua.

Cc sinh vin c trao gii nht mn i s. Ngun: H Phm Vn ng

Trng i hc Phm Vn ng trn trng cm n Hi Ton hc Vit Nam


tin tng chn trng i hc Phm Vn ng lm n v ng cai cng
nh s phi hp t chc thnh cng cc hot ng ca k thi.

18
Xin cm n s c mt ca 85 on ca cc trng i hc, cao ng trong c
nc v tham d k thi Olympic Ton ln th XXII, nm 2014 ti trng
i hc Phm Vn ng Qung Ngi, gp phn lm nn thnh cng ca k
thi.
Vi t cch l n v ng cai, trng i hc Phm Vn ng c rt
nhiu c gng trong cng tc chun b, t chc k thi, tuy nhin vn khng
th trnh c cc thiu st; rt mong c s thng cm v lng th ca
tt c qu v v cc em sinh vin.
C th ni, K thi Olympic Ton sinh vin ton quc ln th XXII ti Qung
Ngi kt thc tt p, xin hn gp li ti k thi ln th 23, nm 2015 - ti
Trng i hc Kinh t, i hc Hu.
Trng i hc Phm Vn ng xin gi li cho tm bit n tt c cc Thy
c gio v sinh vin cc trng, xin hn gp li ti Qung Ngi trong nhng
dp khc.
Xin knh chc cc ng ch lnh o, qu v i biu, cc v khch qu, cc
Thy c gio v cc em sinh vin lun di do sc khe, hnh phc v thnh
cng trong cuc sng !

Phn II
THI

19

THI CHNH THC

I S

Thi gian lm bi: 180 pht.

Bi 1. a) Chng minh rng:

1 a1 a1 (a1 1)
1 a2 a2 (a2 1)
det
1 a3 a3 (a3 1)
1 a4 a4 (a4 1)

a1 (a1 1)(a1 2)
Y
a2 (a2 1)(a2 2)
=
(aj ai ).
a3 (a3 1)(a3 2)
1i<j4
a4 (a4 1)(a4 2)
Y
b) Gi thit a1 , a2 , a3 , a4 l cc s nguyn, chng minh
(aj ai ) chia
1i<j4

ht cho 12.
Bi 2. Cho cc s thc phn bit a1 , a2 , a3 . Chng minh rng vi mi b s
thc b1 , b2 , b3 tn ti duy nht mt a thc P (x) bc khng qu 5 tha mn:
P (ai ) = P 0 (ai ) = bi , i = 1, 2, 3, y P 0 k hiu o hm ca a thc P .
Bi 3. a) K hiu V4 l khng gian vct cc a thc vi h s thc vi bc
khng qu 4. nh ngha nh x e : V4 V4 nh sau: vi mi a thc f V4 ,
4
X
f (i)
, trong f (i) k hiu o hm bc i ca f , (f (0) = f ). Chng
e(f ) :=
i!
i=0
minh rng e l mt nh x tuyn tnh kh nghch t V4 vo chnh n.
b) K hiu V l khng gian vct cc a thc vi h s thc. Vi mi a thc

X
f (i)
f , t e(f ) :=
. Chng minh rng e l mt nh x tuyn tnh kh
i!
i=0
nghch t khng gian V vo chnh n.


Em B
Bi 4. a) Cho ma trn khi X =
c to thnh t cc ma trn
C En
n v Em , En cp m, n tng ng v cc ma trn B, C vi kch thc m n
v n m tng ng. Chng minh rng
det(X) = det(En CB) = det(Em BC).

22



AB
b) Tng qut, cho ma trn khi X =
, trong A, D l cc ma trn
CD
vung, A kh nghch, chng minh rng det(X) = det(A) det(D CA1 B).
Th sinh chn mt trong hai cu ca bi sau:
Bi 5. a) Cho P l mt a thc bc n vi h s hu t. Gi s s thc a l
mt nghim ca P vi bi > n/2. Chng minh rng a l mt s hu t.
b) Trn hnh vung ABCD ta nh ngha ng i gia hai nh X, Y (khng
nht thit phn bit) l mt dy cc nh k nhau XX1 X2 . . . Xn1 Y . Nh vy
X0 = X, X1 , . . . , Xn1 , Xn = Y l cc nh ca hnh vung v Xi Xi+1 l cnh
ca hnh vung, s n c gi l di ca ng i. Vi mi s t nhin n,
gi xn , yn , zn tng ng l s cc ng i di n gia: mt nh v chnh
n, mt nh v mt nh c nh k n, mt nh v nh i din (nh
i xng qua tm). V d, x0 = 1, y0 = 0, z0 = 0, x1 = 0, y1 = 1, z1 = 0, x2 =
2, y2 = 0, z2 = 2.
1. Thit lp cng thc truy hi cho xn , yn , zn .
2. Tm cng thc tng qut ca xn , yn , zn .

2. GII TCH

23

GII TCH

Thi gian lm bi: 180 pht.

p
Bi 1. Cho dy s (un ) tha mn u1 = 1 v un+1 = u2n + an , n 1, trong
a 0. Tm a sao cho (un ) hi t v tm gii hn .
Bi 2. Cho hai hm f (x) v g(x) xc nh trn R v tha mn iu kin



f (x) f (y) g(x) g(y) = 0,
vi mi x, y R. Chng minh t nht mt trong hai hm f hoc g l hm
hng.
Bi 3. 1) Cho hm s f n iu trn [0, ) v
1
lim
x+ x

Zx
f (t)dt = +.
0

Chng minh rng


lim f (x) = +.

x+

2) Kt lun trn cn ng khng khi f l hm lin tc trn [0, ) nhng


khng n iu trn khong ? Ti sao?
Bi 4. Tm tt c cc hm s f (x) xc nh, lin tc trn on [0, 1], kh vi
trong khong (0, 1) v tha mn iu kin
f (0) = f (1) =

2015
;
2014

2013f 0 (x) + 2014f (x) 2015 x (0, 1).

Bi 5. Cho dy s (xn ) c xc nh bi xn+2 =


Tm lim xn vi iu kin x0 4; x1 4.

xn+1 +

xn , n 0.

Bi 6. Th sinh chn mt trong hai cu:


6a. Cho (an ) l dy s xc nh bi

a1 = 3 6, a2 = 3

r
q
q

6 + 6, . . . , an = 3 6 + 6 + ... + 6 .
|
{z
}
n ln

24
Hy chng minh rng chui s

an hi t.

n=1

6b. Cho f l hm s lin tc trn [0, +). Gi s rng


Z x
x3
f 2 (t) dt , x 0.
3
0
Z x
x2
f (t)dt
vi mi x 0.
Chng minh rng
2
0

CC BI XUT: I S

MA TRN

1 1 0
Bi 1.1 (H An Giang). Cho ma trn A = 0 1 1 . Hy tm tt c cc
0 0 1
ma trn X M3 (R) sao cho X giao hon vi A.
Bi 1.2 (H An Giang). Cho ma trn

0 0
0 0
A=
1 0
1 1

1
0
0
0

1
1
.
0
0

Hy tnh A2014 .
Bi 1.3 (H An Giang). Cho K l mt trng v A, B Mn (K) tha mn
A2 = A + B + BA. Chng minh rng AB = BA.
Bi 1.4 (H Bc Liu). Cho ma trn A M2 (R) c hai gi tr ring phn
bit l v . Chng minh rng tn ti X, Y M2 (R) sao cho vi mi s t
nhin n, An = n X + n Y .
Bi 1.5 (H Bc Liu). Cho M, N M3 (R) tha mn

5 6 2
M N = 6 7 2 .
6 6 1
Chng minh N M kh nghch v tm ma trn nghch o ca N M .
Bi 1.6 (H Bch Khoa H Ni). Cho A l ma trn vung cp n v A l ma
trn ph hp ca A. Chng minh rng, nu A l ma trn kh nghch v A c
mt hng gm ton s 1 th tng tt c cc phn t ca A l khc 0.

1 1 1
Bi 1.7 (H Bch Khoa Tp. H Ch Minh). Cho A = 1 1 0. Tnh A14 .
1 0 1

26
Bi 1.8 (H Bch Khoa Tp. H Ch Minh). Cho A, B Mn (R) tha mn
AB = 0, B 6= 0. Chng minh rng tn ti C Mn (R) khc 0 tha mn
AC = CA = 0.
Bi 1.9 (H Bch Khoa Tp. H Ch Minh). Tm tt c cc ma trn A M2 (R)
tha mn A = A> = A1 .
Bi 1.10 (H Bch Khoa Tp. H Ch Minh). Cho A Mn (Z) (cc phn t
nguyn) tha mn I + A + A2 + + A100 = 0. Tm tt c cc s nguyn
dng k 100 tha mn det(Ak + Ak+1 + + A100 ) = 1.
Bi 1.11 (H Bch Khoa Tp. H Ch Minh). Cho A, B Mn (R) l hai ma
trn i xng thc. Chng minh rng: nu tn ti hai ma trn X, Y tha
AX + BY kh nghch th A2 + B 2 kh nghch.
Bi 1.12 (HV Bu chnh Vin thng).
M23 (R) tha mn

2
AB =
2

Cho ma trn A M32 (R) v B

2 2
5 4 .
4 5

a. Tnh (AB)2 .
b. Chng minh rng ma trn BA kh nghch.
c. Tm ma trn BA.
Bi 1.13 (HV Bu chnh Vin thng). Cho A, B l hai ma trn i xng cp
n, chng minh rng Tr((AB)2 ) Tr(A2 B 2 ). Du bng xy ra khi no?
Bi 1.14 (H Cng ngh thc phm Tp. HCM). Cho A, B l hai ma trn
vung cp n sao cho AB = A + B. Chng minh rng rank(A) = rank(B).
Bi 1.15 (H Cng ngh thc phm Tp. HCM). Cho A l ma trn vung cp
n 2 sao cho A kh nghch v c cc h s u dng. Gi zn l s h s 0
ca ma trn A1 . Chng minh rng zn n2 2n.
Bi 1.16 (D b). Ma trn thc A = (aij ), 1 i, j n c gi l ma trn
thc s ngu nhin nu tho mn cc tnh cht sau:
1) aij > 0 vi mi i, j;
2) ai1 + ai2 + + ain = 1 vi mi 1 i n.
Gi A l mt ma trn nh vy. Gi  > 0 l gi tr nh nht ca cc h s
ca A.
1. Chng minh rng Ak l mt ma trn thc s ngu nhin vi mi k 1;

1. MA TRN

27
(k)

(k)

2. Vi mi k 1, 1 j n, k hiu j (tng ng, j ,) l gi tr nh


nht (tng ng, gi tr ln nht) ca cc h s trn ct th j ca Ak .
Chng minh rng:

(k)

(k+1)

j j
(k+1)

v j

(k+1)

(k+1)

(k)

(k)

j ,
(k)

(1 2)(j j ).

3. Chng minh rng lim Ak tn ti v gii hn cng l mt ma trn ngu


k

nhin.
Bi 1.17 (H ng Thp). Cho A, B l hai ma trn thc vung cp n. Chng
minh rng nu ma trn I + AB kh nghch th ma trn I + BA cng kh
nghch.


a b
Bi 1.18 (H Hng Vng, Ph Th). Tm tt c cc ma trn A =
 ck d k 
a b
vi cc phn t thc sao cho vi mi s nguyn dng k, ta c Ak =
.
ck d k
Bi 1.19 (H Hng Vng, Ph Th). Tm tt c ma trn nghch o ca ma
trn sau:

0
a
a 2 a3
a1 0
a a2
.
A=
2
1
a
a
0
a
a3 a2 a1 0
Bi 1.20 (H Hng Vng, Ph
Th). Gi s A
M34 (R), B M42 (R), C
0 1 1
M23 (R) sao cho ABC = 1 0 1 . Tnh CAB v chng minh
1
1
2
2
(BCA) = BCA.

2 4 1
4 1 . Chng minh
Bi 1.21 (C Ng Gia T). Cho ma trn A = 2
2
2
1
A2014 = (22013 1)A2 + (2 22013 )A.
Bi 1.22 (C Ng Gia T). Cho A v B l hai ma trn vung cng cp n.
Chng minh rng rank(AB I) rank(A I) + rank(B I).

28
Bi 1.23 (H Ngoi Thng - H Ni). Cho {un }, {vn }, {wn } l cc dy s
thc c xc nh bi u0 = v0 = w0 , v:

un+1 = un 7vn + 5wn ,


vn+1 = 2un 8vn + 6wn ,

wn+1 = 4un 16vn + 12wn .


a. Hy tnh s hng tng qut ca cc dy s {un }, {vn }, {wn }.
b. Chng minh rng vn 2 chia ht cho 2n vi mi s nguyn dng n.
Bi 1.24 (H Ngoi Thng - H Ni). Cho s nguyn dng n. Gi A l ma
trn vung cp n c cc phn t nm trn ng cho chnh bng 0 v cc
phn t cn li bng 1 hoc n 1. Hy tm tt c cc gi tr c th xy ra i
vi hng ma trn A.
Bi 1.25 (H Ngoi Thng - H Ni). Cho A M2 (Q) (tp hp cc ma trn
vung cp 2 vi h s hu t) tha mn: tn ti n N sao cho An = I2 .
Chng minh rng A2 = I2 hoc A3 = I2 .
Bi 1.26 (H Ngoi Thng - H Ni). t
K2 (Z) = {A|A M2 (Z) v

det A = 1}.

a. Tn ti hay khng A, B, C K2 (Z) sao cho A2 + B 2 = C 2 ?


b. Tn ti hay khng A, B, C K2 (Z) sao cho A4 + B 4 = C 4 ?
Bi 1.27 (H Nng nghip H Ni). Ta nh ngha vt ca ma trn vung
n
P
C = (cij )nn , k hiu l tr(C), l tng
cii . Cho A, B l hai ma trn thc
i=1

vung cp n tha mn AB BA = A. Tnh vt ca ma trn A2014 .


Bi 1.28 (H Phm Vn ng). Cho ma trn vung

3 0 2
A = 0 1 2 .
2 2 2
Tnh A2014 .
Bi 1.29 (H Phm Vn ng). Ta gi vt ca ma trn A, k hiu bi Tr(A),
l tng tt c cc phn t trn ng cho chnh ca A. Cho A = (aij )nn l
ma trn vung cp n tha mn A2014 = 0. Tnh Tr(A).

1. MA TRN

29

Bi 1.30 (HV Phng khng Khng qun). Cho ma trn vung cp n (cc
phn t nm trn ng cho chnh bng 2015; cc phn t cn li bng
2014):

2015 2014 2014 . . . 2014


2014 2015 2014 . . . 2014

A=
............................
2014 2014 2014 . . . 2015
Hy tnh Ak , vi k l s nguyn cho trc.
Bi 1.31 (H Qung Nam). Cho cc ma trn vung (cp n) A, B, C, D tha
BAt v DC t l ma trn i xng v DAt CB t = I. Chng minh rng
Dt A B t C = I.
Bi 1.32 (H Quy Nhn). a. Vi mi ma trn A M2 (R), xc nh ma trn
B, C M2 (R) sao cho
A = B2 + C 2
b. Tn ti hay
 khng
 cc ma trn B, C M2 (R) sao cho BC = CB v
0
1
B2 + C 2 =
.
1 0
Bi 1.33 (H Quy Nhn). Cho A, B l hai ma trn vung cp 2 tha mn
(AB)2 = 0. Chng minh rng (BA)2 = 0. Khng nh trn c cn ng cho
trng hp A, B l cc ma trn vung c cp ln hn 2 khng?
Bi 1.34 (H Sao ). Cho ba dy s {xn } , {yn } , {zn } xc nh bi x0 =
y0 = z0 = 1 v

xn+1 = xn 3yn + 3zn


yn+1 = 3xn 5yn + 3zn

zn+1 = 6xn 6yn + 4zn .


Tm x2014 .
Bi 1.35 (H Sao ). Cho A =

!
p

10 + 2 5 p 5 1

. Tm An vi
1 5
10 + 2 5

n N .
Bi 1.36 (H Sao ). Cho A l ma trn vung tha mn Am = 0 vi m N .
Chng minh rng
rankA = rank(A + A2 + ... + An ).
Bi 1.37 (H S phm H Ni 2). Xc nh tt c cc ma trn X, Y tha
mn


1 0
XY X = Y XY =
.
0 1

30
Bi 1.38 (H S phm H Ni 2). Cho A Mn (R), n > 2015 sao cho trn
mi dng ca ma trn c ng 2015 phn t khc khng, trong phn t
nm trn ng cho chnh thuc vo khong (, 2014) ng thi cc
phn t cn li u thuc vo (1, 1). Ma trn A c l ma trn kh nghch
hay khng?
Bi 1.39 (H S phm Thi Nguyn). Tnh A2014 vi A l ma trn

2014 0 0 0 0 2014
0
1 0 0 1
0

0
1
1
0
0

.
0
0 1 1 0
0

0
1 0 0 1
0
2014 0 0 0 0 2014
Bi 1.40 (H S phm Thi Nguyn). Cho A, B, C Mn (R). Chng minh
rng
a. tr(AA> ) 0.
b. tr(A(A> B > )) + tr(B(B > C > )) + tr(C(C > A> )) 0.
Bi 1.41 (C S phm Vnh Phc). Cho A l ma trn vung cp n, tha mn
A2014 = 0. Chng minh rng B = I A1007 A1008 A2013 l ma trn
kh nghch.
Bi 1.42 (H Tn Tro). Cho A l ma trn cp n n trn trng K. Chng
minh rng tn ti hai ma trn U, V cp n 1 trn K sao cho A = U V > v
Tr(A) = V > U , A2 = Tr(A)A.
Bi 1.43 (H Tn Tro). Cho

a b
b a

b b
A=
. .

b b
b b

b
b
a
.
b
b

...
...
...
...
...
...

b
b
b
.
a
b

b
b

b
Mk (R).
.

b
a

Hy tnh An (n N, n > 1),

NH THC

Bi 2.1 (H An Giang). Cho A M2 (R) sao cho


det(A) = d 6= 0 v det(A + dA ) = 0,
trong A l ma trn ph hp ca A. Chng minh rng det(A dA ) = 4.

2. NH THC

31

Bi 2.2 (HV An ninh nhn dn). Cho cc s thc ai , (i = 1, 2, . . . , n), tha


mn ai + aj 6= 1 (i, j = 1, 2, . . . , n). Xt ma trn A = [aij ] cp n vi
aij =

1
.
1 + ai + aj

Chng minh rng det (A) 0.


Bi 2.3 (H Bc Liu). Tnh nh thc




1
1

1


F1 (cos 1 )
F1 (cos 2 ) F1 (cos n )

Dn =
,
. . . . . . . . . . . . . . . . . . . . . . . . . . . . . . . . . . . . . . . . . . . .
Fn1 (cos 1 ) Fn1 (cos 2 ) Fn1 (cos n )
vi Fk (x) = a0k xk + a1k xk1 + + akk .
Bi 2.4 (HV Bu chnh Vin thng). Cho ma trn A = [aij ] vung cp n v lu
linh (ma trn A c gi l ly linh nu tn ti k N sao cho Ak = 0). Chng
minh rng vi mi ma trn B giao hon vi A, ta c det(A + B) = det(B).
Bi 2.5 (HV Bu chnh Vin thng). Cho ma trn A = [aij ] vung cp n c
vt bng 2014 v hng bng 1. Tnh det(A + I).
Bi 2.6 (H Cng ngh thc phm Tp. HCM). Cho A, B, C l cc ma trn
vung cp 3 tha mn cc iu kin: AB = BA; AC = CA; BC = CB v
A + B + C = I. Chng minh rng
det[A3 + B 3 + C 3 3(A + I)(B + I)(C + I)] = 1.
Bi 2.7 (H Cng ngh thc phm Tp. HCM). Chng minh rng


1 x1 x21 . . . xn2
xn1
1

1 x2 x22 . . . xn2
xn2
2

. . . . . . . . . . . . . . . . . . . . . . . . .


1 xn x2 . . . xn2 xn
n
n
n


1 x1 x21 . . . xn2

xn1
1
1


n1
1 x2 x22 . . . xn2
x
2
2
.
= (x1 + x2 + + xn )
. . . . . . . . .
. . . . . . . . .2. . . . . . . . .n2
1 xn x . . . x

xn1
n
n
n
Bi 2.8 (D b). Cho S l mt tp hp c lc lng n v X1 , . . . , Xm l
mt h cc tp con i mt phn bit ca S sao cho cc tp Xi Xj c
cng lc lng k 1 vi mi i 6= j. K hiu x1 , . . . , xn l cc phn t ca
X v d1 , . . . , dm tng ng l lc lng ca X1 , . . . , Xm . Xy dng ma trn
A = (ai,j ) Mnm (R) nh sau: aij = 1 nu xi Xj .

32
a. Xc nh ma trn A.A> .
b. Tnh nh thc ca ma trn A> A.
c. Chng minh rng m n.



1 2
Bi 2.9 (H ng Thp). Cho A =
v f (x) = x2 4x + 6,
1 4
g(x) = x2014 + x2 5x + 6. Tnh det(g(A)) v [f (A)]2014 .
Bi 2.10 (H ng Thp). Cho ai , bi , i = 1, 2, 3, 4 l cc s nguyn. K hiu


a1 a2
a3
a4

a a1 a4 a3

D(a1 , a2 , a3 , a4 ) = 2

a3 a4 a1 a2
a4 a3 a2 a1
S dng tch D(a1 , a2 , a3 , a4 )D(b1 , b2 , b3 , b4 ), chng minh rng nu hai s
nguyn l tng cc bnh phng ca bn s nguyn th tch ca chng cng
l tng cc bnh phng ca bn s nguyn.
Bi 2.11 (H Hng Vng, Ph Th). Cho A = (aij )4201442014 l mt ma trn
vung cp 42014 vi aii {8, 10, 12, 14}(i = 1, 40214), aij {7, 9, 11, 13}(i 6=
j, i, j = 1, 40214). Chng minh rng det A 6= 0.
Bi 2.12 (C Ng Gia T). Cho A, B l cc ma trn vung cp n tha mn
AB BA = B. Chng minh rng det(B) = 0.
Bi 2.13 (H Ngoi Thng - H Ni). Tnh nh thc cp n sau:


x

1
0
0
.
.
.
0
0


n1
x
2 0 . . . 0 0

n 2 x 3 . . . 0 0
Dn (x) = 0
.............................


0
0
0 0 ... 1 x
Bi 2.14 (HV Phng khng Khng qun). Cho A l ma trn vung cp n.
Chng minh rng det(A4 + 4In ) 0, trong In l ma trn n v cp n.
ng thc xy ra khi no?
Bi 2.15 (HV Phng khng Khng qun). Tnh nh thc ca ma trn

1
2
3
. . . 2013 2014
2
1
2
. . . 2012 2013

2
1
. . . 2011 2012
A= 3
.
..................................
2013 2012 2011 . . .
2
1

2. NH THC

33

Bi 2.16 (H Qung Nam). Cho cc ma trn thc vung A, B tha mn


A> A = I v B > B = I. Bit detA 6= detB. Chng minh rng det(A + B) = 0.
Bi 2.17 (H Qung Nam). Cho cc ma trn thc vung A, B, C i mt
giao hon. Chng minh rng tn ti cc s thc a, b, c khng ng thi bng
0 sao cho
det(aA + bB + cC) = 0.
Bi 2.18 (H Quy Nhn). Cho A = [aij ] l ma trn vung cp n c xc
nh nh sau:

0 nu ij + j i = 3k,
aij = 1 nu ij + j i = 3k + 1,

2 nu ij + j i = 3k + 2.
Tm s t nhin n ln nht sao cho det (A) 6= 0.
Bi 2.19 (H Sao ). Tnh nh thc


2014
2014
2014
0
1
2
3
...
2014


1
0
1
22014
... 20132014
2014
2
1
0
1
... 20122014 .

. . . . . . . . . . . . . . . . . . . . . . . . . . . . . . . . . . . . . . . . . . . . . . . . . .


20142014 20132014 20122004 20112014 ...
0
Bi 2.20 (H S phm H Ni 2). Cho A M2014n (R), n 1. Chng minh
rng
det (AA> + 4I2014 ) 24028 .
Bi 2.21 (H S phm Thi Nguyn). Cho A M2 (R), B M3 (R). Chng
minh rng
1
a. det(A) = [(tr(A))2 tr(A2 )].
2
1
b. det(B) = [(tr(B))3 3tr(B)tr(B 2 ) + 2tr(B 3 )].
6
Bi 2.22 (H S phm Thi Nguyn). Tnh nh thc


a1 1 ... 1

1


1 a2 ... 1
1

... ... ... ...
...

1 1 ... ...
1

1 1 ... 1 a2014
Bi 2.23 (C S phm Vnh Phc). Cho A = (aij ) l ma trn vung cp
n(n 2) sao cho mi phn t aij u c dng p2 (p l s nguyn t ln
hn 3). Chng minh rng det(A) chia ht cho 24n1 .

34
Bi 2.24 (H Tn Tro). Tnh nh thc


a1 x x ... x

x


y a2 x ... x

x


y y a3 ... x
x

D=
.
. . . . . . . . . . . . . . . . . . . . . . . .
y y y ... an1 x


y y y ...
y
an

H PHNG TRNH TUYN TNH

Bi 3.1 (H An Giang). Cho A = [aij ] Mn (R) tha mn A2 = A. Hy gii


h phng trnh

a11 x1 + a12 x2 + + a1n xn = x1

a12 x1 + a22 x2 + + a2n xn = x2


.....................................

an1 x1 + an2 x2 + + ann xn = xn .


Bi 3.2 (H Bc Liu). Cho h phng trnh tuyn tnh

x1 + x2 + x3 + x4 kx5

x1 x2 + kx3 + x4 + kx5
x1 + kx2 x3 + x4 x5

kx1 + kx2 x3 x4 x5

=
=
=
=

0
0
0
0,

trong x1 , x2 , x3 , x4 , x5 l cc n v k l tham s thuc R. Chng minh rng,


nu k 6 {1; 3} v (a, b, c, d, e) l mt nghim ca h th a(3 k) = e(8k 2);
b(3 k) = e(k + 1); c(3 k) = e(2k 2) v d(k 3) = e(k 1)2 .
Bi 3.3 (H Bch Khoa H Ni). Cho aij l cc s nguyn, gii h phng
trnh

x1
a11 x1 + a12 x2 + + a1n xn =

x2
a21 x1 + a22 x2 + + a2n xn =
n

.............................................

an1 x1 + an2 x2 + + ann xn = 1 xn


n

3. H PHNG TRNH TUYN TNH

35

Bi 3.4 (H Hng Vng, Ph Th). Gii h phng trnh sau

x1
+x2
+x3 = 2013
x2
+x3
+x4 = 2013
x3
+x4
+x5 = 2013
x4
+x5
+x6 = 2013
..............................

x2011 +x2012 +x2013 = 2013

x2012 +x2013 +x2014 = 2013

x
+x2014 +x1 = 2013

2013
x2014 +x1
+x2 = 2013
Bi 3.5 (C Ng Gia T). Gii h phng trnh

x1 + 2x2 + + 2014x2014 =
1

x2 + 2x3 + + 2014x1 =
2
.
.
.
.
.
.
.
.
.
.
.
.
.
.
.
.
.
.
.
.
.
.
.
.
.
.
.
.
.
.
.
.
.
.
.
.
.
.
.
.
.
.
.
...

x2014 + 2x1 + + 2014x2013 = 2014


Bi 3.6 (H Phm Vn ng). Cho cc s thc aij + aji = 0, vi i, j =
1, 2, ..., 2014. Hy gii h phng trnh tuyn tnh

(a11
+2014)x1
+a12 x2
+...
+a1,2014 x2014
=0

a21 x1
+(a22
+2014)x2
+...
+a2,2014 x2014
=0
..................................................................

a2014,1 x1 +a2014,2 x2
+...
+(a2014,2014 +2014)x2014 = 0
Bi 3.7 (H Sao ). Cho 3n s nguyn a1 , a2 , ..., an ; b1 , b2 , ..., bn ; c1 , c2 , ..., cn
tha mn a1 b1 + a2 b2 + ... + an bn = 0. Chng minh rng h phng trnh sau
lun c nghim

a1 b1 x +a2 b2 x +... +an bn x = c1 x1

a2 b1 x +a2 b2 x +... +a2 bn x = c2 x2


........................................

an b1 x +an b2 x +... +an bn x = cn xn


Bi 3.8 (H S phm H Ni 2). Gii h phng trnh sau:

1
x1
x1
+2x2
+...
+2014x2014
= 2014

1
x1
x1
+2x2
+...
+2014x2014
= 2014
................................................

1
x1 +22014 x2 + . . . +20142014 x2014 = 2014
x2014

36
Bi 3.9 (H S phm Thi Nguyn). Cho a thc
P (x) = (x 2)(x 3)...(x 2014).
Gi s P (x) = a1 + a2 x + ... + a2014 x2013 . Gii h phng trnh sau:

a1 x1 +a2 x2 +... +a2013 x2013 +a2014 x2014 = 0

a2014 x1 +a1 x2 +... +a2012 x2013 +a2013 x2014 = 0


...
...
...
...
...
...

a2 x1 +a3 x2 +... +a2014 x2013 +a1 x2014 =


0
Bi 3.10 (H Tn Tro). Cho h phng trnh

x1 +x2 = 2a1

x2 +x3 = 2a2

x3 +x4 = 2a3
.
.
..................

x
+xn = 2an1

n1
xn +x1 = 2an
a. Gii h phng trnh vi n = 4, n = 5
b. Gii h phng trnh vi mi n N.

KHNG GIAN VC T V NH X TUYN TNH

Bi 4.1 (HV An ninh Nhn dn).

a1
a2

a3

A = ..
.

an1
an

Cho ma trn A Mn (R) nh sau

1 0... 0 0
0 1
... 0 0

0 0
... 0 0

..
..
. . . .. ..
.
.
. .

0 0
... 0 1
0 0
... 0 0

a. Hy tm mt vc t x Rn sao cho h vc t {x, Ax, A2 x, . . . , An1 x} c


lp tuyn tnh.
b. Gi s ma trn A ng dng vi ma trn ng cho diag(b1 , b2 , . . . , bn ).
Chng t rng cc s b1 , b2 , . . . , bn khc nhau tng i mt.
Bi 4.2 (H Bch Khoa H Ni). Cho V1 v V2 l hai khng gian con ca
khng gian vect V hu hn chiu. Chng minh rng, nu dim(V1 + V2 )
dim(V1 V2 ) = 1 th V1 V2 l khng gian con ca V .

4. KHNG GIAN VC T V NH X TUYN TNH

37

Bi 4.3 (H Bch Khoa Tp. H Ch Minh). Cho H l tp cc ma trn thc


cp 3 c tng cc phn t trn mi hng bng nhau v bng tng cc phn
t trn mi ct. Chng mnh rng H l khng gian con ca R3 . Tm s chiu
ca H.
Bi 4.4 (H Bch Khoa Tp. H Ch Minh). Cho 3 vc t X1 , X2 , X3 R3
c lp tuyn tnh. Tm tt c cc ma trn thc A tha AX1 = X2 , AX2 =
X3 , AX3 = X1 .
Bi 4.5 (HV Bu chnh Vin thng). Cho P1 (x), P2 (x), ..., Pr (x) l cc a thc
bc n1 , n2 , ..., nr . Chng minh rng nu
n1 + n2 + ... + nr <

r(r 1)
2

th P1 (x), P2 (x), ..., Pr (x) ph thuc tuyn tnh.


Bi 4.6 (H Cng ngh thc phm Tp. HCM). Cho A l ma trn ng cho
cp n trn R c a thc c trng l
p(t) = (x c1 )r1 (x c2 )r2 . . . (x ck )rk ,
vi ci 6= cj , i 6= j. Gi V l khng gian cc ma trn B Mn (R) sao cho
AB = BA. Chng minh rng dim V = r12 + r22 + + rk2 .
Bi 4.7 (D b). Mt a thc theo n bin x1 , x2 , . . . , xn c gi l thun
nht bc d nu tng cc s m ca cc bin trong tng n thc ca a thc
bng d. V d x21 x32 + x53 l a thc thun nht bc 5 theo ba bin x1 , x2 , x3 .
Tm s chiu ca khng gian vc t thc bao gm a thc 0 v cc a thc
thun nht bc d theo n bin vi h s thc.
Bi 4.8 (D b). Cho V l mt khng gian vct n chiu trn trng s thc
v f l mt t ng cu tuyn tnh ca V . Vi mi s nguyn khng m k, k
hiu f k hp thnh k ln ca f vi chnh n (nh vy f 0 = IdV , f 1 = f, f 2 =
f f, . . .. Chng minh rng
1. Tn ti mt s nguyn khng m m n sao cho

Im f 0 ) Im f 1 ) Im f 2 ) ) Im f m = Im f m+1 = Im f m+2 =
ker f 0 ( ker f 1 ( ker f 2 ( ( ker f m = ker f m+1 = ker f m+2 =
2. {dim ker f k+1 dim ker f k }k=0,1,... l mt dy s gim;
3. V = ker f m Im f m ;

38
4. Mi ma trn vung
cp

 n vi h s thc u tng ng vi mt ma
N 0
trn c dng
, trong N l ma trn vung lu linh v C l
0 C
ma trn vung kh nghch.
Bi 4.9 (D b). Cho mt tp hp X. Ta trang b cho khng gian F (X, R)
cu trc R-khng gian vct cm sinh t cu trc khng gian vct ca R.
Cho n 1 phn t f1 , . . . , fn F (X, R). Chng minh rng f1 , . . . , fn l mt
h c lp tuyn tnh khi v ch khi tn ti n phn t x1 , . . . , xn X sao cho
ma trn (fi (xj ))1i,jn l kh nghch.
Bi 4.10 (D b).
1. Cho f l mt t ng cu tuyn tnh ca khng gian
n
C . Chng minh rng hai iu kin sau l tng ng
i) f cho ho c;
ii) f 2 cho ho c v ker f = ker f 2 .
2. Tm iu kin cn v trn b cc s phc (a1 , . . . , an ) ma trn

0 0 an
0 0 0

.. ..
..
. . .

0 a2 0
a1 0 0
cho ho c?
Bi 4.11 (H ng Thp). Cho V l mt khng gian vc t trn mt trng
K. Cho F1 , F2 , . . . , Fn (n N, n 1) l cc khng gian con hu hn sinh ca
V . Chng minh rng
dim(F1 + F2 + + Fn ) dim F1 + dim F2 + + dim Fn .
Hn na, du bng xy ra khi v ch khi F1 + F2 + + Fn l tng trc tip
trong V .
Bi 4.12 (H Ngoi Thng - H Ni). Cho V1 , V2 , V3 l ba khng gian vc t
con ca khng gian vc t 2014 chiu V trn trng s thc R. Chng minh
rng nu dim V1 + dim V2 + dim V3 > 4028 th V1 V2 V3 6= 0.
Bi 4.13 (H Nng nghip H Ni). Cho E l mt R-khng gian vct hu
hn chiu v mt t ng cu u ca E gi l ly linh nu c s nguyn dng
k uk = 0. Ta nh ngha bc ly linh ca mt t ng cu ly linh u l s
nguyn dng p b nht m up = 0.
Cho u l mt t ng cu ly linh bc p. Chng minh rng tn ti mt vct
x ca E sao cho h vct {x, u(x), . . . , up1 (x)} c lp tuyn tnh.

5. GI TR RING V VC T RING

39

Bi 4.14 (H S phm H Ni 2). Cho E l khng gian vc t trn trng s


thc, mt t ng cu tuyn tnh f ca E c gi l t ng cu tuyn tnh
ly linh nu tn ti mt s k N sao cho f k = 0 l t ng cu tuyn tnh
khng (s k nh nht tha mn iu kin ny c gi l cp ly linh ca
f ). Chng minh rng nu f l t ng cu tuyn tnh ly linh cp p ca E th
h {idE , f, . . . , f p1 } l c lp tuyn tnh trong khng gian vc t End(E).
Bi 4.15 (C S phm Vnh Phc). Xt V l khng gian cc hm thc lin
tc trn on [a, b] v gi s f1 , f2 , . . . , fn V sao cho
det

 Zb


fi (x)fj (x)dx = 0.

Chng minh rng h vct {f1 , f2 , . . . , fn } ph thuc tuyn tnh trn V .

GI TR RING V VC T RING

Bi 5.1 (H Bch Khoa H Ni). Cho A, B l hai ma trn vung cp n, trong


B ly linh v AB = BA. Chng minh rng l tr ring ca A khi v ch
khi l tr ring ca A + B.
Bi 5.2 (H Bch Khoa H Ni). Cho ma trn A = (aij ) vung cp n, trong
 ij
b , nu i 6= j
vi b 6= 0. Tm cc tr ring ca A v tnh det(A).
aij =
0, nu i = j
Bi 5.3 (C Ng Gia T). Cho A, B l cc ma trn vung cp n tha mn
A A2 + B B 2 = AB. Chng minh rng A + B c gi tr ring l 1 khi v
ch khi A hoc B c gi tr ring l 1.
Bi 5.4 (H Sao ). Cho A, B l cc ma trn vung cp n.
a. Chng minh rng tp cc gi tr ring ca AB v BA trng nhau.
b. A, B l hai ma trn giao hon v ly linh th A + B ly linh.
Bi 5.5 (H Tn Tro). Gi s A l mt ma trn vung trn C c tnh cht:
A A = AA (A l ma trn lin hp ca ma trn A). Chng minh rng nu
x l mt vc t ring ca A ng vi gi tr ring th x cng l vc t ring
ca A ng vi gi tr ring ( l s phc lin hp ca s phc ).

A THC

Bi 6.1 (H Bch Khoa H Ni). Cho a thc f (x) bc n, tha mn f (x)


n
P
0, x R. Chng minh rng
f (k) (x) 0, x R.
i=1

40
Bi 6.2 (HV Bu chnh Vin thng). Cho a thc h s thc P (x) = x2014 +
a2013 x2013 +...+a1 x+a0 ch c cc nghim thc l x1 , x2 , ..., x2014 . Chng minh
rng vi mi s thc a ln hn tt c cc nghim thc y th ta c bt ng
thc
2013
P 0 (a) 2014(P (a)) 2014 0.
Du bng xy ra khi no?
Bi 6.3 (HV Bu chnh Vin thng). Cho a thc f (x) = x3 3x + 1. Tm s
nghim thc phn bit ca phng trnh f (f (x)) = 0.
Bi 6.4 (H Cng ngh thc phm Tp. HCM). Tm tt c cc a thc P (x)
c h s nguyn sao cho
P (P 0 (x)) = P 0 (P (x)), x R.
Bi 6.5 (H Hng Vng, Ph Th). Tm tt c cc a thc P (x) vi h s
thc bit: P (x)P (x + 1) = P (x2 + x + 1).
Bi 6.6 (H Hng Vng, Ph Th). Tm tt c cc a thc P (x) vi h s
thc bit: P (x)P (x 1) = P (x2 ).
Bi 6.7 (C Ng Gia T). Cho a thc
f (x) = x3 + x2 + x + .
Hai ngi chi tr chi nh sau: mi lt chi, ngi chi vit mt s
nguyn khc 0 bt k vo mt v tr c du . Nu sau khi thay xong cc du
bi cc s nguyn m f (x) c 3 nghim nguyn th ngi i trc thng,
nu ngc li th ngi i sau thng. Hy ch ra cch chi ngi i trc
lun thng.
Bi 6.8 (H Nng nghip H Ni). Tm tt c cc a thc P (x) tha mn:

2
P (P (x)) + 1 = P 2 (x) + 2P (x) + (x2 + 3x + 1)2 . (1)
Bi 6.9 (H Phm Vn ng). Cho a thc P (x) = xn + xn1 + an2 xn2 +
... + a1 x + a0 c n nghim thc x1 , ..., xn i mt phn bit. Chng minh rng
xn1
xn2
xnn
+
+
....
+
= 1,
P 0 (x1 ) P 0 (x2 )
P 0 (xn )
trong P 0 (x) l o hm ca P (x).

6. A THC

41

Bi 6.10 (HV Phng khng Khng qun). Cho cc a thc vi h s phc:


P (x) = xn + a1 xn1 + a2 xn2 + . . . + an1 x + an
Q(x) = xm + b1 xm1 + b2 xm2 + . . . + bm1 x + bm ,
bit rng P (x) chia ht cho Q(x) v tn ti k (k = 1, 2, 3, . . . , m) sao cho
k
|bk | > 2014k Cm
. Chng minh rng tn ti t nht ai (i = 1, 2, . . . , n) sao cho
|ai | > 2013
Bi 6.11 (H Qung Nam). Cho cc a thc P (x), Q(x) c cc h s nguyn
v P (x) chia ht cho Q(x) trn vnh Z[x]. Chng minh rng nu a thc
P (x) 2014 c 33 nghim nguyn phn bit th deg Q(x) 5.
Bi 6.12 (H S phm H Ni 2). C tn ti hay khng a thc P (x), Q(x)
R[x] sao cho:
P (x)
= x2 + 4x + 2014.
Q(x)
Bi 6.13 (H S phm Thi Nguyn). Cho P (x) l a thc khc hng vi
h s thc. Gi s rng tt c cc nghim ca P (x) u l thc. Chng minh
rng
(P 0 (x))2 P (x)P 00 (x) vi mi x R.
Bi 6.14 (C S phm Vnh Phc). Cho a thc f (x) R[x] bc 2014 c
2014 nghim thc. Gi s F (x) l nguyn hm ca f (x) v F (x) c t nht
2014 nghim thc. Chng minh rng vi mi R, a thc F (x) + f (x) c
2015 nghim thc.
Bi 6.15 (H Tn Tro). Cho a, b R. Tm tt c cc a thc P (x) tha mn
iu kin
xP (x a) = (x b)P (x), x R.

CC BI XUT: GII TCH

DY S

Bi 1.1 (HV Bu chnh Vin thng). Cho dy s


xk =

1
2
k
+ + +
2! 3!
(k + 1)!

vi k = 1, 2, , 2014. Tm gii hn
p
I = lim n xn1 + xn2 + + xn2014 .
n

Bi 1.2 (H Cng ngh thc phm Tp. HCM). .


a) Tm
q

lim

2n
Y

in+

n2

+ (i n)

n2n

i=1

b) Cho hai dy s {an } v {bn } c xc nh nh sau




b1 =
b
a1 =
a
v
an+1 +bn
an +bn
an+1 =
bn+1 =
2
2
Chng minh rng
lim an = lim bn .

Bi 1.3 (D b). Cho dy s tha mn


(
x1
= 1,
xn+1 = 2xn + 3n + 2n , vi n = 1, 2, ...
Hy tm s hng x2013 .
Bi 1.4 (D b). Cho dy s (an ) c xc nh bi
an+1 =

a2n
.
a2n an + 1

1. DY S

43

1. Chng minh rng an l dy gim v tm gii hn limn an .


P
2. Tm gii hn limn nk=1 ak .
Bi 1.5 (H Hng Vng, Ph Th). Cho dy s {xn } xc nh bi
(
x1 = 1;
xn+1 = 2013 ln(x2n + 20142 ) 20142 , n 1.
Chng minh rng dy {xn } c gii hn trong khong (20142 ; 0).
Bi 1.6 (C Ng Gia T). Cho dy s un xc nh bi:

u1 = 2014
u
+1
un = n1
2un1
Tm s hng tng qut v tnh lim un .
Bi 1.7 (H Qung Bnh). Tnh gii hn
lim

1
1
2

1
3

1 + + + ... +

1.
n

Bi 1.8 (H Qung Nam). Cho dy s {un } xc nh nh sau


(
u1 = u2 = 1

1
1
un n+1
un1 , n 2
un+1 = 2 n+1
Hy tnh un theo n.
Bi 1.9 (H Quy Nhn). Cho cc s thc a1 , a2 , ..., am tho mn

n
n
n
1 + 2 + ... + 2014 = n a1 + n a2 + ... + n am , n N.
Hy tnh
P = a1 a2 ...am .
Bi 1.10 (H Sao ). Tnh
lim

n 
X
k=1


k
1+ 2 1 .
n

Bi 1.11 (H S phm H Ni 2). Cho x1 , x2 > 0 v

xn+2 = xn+1 xn , n = 1, 2, . . .
Dy ny c hi t hay khng? Hy tm lim xn nu n hi t.
n

44
Bi 1.12 (C S phm Nam nh). Cho dy {un } tha mn:
u1 = u2 = 1, un+1 =

u2n (un1 + 1)
vi mi n 2.
un1

Tnh cc gii hn sau:


un
n+ un+1
(u1 + u2 + + un ) .un
lim
n+
2un+1 + 1
lim

Bi 1.13 (C S phm Qung Ninh). Cho dy s {xn }n xc nh bi


xn =
Tnh lim

n
X

n3 + 6n2 + 11n + 5
(n + 3)!

xk .

k=1

Bi 1.14 (H S phm Thi Nguyn). Tm s hng tng qut ca dy {an }


tha mn
n(an 2an1 + an2 ) = 2(an1 an2 ),
vi mi n 3, trong a1 , a2 l hai s thc cho trc.
Bi 1.15 (C S phm Vnh Phc). Cho dy s {u}n xc nh bi

u1 R
1

un+1 =
arctan un .
2014 2
Chng minh rng dy s cho hi t.
Bi 1.16 (H Tn Tro). Cho dy s {an } vi
an+1 = 1

3
2

< an <

n1
n2
n
1

+ (1 + n )an a2n
n+1
2n+1
2
2
2
2

5
2

v
n N.

Tnh
lim an .

Bi 1.17 (H Tn Tro). Cho dy s (un ) vi mi n = 1, 2, ...xc nh nh


sau
(
u1 = 1
un+1 = 1 + u1 u2 . . . un , vi mi n = 1, 2, . . . .
n
P
1
t Sn =
. Tm
uk
k=1

lim Sn .

n+

2. HM S

45

HM S

Bi 2.1 (H Bch khoa H Ni). Chng minh rng hm s f (x) = sin (x2014 )
khng tun hon trn R.
Bi 2.2 (H Bch khoa H Ni). a) Gi s f (x) l hm s xc nh trn
[0; +), lim f (x) = 0, g(x) l hm s tun hon trn R, f (x) + g(x) tng
x+

trn [0; +). Chng minh rng g(x) l hng s.


b) p dng: Tm tt c cc hm s (x) xc nh, tng trn [0; +), tho
mn
(x + 1) (x) = ex , x [0; +).
Bi 2.3 (H Cng ngh thc phm Tp. HCM). Cho f l mt hm s lin tc
Z1
trn [0, 1]. Tnh gii hn lim
xn f (x)dx.
n

Bi 2.4 (H Hng Vng, Ph Th). Cho f (x) l mt hm s dng lin tc


trn R. t
Rx
tf (t)dt
0
(x > 0).
(x) = Rx
f (t)dt
0

1) Chng minh rng (x) l hm tng trn khong (0; +). Tnh lim+ (x).
x0
2015
P  2014n + i 
2014n + i 1 
2) Tnh lim

.
n+ i=1
n
n
Bi 2.5 (H Qung Nam). Cho cc hm s f, g : R R tha
(f (x) f (y)) (g(x) g(y)) = 0,

x, y R.

Chng minh rng mt trong hai hm s l hm hng.


Bi 2.6 (C S phm Nam nh). Cho f (x) = x ln(1 + x)
a) Chng minh rng: vi mi x > 0 tn ti duy nht c(x) (0, x) sao cho
f 0 (c(x)) = ln(1 + x).
b) Tnh gii hn
lim+

x0

c(x)
.
x

46
R x2012
Bi 2.7 (C S phm Qung Ninh). Tnh gii hn lim+

x0

tan 1006 tdt


.
x2014

Bi 2.8 (C S phm Qung Ninh). Tm a, b hm s f (x) lin tc ti x = 1


vi

3x 1
nu < x 1
f (x) = ax2 x

b nu 1 < x < +
x1
Bi 2.9 (H S phm Thi Nguyn). Cho f : [0, 1] [0, 1] l hm lin tc
khc hng. Chng minh rng tn ti x1 , x2 [0, 1], x1 6= x2 sao cho
|f (x1 ) f (x2 )| = |x1 x2 |2014 .
Bi 2.10 (H Tn Tro). Cho f (x) l hm s lin tc v xc nh trn khong
[0, 1] tha mn:
1
x [0; 1] v f (0) = f (1) = .
2

0 f (x) 1,
Cho
Z
c=

Z
f (x)dx, F (x) =

Chng minh rng:


a.

f (t)dt.
0

(
F (x) x
F (x) c

b.

khi 0 x c
khi c x 1.

F (x)dx < c
0

c2
.
2

PHP TNH VI PHN

Bi 3.1 (HV An ninh nhn dn). Cho f (x) l hm lin tc trn [0, 1], kh vi
trn khong m (0, 1) tha mn f (0) = 0, f (1) = 1 v tn ti s k (0, 1) sao
cho

 
1
1
1k
|f
|
.
2
2
2
Chng minh rng tn ti cc s x1 , x2 phn bit thuc khong (0, 1)sao cho
f (x1 ).f (x2 ) = k.

3. PHP TNH VI PHN

47

Bi 3.2 (H Bch khoa H Ni). Cho hm s f (x) lin tc trn [0; +), c
o hm trn (0; +), lim f (x) = f (0). Chng minh rng tn ti c > 0 sao
x+

cho f 0 (c) = 0.
Bi 3.3 (HV Bu chnh Vin thng). Cho hm f (x) kh vi trn [0, +] v
0

lim f (x) = 0.

x+

Chng minh rng:


f (x)
= 0.
x+ x
lim

Bi 3.4 (H Cng ngh thc phm Tp. HCM). Chng minh rng phng
trnh
a1 cos x + a2 cos 2x + + an cos nx = 0
c t nht mt nghim thuc khong (0, ) vi mi s thc ai , i = 1, 2, ..., n
cho trc.
Bi 3.5 (H Cng ngh thc phm Tp. HCM). Gi s hm s f kh vi n
cp 2 trn on [1, 1], |f (x)| 1, x [1, 1] v f 2 (0) + f 02 (0) = 3. Chng
minh rng tn ti c [1, 1] sao cho f (c) + f 00 (c) = 0.
Bi 3.6 (H Hng Vng, Ph Th). Cho hm f lin tc trn [0; 1], kh vi
trn (0; 1) tha mn f (1) = 0. Chng minh rng tn ti x0 (0; 1)
x0 .f 0 (x0 ) + 2014 =

1
x2013
.ef (x0 )
0

Bi 3.7 (H Hng Vng, Ph Th). Chng minh rng nu hm s f kh


vi lin tc trn [0; 2] v tha mn cc iu kin: f (0) = 1, f (2) = 3; |f 0 (x)|
2, x [0; 2] th
Z2
f (x)dx > 2.
0

Bi 3.8 (H Ngoi thng H Ni). Gi s hm s f kh vi hai ln trn R v


3
tha mn iu kin f (0) = f (1) = 251, min f (x) = . Chng minh rng
x[0;1]
4
max f 00 (x) 2014.
x[0;1]

Bi 3.9 (C Ng Gia T). Cho f (x) l hm s lin tc trn on [a; b] v


ab
ba
a+b
kh vi trn khong (a; b) tha mn f (a) =
, f (b) =
; f(
) 6= 0.
2
2
2
Chng minh rng tn ti cc s c1 , c2 , c3 (a; b) sao cho
f 0 (c1 ).f 0 (c2 ).f 0 (c3 ) = 1.

48
Bi 3.10 (H Nng nghip). Cho f : R 7 R kh vi lin tc n cp 3 trn
R. Chng minh rng tn ti s thc a sao cho f (a)f 0 (a)f 00 (a)f 000 (a) 0.
Bi 3.11 (HV Phng khng Khng qun). Cho hm s f (x) c o hm ti
cp 2 trn khong [0; +). Bit rng
00

f (x)f (x)
2
f (x) > 0, f (x) > 0,
[f 0 (x)]2
0

vi mi x > 0. Hy tnh gii hn


0

f (x)
lim
.
x+ [f (x)]2
Bi 3.12 (H Qung Bnh). Cho P (x) l mt a thc bc n 1 tha mn
P (x) 0 vi mi x R. Chng minh
P (x) + P 0 (x) + + P (n) (x) 0, x R.
Bi 3.13 (H Qung Nam). Chng minh rng hm s
a0
+ cos x + a2 cos 2x + + a2014 cos 2014x
f (x) =
2
vi a0 , a2 , , a2014 R, |a0 | < 1 c th nhn nhng gi tr dng v cng c
th nhn nhng gi tr m trong khong [0, 2).
Bi 3.14 (H Qung Nam). Cho hm s f (x) lin tc trn R. Chng minh
rng nu hm s
Z x
g(x) = f (x)
f (t)dt
0

l hm gim th f (x) = 0 vi mi x R.
Bi 3.15 (H Quy Nhn). Cho hm lin tc trn f : [0; 2 ] R tho mn
Z
2
f (x)dx = 1. Chng minh rng tn ti x0 (0, 2 ) sao cho
0

x30
f (x0 ) x0 .
6

x0

Bi 3.16 (H Sao ). Cho f (x) l hm kh vi trn R v tha mn


|f (x)|

ex2

1
1

x R.

Chng minh rng tn ti c R sao cho


0

f (c) =

2c
ec2 1

3. PHP TNH VI PHN

49

Bi 3.17 (H Sao ). Cho f l mt hm s kh vi lin tc n cp 2 trn


[0, ] v tha mn: f (0) = f () = 0. Chng minh rng tn ti c (0, ) sao
cho
00
f (c) = f (c).
Bi 3.18 (H Sao ). Cho hm s f C 2 ([0, 1]), f (0) = f (1) = 0 v
0
f (0) = 2014. Tm gi tr nh nht ca
1

00

(f (x))2 dx.

Bi 3.19 (H S phm H Ni 2). Cho hm f xc nh, lin tc trn [0, 1]


kh vi trn (0, 1) v f (0) = 0. Chng minh vi mi  > 0 tn ti s t nhin
n v s thc dng x <  sao cho:
n2 x f (x )
.
2014(2014 n x )

f (x ) =

Bi 3.20 (C S phm Nam nh). f (x) l hm kh vi lin tc trn on


[0, 1] sao cho
Z1
(f 0 (x) + (1 2x)f (x)) dx = 5
0

Chng minh rng tn ti c (0, 1) sao cho f 0 (c) = 6.


Bi 3.21 (C S phm Nam nh). Cho f : [0, 1] [1, 1] kh vi lin tc
trn on [0, 1] tha mn f (0) = f (1) = 0.
Chng minh rng tn ti c (0, 1) sao cho f 0 (c) = 2c tan f (c)
BiZ3.22 (C S phm Qung Ninh). Cho hm s f lin tc trn [2013; 2015]
2015
v
f (x)dx = 0. Chng minh rng tn ti s c (2013; 2015) sao cho
2013

2014

f (x)dx = cf (c).
2013

Bi
R 1 3.23 (H
R 1S phm Thi Nguyn). Cho f : [0, 1] R lin tc sao cho
f (x)dx = 0 xf (x)dx. Chng minh rng tn ti c (0, 1) sao cho
0
Z
f (c) = 2014

f (x)dx.
0

50
Bi 3.24 (C S phm Vnh Phc). Cho hm s f lin tc trn on [a; b] v
Z b
kh vi trn (a; b) vi a > 0 v
f (x)dx = 0. Chng minh rng tn ti s
a

c (a; b) sao cho


Z

Z
f (x)dx 2013cf (c) + 2012

2014c
a

f (x)dx = 0.
a

Bi 3.25 (C S phm Vnh Phc). Cho hm s f : [1; 1] R lin tc .


Chng minh rng phng trnh
xf 2014 (x) 2014f (x) = 2013x
c nghim trn on [1; 1].
Bi 3.26 (H Tn Tro). Cho hm s (x) c o hm cp mt trn on
[1, 2], c o hm cp hai trn khong (1, 2) v
Z 2
0
(x)dx = 2(2) (1) 1, (1) = 1.
1

Chng minh rng phng trnh


00

x (x) + (x) = 0
lun c nghim trong khong (1, 2) .

PHP TNH TCH PHN

Bi 4.1 (HV An ninh nhn dn). Cho hm f : [0, 1] R lin tc tha mn


Z 1
xf (x)dx = 0.
0

Chng minh rng


2 2

max |f (x)|.
x f (x)dx
x[0,1]
6
0
Z 3
2
Bi 4.2 (H Bch khoa H Ni). Tnh tch phn I =
Z

dx
.
2 + cos x

Bi 4.3 (HV Bu chnh Vin thng). Cho hm f (x) lin tc trn [0, 1] c gi
tr nh nht m > 0 v gi tr ln nht M . Chng minh rng:
Z 1
Z 1
1
(m + M )2
1
f (x)dx.
dx
.
4mM
0
0 f (x)

4. PHP TNH TCH PHN

51

Bi 4.4 (HV Bu chnh Vin thng). Cho hai hm s f v g lin tc trn on


[0;1], g(x) > 0 x [0; 1] v
1

g(x)dx = m.
0

Chng minh rng tn ti c [0; 1]sao cho


1

f (x)g(x)dx =
0

1
f (c).
m

Bi 4.5 (HV Bu chnh Vin thng). Cho 0 < < . Tnh tch phn suy rng

Z
I=
0

ex ex
dx.
x

Bi 4.6 (H Cng ngh thc phm Tp. HCM). Cho hm s f dng, kh vi


lin tc n cp 2 trn R sao cho f (0) = 1, f 0 (0) = 0 v f 00 (0) = 1. Tm
x
 
2
.
lim f
x+
x
Bi 4.7 (D b). Tnh
2013x
Z

lim+

x0

(sin t)2012
dt
t2013

Bi 4.8 (D b). Gi s f l hm s lin tc v dng, xc nh trn on


[0, 1]. Chng minh vi mi [0, 2] , phng trnh (n l )
Zx2

Z1
f (t)dt +

Z1
f (t)dt =

f (t)dt,
0

1x2

c nghim duy nht x() [0; 1]. Tm gii hn


lim+

x2 ()
.

+
R

Bi 4.9 (H Hng Vng, Ph Th). Chng minh rng nu tch phn

hi t v bng J th tch phn

+
R

f x

1
dx cng hi t v bng J.
x

f (x)dx

52
Bi 4.10 (H Ngoi thng H Ni). Cho hm lm, lin tc f : [0, 1] R,
c f (0) = 1. Chng minh rng
Z1

1
2
Z
2
xf (x)dx f (x)dx
3
0

Bi 4.11 (H Ngoi thng HN). Cho a R+ , f : [0, a] R l nh x kh vi


lp C 1 sao cho
x [0, a], f 0 (x) > 0 v f (0) = 0.
a) Chng minh rng
Zx
x [0, a] ,

f (x)
Z
f (t)dt +
f 1 (t)dt = xf (x)

, k hiu f 1 : [0, f (a)] R l nh x ngc ca f .


b) T suy ra
Zy

Zx
(x, y) [0, a] [0, f (a)] ,

f (t)dt +

f 1 (t)dt xy.

Bi 4.12 (C Ng Gia T). Cho a > 0, b 0 v a > b t


4
}.
(a b)(b + 1)2

f (x) = min{a +
Z 2014
3 f (x)dx.
Tnh
0

Bi 4.13 (C Ng Gia T). Cho f : [0; 1] R sao cho f 00 (x) > 0 vi mi


x [0; 1]. Chng minh rng
Z
2

Z
f (t)dt > 3

f (t2 )dt f (0).

Bi 4.14 (H Nng nghip). Mt b nc c th tch 2400 lt ng y nc.


Ngi ta m mt vi nc y b tho nc ra ngoi. Bit lu lng
chy l r(t) = 100 2t (lt/pht). Hi sau bao lu b cn nc.

4. PHP TNH TCH PHN

53

Bi 4.15 (H Nng Nghip). Cho f (x) l hm kh vi lin tc trn on [0, 2]


v f (1) = 0. Chng minh rng
Z
Z 2
i2
3h 2
0
2
[f (x)] dx
f (x)dx .
2 0
0
Bi 4.16 (HV Phng khng Khng qun). Tnh tch phn
Z
2
cos2014 x

dx.
I=
x2 + 1
2 1 x +
Bi 4.17 (HV Phng khng Khng qun). Tnh tch phn
Z
2014
1
I=
dx.
cos
1 + e 2014x
0
Bi 4.18 (HV Phng khng Khng qun). Cho f(x) l hm nhn gi tr
dng, lin tc trn v l hm tun hon vi chu k T = 2014. Chng minh
rng:
Z 2014
f (x)
+
2014 dx 2014, n N .
f (x + n )
0
Bi 4.19 (H Qung Bnh). Tnh tch phn
Z
2
ex . sin xdx
.
I=
2
0 (sin x + cos x)
Bi 4.20 (H Qung Bnh). Gi s f l mt hm s lin tc trn [0; 1],
Z1
1 x2
f (t)dt
, x [0, 1] v F l mt nguyn hm ca f trn [0, 1].
2
x

a) Chng minh rng:


Z1
F (1) =

Z1
xf (x)dx +

Z1
b)
0

1
xf (x)dx v
3

Z1

F (x)dx.
0

1
f 2 (x)dx .
3

Bi 4.21 (H Qung Nam). Tnh tch phn


Z
4
ln(1 + tan x)dx
I=
0

54
Bi 4.22 (C S phm Qung Ninh). a) Cho hm f lin tc trn (0; +) v
lim f (x) = 2014. Tnh
x+
Z
1 x
f (t)dt.
lim
x+ x 0
Z b
Z b
b) Cho f lin tc trn [a; b] v |
f (x)dx| =
|f (x)|dx. Chng minh rng
a

f khng i du trn on [a; b].


Bi 4.23 (H Quy Nhn). Tnh tch phn
1

Z
I=
0

x2014
2
1 + x + x2! + ... +

x2014
2014!

dx

Bi 4.24 (H S phm H Ni 2). Tnh tch phn


p
ln(9 x)
p
p
dx.
22
ln(9 x) + 22 ln(x + 3)

22

Bi 4.25 (H S phm H Ni 2). Cho hm s f kh vi lin tc trn on


[a, b] v f (a) = 0. Chng minh rng
b

(b a)2
f (x)dx
2

[f (x)]2 dx.

Bi 4.26 (H S phm Thi Nguyn). Cho hm f lin tc, kh vi trn [0, 1]


R
2
R 2/3
R1
1
sao cho 1/3 f (x)dx = 0. Chng minh rng 0 (f 0 (x))2 dx 27 0 f (x)dx .
Bi 4.27 (C S phm Vnh Phc). Cho hm s f : [0; 1] [0; 1] l hm n
iu tng v lin tc. Chng minh rng
Z

f (x

2014

)dx

f (x)dx +
0

1
.
2015

Bi 4.28 (C S phm Vnh Phc). Cho f C ([0; 1]) v

f (x)dx = 0.
0

Chng minh rng


Z
(
0

1
f (x)dx)
12
2

Z
0

(f 0 (x))2 dx.

1/2

5. CHUI S

55

CHUI S

Bi 5.1 (H Bch khoa H Ni). Xt s hi t ca chui s


+
X



(1)n
ln 1 +
.
n
n=2

Bi 5.2 (HV Bu chnh Vin thng). Chng minh rng:


X

X
n=1 k=1

1
=
n(n + 1) (n + k)

ex 1
dx.
x

Bi 5.3 (D b). Cho xn l dy s n iu v limn xn = a > 0. Chng


minh rng chui s


X
xn
1
,
xn+1
n=1
hi t.
Bi 5.4 (H Hng Vng, Ph Th). Gi s {an } l dy cho bi a1 =
4; an+1 = a2n 3.an + 4 vi mi n N . Chng minh rng

X
n=1

1
1
= .
an 1
2

Bi 5.5 (HV Phng khng Khng qun). Cho hm s f (x) c khai trin
di dng chui lu tha:
f (x) = 1 + a1 x + a2 x2 + + an xn +
Gi s hm

f (x)
g(x) =
f (x)
cng c khai trin thnh chui lu tha v gi thit rng tt c cc h s
ca khai trin c gi tr tuyt i khng vt qu 2. Chng minh rng:
|an | n + 1

n N.

Bi 5.6 (H Quy Nhn). a) Cho dy s gim {an }n tho mn chui

X
n=1

an

56
hi t. Chng minh rng lim nan = 0.
n

b) Kim tra s hi t ca chui

3
sin n3 + n.

n=1

Bi 5.7 (H Sao ). Cho (an ) l mt dy s thc khng m tha mn cc


iu kin sau:
a2n a2n+1 a2n , a2n+1 a2n+2 an an+1
vi mi n 1 v
1
lim nan < .
n
4
Chng minh rng chui s

an hi t.

n=1

Bi 5.8 (H S phm H Ni 2). Chng minh rng nu chui s dng

P
an hi t th chui s
n=1

ann+1

n=1

cng hi t.
Bi 5.9 (C S phm Nam nh). Xt s hi t v tnh tng chui s
+
X
n=1

n2 + n + 1
.
n(n + 1)(n + 2)(n + 3)

Bi 5.10 (H S phm Thi Nguyn).


Cho 0 a 2 v {an } l dy
p
P s thc
2
xc nh bi a1 = a, an+1 = 2n 2n (2n an ) vi mi n 1. Tm
n=1 an .

6. PHNG TRNH HM

57

PHNG TRNH HM

Bi 6.1 (HV Bu chnh Vin thng). Cho hm f : R R kh vi lin tc trn


[0;1] tha mn
f (2015x + 2014) = 2015f (x)

x R.

Tm hm f (x).
Bi 6.2 (H Cng ngh thc phm Tp. HCM). .
a) Cho > 0, tm hm s f : [0; +) [0; +) tho mn iu kin
f (f (x)) + f (x) = 22 x, x [0, +).
b) Tm hm s f xc nh, lin tc trn R v tho mn iu kin


x+y
(x + y)f
= xf (x) + yf (y), x, y R.
2
Bi 6.3 (H Hng Vng, Ph Th). Tm tt c cc hm f lin tc trn [0; 1],
c o hm trn khong (0; 1) v tha mn 2 iu kin:
(
18.f 0 (x) 1.f (x) + 2014 0, x (0; 1)
f (0) = f (1) = 2014.
Bi 6.4 (C Ng Gia T). Tm tt c cc hm s f xc nh trn [0; 1] kh vi
trong khong (0; 1) tha mn iu kin
(
f (0) = f (1) = 1
2013f 0 (x) + 2014f (x) 2014
Bi 6.5 (H Nng nghip). Tm tt c cc hm f C 2 [0, 1], tc hm f (x) c
o hm ti cp 2 v f 00 (x) lin tc trn [0, 1] tha mn:
Z 1
3
0
00
f (0) = f (0) = 1, f (x) 0, x (0, 1) v
f (x)dx = .
2
0
Bi 6.6 (H Qung Bnh). Tm hm s f (x) xc nh v lin tc trn R bit
rng:
x
2f (x) = f ( ) + 2x, x R.
2
Bi 6.7 (H Sao ). Tm tt c cc hm s f tha mn
f (y)

f (x + y) = f (x)e 2014 1

x, y R.

58
Bi 6.8 (H S phm H Ni 2). Chng minh khng tn ti hm f : R R
vi f (0) > 0 v
f (x + y) f (x) + yf (f (x)),

x, y R.

Bi 6.9 (C S phm Nam nh). Tm hm f : R R tha mn:


!
2x + y
f (x) + f (2y)
f
=
, x, y R.
3
3
Bi 6.10 (C S phm Nam nh). Tm hm f : [0, 1] (0; +) kh vi lin
tc trn on [0, 1] tha mn f (0) = 1, f (1) = 4 v
Z1

(f 0 (x))2 dx
4.
f (x)

Bi 6.11 (C S phm Qung Ninh). Tm tt c cc hm s thc f (x) lin


tc tha mn
(
f (0) = 2014
f (2014x) f (x) = x,
x R.
Bi 6.12 (H S phm Thi Nguyn). Tm tt c cc hm f : R R+ tha
mn
x+1
;
x2 + 2
[(xy 2)2 + 2(x + y)2 ]f (x)f (y)
f (x + y)
(x + y)2 + 2
f (x)

vi mi x, y R.
Bi 6.13 (C S phm Vnh Phc). Tm tt c cc hm f : R R tha mn
f (x + y) f (x).f (y) 2014x+y , x, y R.
Bi 6.14 (H Tn Tro). Tm hm s f : R R tha mn:
(x y)f (x + y) (x + y)f (x y) = 4xy(x2 + y 2 )

x, y R.

Phn III
HNG DN GII

59

THI CHNH THC


1

I S

Bi 1.

a) Gi C1 , . . . , C4 l cc ct ca ma trn cho. t
i1
Ci0 = (ai1
1 , . . . , a4 ).

0
.
D thy rng Ci = Ci0 + t hp tuyn tnh ca cc C10 , . . . , Ci1
T suy ra nh thc cn tnh bng nh thc ca ma trn c to
thnh t cc ct C10 , . . . , C40 .
y l nh thc Vandermonde quen thuc v do gi tr cn tm
bng
Y
det(C10 , . . . , C40 ) =
(aj ai ).
1i<j4

b) Trong ma trn ban u, mi h s trn ct th 3, 4 tng ng l tch


ca 2, 3 s nguyn lin tip, do chia ht cho 2!, 3!. T suy ra iu
phi chng minh.
Bi 2. Gi thit P (x) =

5
P

ci xi . T cc iu kin ca bi ton ta suy ra mt

i=0

h 6 phng trnh tuyn tnh vi 6 n l c0 , . . . , c5 :


5
X

aik ci

= bk ,

i=0

5
X

iai1
k c i = bk ,

k = 1, 2, 3

i=1

Nu b1 = b2 = b3 = 0 th a
Q thc 0 l a thc duy nht tha mn. Tht vy,
t gi thit suy ra P (x) = (x ai )Q(x) vi Q(x) l a thc bc khng qu
i

2. T h thc P 0 (ai ) = 0 ta suy ra Q(ai ) = 0. Do Q 0. Theo trn, khi cc


h s bk u bng 0 th h c nghim duy nht. Do ta suy h c nghim
duy nht vi mi b bk .
Cch khc:
- Xt nh x t khng gian cc a thc bc 5 vi h s thc vo R6 gi
mi a thc P ln (P (a1 ), P 0 (a1 ), . . . , P (a3 ), P 0 (a3 )). Bi ton yu cu chng
minh l mt song nh. Hin nhin l nh x tuyn tnh gia cc khng
gian c cng s chiu bng 6. D dng kim tra c rng ker = 0 v bi
ton c chng minh.
- Cng c th xy dng trc tip a thc P (x) bng phng php ni suy.

62
Thit lp cng thc ni suy Lagrange
Xc nh c a thc bc 2 nhn gi tr ti ai
Kt thc bi ton
x2 x3 x4
, , .
2! 3! 4!
Do ma trn ca e theo c s trn l ma trn ng cho vi cc gi tr ring
khc 0 nn nh x l kh nghch.

Bi 3. a) Trc ht thit lp ma trn nh x o hm trong h c s 1, x,

b) Theo cng thc Taylor, ta c, vi mi f R[x] th


f (x + 1) =

X
f (i) (x)
i=0

i!

Ni cch khc, e(D) gi a thc f (x) ln f (x + 1). Hin nhin y l mt nh


x tuyn tnh kh nghch.
Ghi ch:
Th sinh c th dng phng php ca cu a) gii cu b).
Th sinh c th chng minh cu b) trc, t suy ra cu a).
Bi 4. S dng bin i s cp theo hng ta c


Em
B
det(X) = det
0
En CB
T s dng khai trin Laplace ta c iu phi chng minh.
b) Vi A kh nghch, ta c khai trin



A 0
Em A1 B
X=
.
0 En
C
D


Em A1 B
S dng cc bin i s cp i vi ma trn
nh trong cu a)
C
D
ta c iu phi chng minh.
Bi 5. a. S dng phn chng. Gi s a l s v t. Gi s P = P1 Pk
vi P1 , . . . , Pk l cc a thc h s hu t v bt kh qui trn Q. Bi v a l
nghim ca P , d nhin a l nghim ca mt s a thc Pi . Khng mt tng
qut, gi s P1 , . . . , Pm nhn a lm nghim. Do P1 , . . . , Pm c h s hu t v
nhn s v t a lm nghim ta suy ra chng c bc 2. Ta nhc li kt qu
quen bit sau y: mi a thc bt kh qui trn Q ch c nghim n trong R
(trong bt k trng cha Q). T suy ra bi ca a trong P bng m. Suy ra
deg P deg P1 P2 . . . Pm 2m > 2

n
= n.
2

1. I S

63

y l iu mu thun cn tm v bi ton c gii quyt.


Nhn xt: Bi ton cn c nhiu tip cn khc: qui np theo bc ca P , xt
ian ca Q[x] gm cc a thc nhn a lm nghim, v.v...
b. 1) Theo nh ngha xn l s ng i di n gia A v A. Mt ng i
bt u t A v kt thc ti A, ngay trc khi kt thc phi dng li ti B
hoc D. iu ny cho thy mt ng i di n gia A v chnh n c
to thnh t mt ng i di n 1 t A ti B v cnh BA hoc mt
ng i di n 1 t A ti D v cnh DA. T suy ra
xn = 2yn1 .
Tng t, mt ng i di n t A ti B c to thnh t mt ng i
di n 1 t A ti A v cnh AB hoc mt ng i di n 1 t A ti
C v cnh CB. Do
yn = xn1 + zn1 .
Tng t ta c
zn = 2yn1 .
Mt cch tng ng, ta c

xn
0 2 0
xn1
1 0 1 yn1 = yn .
0 2 0
zn1
zn
2) Ta c xn = zn = 2yn1 vi mi n. T yn = 2xn1 = 4yn2 . Quan h yn =
4yn2 cng vi gi tr ban u y0 = 0, y1 = 1 chng t y2k = 0, y2k+1 = 22k . T
y, ta suy ra x2k = z2k1 = 2y2k1 = 22k1 , z2k+1 = x2k+1 = 0.

64

GII TCH

Bi 1. Ta c u2n+1 = u2n + an v vy
u2n = 1 + a + + an1 .
q

n 1
Tnh c cng thc un = aa1
khi a 6= 1 v un = n khi a = 1.
q
1
Ch ra khi a < 1 dy c gii hn v gii hn l 1a
.
Bi 2. Gi s f khng phi l hm hng. Khi tn ti cc s a, b sao cho
f (b) f (a) 6= 0. iu ny ko theo g(a) = g(b) = m.
Ln lt ly y = a v y = b ta c h






f (x) f (a) g(x) m = 0;
f (x) f (b) g(x) m = 0.
Tr cc phng trnh ca h cho nhau ta thu c (f (a)f (b))(g(x)m) = 0
vi mi x. Do f (a) f (b) 6= 0 nn g(x) = m x R.
Bi 3. Nu f n iu gim th f (x) f (0) x [0, +). Do vy,

1
x

Rx

f (t)dt

f (0). iu ny tri vi gi thit.


Vy f (x) l hm khng gim trn [0, ). Khi , f (t) f (x) t x. Do
Rx
Rx
1
1
f
(t)dt

f (x)dt = f (x) vi mi x > 0. iu ny ko theo


x
x
0

1
lim
x x

Zx
f (t)dt lim f (x).
x

2) Kt lun khng cn ng. Ta xt th d sau: xt hm s f (x) = |sinx|x2 .


Ta c
Z 2n
n Z (2k+1)
n
X
X
f (x) dx
x sin x dx =
(4k + 1) = n(2n + 3)
0

k=1

2k

k=1

Do ,
1
lim
x x

Z
0

1
f (t) dt lim
x [x/2]2 + 2

D thy khng tn ti limx f (x).

[x/2]2

f (t) dt = .
0

2. GII TCH

65

Bi 4. Chia hai v cho 2013 ta nhn c


f 0 (x) + kf (x)
t g(x) = ekx (f (x) m) vi m =

2014
2015
; vi k =
.
2013
2013

2015
2014

ta c

g 0 (x) = ekx [k(f (x) m) + f 0 (x)] 0.


T suy ra g l hm khng gim.
Mt khc, g(0) = g(1) = 0 nn ta suy ra g(x) 0. T f (x) m.
Bi 5. Cch 1:
Bng quy np chng minh c xn 4.

Xt dy bn+1 = 2 bn vi b0 = max{4, x0 , x1 }. Do

bn+1
bn

bn
bn1

v b1 b0

nn ta suy ra l dy khng tng v b chn di, do limn bn = 4. Ta chng


minh max{x2n , x2n+1 } bn vi mi n.
p

Vi n = 1 ta c x2 = px1 + x0 2 max{x
,
x
}

2
b0 = b1 .
1
0
p

x3 = x2 + x1 2 max{x2 , x1 } 2 max{b1 , b0 } b1
Gi thit max{x2k , x2k+1 } p
bk vi mi k = 0, 1, ..., n p
1. Khi ,

x2n = x2n1 + x2n2 2 max{x2n1 , x2n2 } 2 bn1 = bn .


p
p

x2n+1 = x2n + x2n1 2 max{x2n , x2n1 } 2 max{bn , bn1 } bn .


Tng kt li, ta c 4 max{x2n , x2n+1 } bn vi mi n v limn bn = 4. Vy
limn xn = 4.
Cch 2:
Gi thit x1 x0 . Khi
x2 =
do x0 4.
x3 =

x1 + x 0 2 x0 x0 ,

x2 + x1 x1 + x0 = x2 .

Nh vy bng quy np ta chng minh c

x2n+2 2 x2n x2n ;


x2n+1 x2n .
Nh th dy (x2n ) l dykhng tng b chn di bi 4. T suy ra tn ti
gii hn lim x2n = a 2 a. Hay l a 4. Kt hp vi iu kin x2n 4 ta c
a = 4.
Kt hp iu ny vi bt ng thc th 2 trn ta c lim x2n+1 = 4.
Vy lim xn = 4.

Nu x1 x0 ta thy x2 = x1 + x0 2 x1 x1 v l lun nh trn khi


thay x0 bi x1 cn x1 bi x2 .

66
Bi 6. 6a.r

t bn = 6 + 6 + ... + 6 . Do bn+1 = 6 + bn . Bng quy np ta chng


|
{z
}
n ln

minh c bn l dy tng b chn trn bi 3 v bn 3.


Mt khc,

an+1
3 bn+1
3 6 + bn
1
1

=
=
=
.
an
3 bn
3 bn
6
3 + 6 + bn
Theo tiu chun DAlambert chui s hi t.
6b.
T gi thit
Z

t2 dx.

f (t) dt
0

Theo bt ng thc Cauchy-Schwartz ta c


Z

2 Z
tf (t) dt

Z

f (t) dt

t dt

2
t dt .
2

V vy
Z

Z
tf (t) dt

t(t f (t)) dt 0.

t dt hay l F (x) =
0

Mt khc,
Z

1
(t f (t)) dt =
t(t f (t)) dt
0
0 t
Z x
Z x
1
F (t)
F (x)
=
dF (t) =
+
dt 0.
x
t2
0 t
0
Rx
Rx
Rx
2
T , 0 (t f (t)) dt 0 hay l 0 f (t) dt 0 t dt = x2 .

CC BI XUT: I S

MA TRN

a b c
Bi 1.1 (H An Giang). Xt X = d e f . Khi
g h i

d
ea
2c + f b
h d 2f + i e ,
AX XA = g
2g 2h g
h
nn X giao hon vi A khi v ch khi d = g = h = 0, e = a, f = 2c b, i =
4c 2b + a, ngha l X c dng

a b
c
2c b .
X = 0 a
0 0 4c 2b + a
Bi 1.2 (H An Giang). a thc c trng ca A l
p(t) = t4 3t2 + 1 = (t2 t 1)(t2 + t 1).

1 5
1+ 5
, 2 =
,
Do A c 4 gi tr ring n phn bit l 1 =
2
2

1 + 5
1 5
3 =
, 4 =
.
2
2
C s ca cc khng gian ring tng ng vi tng tr ring l:
E(1 ) c c s {u1 = (1 , 1, 1, 1 )};
E(2 ) c c s {u2 = (2 , 1, 1, 2 )};
E(3 ) c c s {u3 = (3 , 1, 1, 3 )};
E(4 ) c c s {u4 = (4 , 1, 1, 4 )}.
Do A cho ha c, vi dng cho l B = P 1 AP , trong
> > >
B = diag(1 , 2 , 3 , 4 ), P = (u>
1 u2 u3 u4 ).

Suy ra A2014 = P.B 2014 .P 1 . T y ta thu c kt qu.


Ch . y l bi ton n thun tnh ton, tuy nhin vic tnh ton rt phc
tp.

68
Bi 1.3 (H An Giang). Ta c (A B 2I)(A + I) = A2 A B BA 2I =
2I Suy ra (A + I)(A B 2I) = 2I, ngha l A2 = A + B + AB. Nh vy
AB = BA.
Ch . Trong li gii c s dng tnh cht Nu A, B Mn (K) sao cho AB = I
th BA = I.
Bi 1.4 (H Bc Liu). a thc c trng ca A l p(x) = (x )(x ).
Chia xn cho p(x) ta c dng biu din
xn = p(x).q(x) + ax + b.
Ln lt thay x bi v trong ng thc trn, ta c n = a + b v
n n
n n
v a =
. Do p(A) = 0 nn bng
n = a + b. Suy ra a =

cch thay x bi A vo ng thc trn, ta c


 1

 1

n
n
n
A = a.A + b.I =
(A I) +
(I A) .

Ngha l tn ti
X=

1
1
(A I); Y =
(I A)

tha mn An = n X + n Y .
Bi 1.5 (H Bc Liu). Ta c det(N M ) = det(M N ) = 1. Do N M kh
nghch v M, N kh nghch. Hn na (M N )2 = I, nn M N = (M N )1 =
N 1 M 1 . Do (N M )2 = N (M N )M = N (N 1 M 1 )M = I, ngha l
(N M )1 = N M .
Bi 1.6 (H Bch Khoa H Ni). Gi s hng th i ca A gm ton s 1. Khi
hng i ca AA c h s mi ct l tng tt c cc h s ct ca
A . Do tng tt c cc phn t ca A bng tng tt c cc phn t hng
th i ca AA . M A.A = det(A).In , nn tng ny chnh bng det(A). Do
ta c kt qu cn chng minh.
Bi 1.7 (H Bch Khoa Tp. H Ch Minh). a thc c trng ca A l P () =
( 1)3 do (A I)3 = 0. Phn tch
x14 = (x 1)3 q(x) + ax2 + bx + c.
Ta c

14

1 = a + b + c
14.113 = 2a + b

14.13.12012 = 2a

Thay A vo () ta c: A14

a = 91
b = 168

c = 78.

1
14 14
= aA2 + bA + cI = 14 90 91 .
14 91 92

()

1. MA TRN

69

Bi 1.8 (H Bch Khoa Tp. H Ch Minh). Nu A kh nghch th B =


A1 .AB = 0 v l. Do det(A) = 0. iu ny chng t tn ti x, y Mn,1
khc 0 tha Ax = A> y = 0.
Xt C = xy > l ma trn cp n khc 0 tha
AC = Axy > = 0.y > = 0; CA = xy > A = x.(A> y)> = 0.


a b
>
Bi 1.9 (H Bch Khoa Tp. H Ch Minh). V A = A nn A =
. Dn
b c
 2

a + b2 b(a + c)
n A2 =
= I. T suy ra
b(a + c) b2 + c2

b(a + c) = 0 (1)
a2 + b2 = 1 (2)

2
b + c2 = 1 (3)
T (1) suy ra b = 0 c = a
1. b = 0 = a = 1 = c = 1. Ta c 4 ma trn

 
 
 

1 0
1 0
1 0
1 0
;
;
;
.
0 1
0 1
0 1
0 1
Th li tha.

2. b 6= 0 = c = a (0, 1) = b = 1 a2 . Ta c 4 ma trn dng ma


trn

 


2
2
a

a
1

a
1

;
;
1 a2
a
1 a2
a


 

2
2
a
1

a
1

a
a

;
1 a2
a
1 a2
a
Bi 1.10 (H Bch Khoa Tp. H Ch Minh). Ta c
A101 I = (A I)(I + A + + A100 ) = 0
v A khng c tr ring bng 1. t
f (A) = I + A + + Ak1 = (Ak + Ak+1 + + A100 ).
Suy ra f (A) kh nghch. V 101 nguyn t nn lun tn ti s, t Z+ tha mn
sk t101 = 1.
t p(x) = 1 + xk + + x(s1)k . Ta c
f (A).p(A) =

sk1
X
i=0

Ai =

t101
X

Ai = I.

i=0

Ch A c phn t nguyn nn c f (A) v p(A) u nguyn nn c nh


thc nguyn. iu phi chng minh r.

70
Bi 1.11 (H Bch Khoa Tp. H Ch Minh). Phn chng, ta gi s |A2 +
B 2 | = 0 = 0 6= x Rn : (A2 + B 2 )x = 0 = x> A2 x + x> B 2 x = 0
(Ax)> Ax + (Bx)> Bx = 0 ||Ax||2 + ||Bx||2 = 0 Ax = Bx = 0.
Lc ny mi ma trn X, Y , ta c (AX + BY )> x = X > Ax + Y > Bx = 0 =
det(AX + BY ) = 0. iu ny l khng ng vi gi thit.
Bi 1.12 (HV Bu chnh Vin thng). a) Ta c ABAB = 9AB
b) 2 = r(AB) = r(9AB) = r(ABAB) r(BA) 2. Ma trn BA vung cp
2 c r(BA) = 2, do kh nghch
c) ABAB = 9AB B(ABAB)A = B(9AB)A (BA)(BA)(BA) = 9(BA)(BA).
S dng tnh kh nghch ca ma trn BA v ng thc trn suy ra BA =
9I2 .
Bi 1.13 (HV Bu chnh Vin thng). D dng thy rng nu AB = BA th
bt ng thc trn c du =.
=
Gi T l ma trn rc giao sao cho T 1 AT = A l ma trn cho. t B
1
T BT (khng chc la ma trn cho). Ta c
2 ) = Tr(T 1 A2 B 2 T ) = Tr(A2 B 2 ); Tr(AB
AB)
= Tr(T 1 ABABT ) = Tr(ABAB).
Tr(A2 B
AB)
=
Tr(AB

n X
n
X

aii bij ajj bji =

i=1 j=1

2aii ajj b2ij

n
X

a2ii b2ii

i=1

1i<jn

i xng, A l ma trn cho).


( V B
2) =
Tr(A2 B

n X
n
X

a2ii bij bji =

i=1 j=1

n X
n
X

a2ii b2ij =

i=1 j=1

(a2ii + a2jj )b2ij +

n
X

a2ii b2ii

i=1

1i<jn

Vy
2 )Tr(AB
AB)
=
Tr(A2 B

(a2ii +a2jj 2aii ajj )b2ij =

1i<jn

(aii ajj )2 b2ij 0.

1i<jn

Du "=" (aii aij )2 b2ij = 0; i < j aii bij = bij ajj ; i < j AB = BA.
Bi 1.14 (H Cng ngh thc phm Tp. HCM). Ta c AB = A + B nn
(A I)(B I) = I. Do A I v B I kh nghch v l nghch o
ca nhau. Vi mi vect u ta c Au = 0 (A I)u = u u =
(B I)u Bu = 0. Suy ra hai h phng trnh AX = 0 v BX = 0 c
cng tp hp nghim. Do rank(A) = rank(B).
Bi 1.15 (H Cng ngh thc phm Tp. HCM). t A = (aij ) v A1 = (bij ).
n
P
Do AA1 = I nn vi mi i 6= j, ta c
aik bkj = 0. Do aik > 0, k = 1, 2, ..., n
k=1

nn tn ti t nht 2 trong cc s cc bkj tri du nhau, ngha l c t nht 2


s khc 0 trong mt ct ca A1 . Suy ra s h s 0 ca A1 l zn n2 2n.

1. MA TRN

71

Bi 1.16 (D b).
1. T nh ngha, ta d dng suy ra rng mt ma trn l
thc s ngu nhin nu cc h s l > 0 v nhn (1, 1, . . . , 1) lm vector
ring vi gi tr ring 1. T y, ta d dng suy ra rng Ak l thc s
ngu nhin vi mi k 1.
(k)

2. t Ak = (aij ). Th th t ng thc Ak+1 = A Ak ta c


(k+1)
aij

n
X

(k)

ais asj .

s=1
(k)

Theo gi thit, j

(k)

= max1sn asj . Suy ra


(k+1)
aij

(k)
j

n
X

(k)

ais = j .

s=1

T ,
n

(k+1)

jk+1 = max aij


i=1

(k)

j .
(k)

(k+1)

Lp lun mt cch tng t, ta cng thu c j j


C nh j. Gi i l ch s sao cho

(k)
j

(k+1)
j

(k)
j

(k+1)
j

(k+1)
aij

(k+1)
aij .

(k)
j

Th th,
n
X

(k)

ais asj

s=1

n
X

(k)

(k)

(k)

(k)

ais (j asj ) (j j )

s=1
(k)

(Bi v cc h s l dng v tn ti t nht mt ch s s asj = j k)


(k+1)
(k+1)
v asj ). Tng t, gi i0 l ch s tho mn j
= ai0 j . Th th,
(k+1)
j

(k)
j

(k+1)
ai 0 j

(k)
j

n
X

(k)

ai0 s asj j k)

s=1

n
X

(k)

(k)

(k)

ai0 s (asj j k)) (j j )

s=1

Bng cch ly tng ca cc bt ng thc thu c trn ta suy ra


(k)

(k)

(k+1)

j j (j

(k)

jk+1 ) 2(j k) j ).

Bt ng thc ny tng ng vi bt ng thc cn chng minh.

72
3. Trng hp n = 1 l tm thng. Ta gi s n 2. Khi ,  1/2 v
(k)
(k)
nh vy 0 1 2 < 1. Ta suy ra dy j j tin ti 0 khi k tin ti
(k)
(k)
v cng. T y, ta d dng suy ra rng cc dy j , j cng tin ti
mt gii hn hu hn xj . Theo nh l kp, ta suy ra
(k)

lim aij = xj .

Nh vy,

x1 x2 . . . xn
x1 x2 . . . xn

k
lim A = M = ..
.
.
.
.
.
.
.
k
.
.
.
.
x1 x2 . . . xn
P
(k)
Mt khc, vi mi i, k ta c nj=1 aij = 1 nn x1 + x2 + . . . + xn = 1. Ta
suy ra M cng l mt ma trn ngu nhin.
Bi 1.17 (H ng Thp). Gi s I + BA khng kh nghch, tc l det(I +
BA) = 0. Suy ra tn ti X 6= 0 sao cho (I + BA)X = 0, suy ra X + BAX =
0().
T () suy ra AX + ABAX = 0, hay (I + AB)AX = 0. Nu AX = 0 th t
() ta c X = 0, v l. Nu AX 6= 0 th I + AB khng kh nghch, v l. Vy,
I + BA kh nghch.


a b
Bi 1.18 (H Hng Vng, Ph Th). Gi s A =
thch hp. T
c d
 2 2 
a b
A2 =
, suy ra
c2 d 2

bc = 0
(a + d b)b = 0

(a + d c)c = 0

a3 0
Gi s c 6= 0. Khi b = 0 v a + d = c. T A =
, suy ra
c3 d3

0 0
2
2
3
c(a + ad + d ) = c . Sau ad = 0. Nh vy A c dng
hoc
c c


a 0
.
a 0




0 0
a a
Tng t nu b = 0 th A s c dng
hoc
.
b b
0 0
3

1. MA TRN

73


a 0
.
0 d
Vy A thuc mt trong nhng tp



a 0
2
: a, d R
0 d

Nu b = c = 0 th A =

hoc

R

1 1
0 0


R

0 0
1 1


R

1 0
1 0


R

0 1
0 1


.

Bi 1.19 (H Hng Vng, Ph Th). t

1
a1

2
3

U =
a2 , V = 1 a a a .
a3
Ta c V U = 4,

1
a
a 2 a3
a1 1
a a2
= A + I4 R A = U V I4
UV =
a2 a1 1
a
a3 a2 a1 1
Ta c
A2 = (U V I4 )2 = U V.V U U V U V + I4
= 4U V 2U V + I4 = 2U V + I4 = 2(A + I4 ) + I4 = 2A + 3I4
1
= R A2 2A = 3I4 A(A 2I4 ) = 3I4 R A1 = (A 2I4 )
3

2 a
a 2 a3
1
1 a
2 a a2

Vy A =
3 a2 a1 2 a
a3 a2 a1 2

0 1 1
0 1 , kim
Bi 1.20 (H Hng Vng, Ph Th). t M = 1
1
1
2
2
chng rng M = M, rank(M ) = 2. Suy ra:
ABC = M = M 2 = (ABC)2 .
Ta c: 2 = rank(ABC) = rank((ABC)2 ) = rank(AB(CAB)C) (CAB).

74
V CAB M2 (R) nn rank(CAB) 2. Do rank(CAB) = 2 hay CAB kh
nghch. T iu ny v h thc
(CAB)2 = C(ABC)AB = C(ABC)2 AB = (CAB)3
ta suy ra CAB = I2 . Vy
(BCA)2 = B(CAB)CA = BI2 CA = BCA.
l iu phi chng minh.
Bi 1.21 (C Ng Gia T). a thc c trng ca A l p(x) = x3 3x + 2 =
x(x 1)(x 2). Chia x2014 cho p(x) ta c dng biu din
x2014 = p(x).q(x) + ax2 + bx + c.
Ln lt thay x bi 0; 1; 2 trong ng thc trn, ta c c = 0; a + b = 1 v
4a + 2b = 22014 , ngha l a = 22013 1, b = 2 22013 , c = 0. Do p(A) = 0 nn
bng cch thay x bi A vo ng thc trn, ta c
A2014 = p(A).q(A) + (22013 1)A2 + (2 22013 )A = (22013 1)A2 + (2 22013 )A.
Bi 1.22 (C Ng Gia T). Ta c rank(AB I) = rank[(A I)B + (B I)]
rank[(A I)B] + rank(B I) rank(A I) + rank(B I).
Nhn xt. Bi ton ny s dng hai tnh cht quan trng ca hng ma trn
l
rank(A + B) rank(A) + rank(B) v rank(AB) rank(A).
Bi 1.23 (H Ngoi Thng - H Ni). t

1 7 5
un
A = 2 8 6 , Xn = vn .
4 16 12
wn
T gi thit ta c Xn+1 = AXn , n N. Hay
Xn = AXn1 = A2 Xn2 = . . . = An X0 .
Xt phng trnh c trng ca ma trn A ta c
|A I| = 0 ( 1)( 2) = 0 = 0, 1, 2.
Ma trn A c ba gi tr ring phn bit nn A cho ha c.
+ vi = 0, h phng trnh xc nh vc t ring l

x1 = x3

3
x1 7x2 + 5x3 = 0
x = 2x
6x2 4x3 = 0
2
3

x3 R.

1. MA TRN

75

Chn mt vc t ring a = (1, 2, 3).


+ Tng t vi = 1, = 2 chn c cc vc t ring tng ng l
b = (3, 2, 4), c = (1, 2, 2). Do ma trn lm cho ha A l

1 3 1
T = 2 2 2
3 4 2
Tnh ma trn nghch o ca T ta c

0
2 1
1 1
T 1 = 1
2 5 4

0 0 0
Ta c T 1 AT = 0 1 0 = D A = T DT 1 An = T Dn T 1 .
0
0 2

3 3.2n
Do Xn = An X0 = 2 3.2n .
4 6.2n
b. Theo phn a, c vn 2 = 3.2n nn ta c iu phi chng minh.
Bi 1.24 (H Ngoi Thng - H Ni). Chng ta xt cc trng hp sau:
TH 1: n = 1 th hin nhin r(A) = 0.
TH 2: n = 2 th hin nhin det A 6=
0 do r(A)
= 2.
0 d e
TH 3: n = 3 ma trn A c dng : a 0 f , trong a, b, c, d, e, f nhn
b c 0
cc gi tr l 1 hoc 2. Ta c det A = bdf + aec 6= 0. Do r(A) = 3.
TH 4: n > 3, ta thay tt c cc phn t n1 ca A bi 1 th nhn c ma trn
B c cc phn t cn li bng 1. R rng ta c det A det (B) (mod n 2).
M n 2 > 1 nn det A 6= 0 do r(A) = n.
Kt lun: vi n = 1 th r(A) = 0; vi n > 1 th r(A) = n.
Bi 1.25 (H Ngoi Thng - H Ni). Xt
fA (x) = det (A xI2 ) = x2 sx + p Q[x]
l a thc c trng ca A. Gi 1 , 2 l c nghim ca fA (x), khi 1 , 2
l cc gi tr ring ca A. Theo nh l Vit ta c:
1 + 2 = s Q; 1 2 = p Q.
Theo nh l Cayley-Hamilton ta c: A2 sA + pI2 = 02 . Vi C l gi tr
ring ca A th tn ti vc t ring X 6= 0 tha mn: AX = X. Bng quy
np ta c: An X = n X n = 1. Suy ra: n1 = n2 = 1.

76
Nu 1 R th 2 R, t vi n l ta c 1 = 2 = 1. Khi (A + I2 )2 =
A2 + 2A + I2 = 02 . Suy ra I2 = An = (A + I2 I2 )n = n(A + I2 ) I2 A =
I2 A3 = I2 .
Nu 1 6 R th 2 = 1 6 R. V |i | = 1 nn tn ti t sao cho 1 = cos t + i sin t
v 2 = cos t i sin t. Khi 1 + 2 = 2 cos t = s Q.
Li c: n1 = 1 cos nt + i sin nt = 1 cos nt = 1.
Nhn xt: Vi mi n th tn ti a thc n khi Pn (x) Z[x] bc n tha
mn:
2 cos nt = Pn (2 cos t).
V x = 2 cos t l nghim hu t ca Pn (x) + 2 = 0, nn 2 cos t Z. M
|2 cos t| 2 2 cos t {2; 1; 0; 1; 2}.
Nu 2 cos t = 2 1 , 2 R, xt trn.
Nu 2 cos t = 1, ta c A2 +A+I2 = 02 (AI2 )(A2 +A+I2 ) = 02 A3 = I2 ,
suy ra: An {I2 , A, A2 }, v l vi An = I2 .
Nu 2 cos t = 1, ta c A2 A+I1 = 02 (A+I2 )(A2 A+I2 ) = 02 A3 = I2 .
Nu 2 cos t = 0, ta c A2 + I2 = 02 .
Bi 1.26 (H Ngoi Thng - H Ni). a. Cc ma trn






1 1
0 1
0 1
A=
,B =
,C =
,
1 0
1
1
1
0
tha mn yu cu bi ton.
b. Vi A M2 (R) ta c: A2 (trA)A + (det A)I = 0. Gi s tn ti A, B, C
K2 (Z) sao cho A4 + B 4 = C 4 . t a = trA; b = trB; c = trC. Khi
A4 = (aA I)2 = a2 A2 2aA + I = (a3 a)A + (1 a2 )I.
Tng t cho B 4 v C 4 , ta c:
(a3 2a)A + (b3 b)B + (2 a2 b2 )I = (c3 2c)C + (1 c2 )I
Ta tnh vt hai v ta c:
a4 + b4 4(a2 + b2 ) = c4 4c2 2.
Suy ra: a4 + b4 c4 2(mod4), suy ra a, b l v c chn.
Tuy nhin, khi
a4 + b4 4(a2 + b2 ) 2
l iu v l.

(mod 8), v c4 4c2 2 2

(mod 8),

1. MA TRN

77

Bi 1.27 (H Nng nghip H Ni). Cho p l mt s t nhin ln hn hoc


bng 2. Ta c
Ap B BAp = Ap B Ap1 BA + Ap1 BA Ap2 BA2
+Ap2 BA2 . . . + ABAp1 BAp
p1
X
=
Apk BAk Apk1 BAk+1
k=0
p1

Apk1 (AB BA)Ak

k=0
p1

Apk1 AAk = pAp .

k=0

Nn
2014tr(A2014 ) = tr(2014A2014 )
= tr(A2014 B) tr(BA2014 ) = 0
Vy tr(A2014 ) = 0.
Bi 1.28 (H Phm Vn ng). Ta c a thc c trng ca ma trn A l


3 k

0
2



1k
2 = k 3 + 6k 2 3k 10.
P (k) = 0
2
2
2 k
P (k) c cc nghim 2, 1, 5. Theo nh l Caley-Hamilton th
A3 6A2 + 3A + 10 = 0.
Thc hin php chia a thc x2014 cho a thc x3 6x2 + 3x + 10, ta c
x2014 = (x3 6x2 + 3x + 10)g(x) + ax2 + bx + c.
Ln lt th x = 2, 1, 5 vo ta c h phng trnh theo a, b, c,

52014 22015 + 1

a
=
2014

18
4a + 2b + c = 2

6.22014 52014 12
b=
ab+c=1

18

2014

25a + 5b + c = 5

5.22014 52014 + 5

c =
.
9
Vy
A2014 =

52014 22015 + 1 2 6.22014 52014 12


5.22014 52014 + 5
A +
A+
I.
18
18
9

78
Bi 1.29 (H Phm Vn ng). Gi k l mt gi tr ring bt k ca A, ta c:
det(AkI) = 0. Suy ra: det(A+kI). det(AkI) = 0, dn n det(A2 k 2 I) =
0. T gi thit: A2014 = 0 nn A2 = 0 k = 0. Vy vi mi gi tr ring ki ca
A u bng 0.
Mt khc, gi tr ring k l nghim ca phng trnh:
xn + (1)n

n
X

aii xn1 + b2 xn2 + ... + bn1 x + bn = 0

i=1

Theo nh l Vi-et ta c:

n
P

ki = (1)n

i=1

n
P

aii

aii = 0. Vy Tr(A) = 0.

i=1

Bi 1.30 (HV Phng khng Khng qun). t

1 1 1 ... 1
1 1 1 ... 1
B=
...............
1 1 1 ... 1
Do

2015 2014 2014 . . . 2014


2014 2015 2014 . . . 2014

A=
............................
2014 2014 2014 . . . 2015

2014 + 1
2014
2014 . . .
2014
2014
2014 + 1 2014 . . .
2014
=
........................................
2014
2014
2014 . . . 2014 + 1

nn ta suy ra A = 2014B + In

(1). Ta c

n n n ... n
n n n ... n

B2 =
............... ,
n n n ... n
2 2 2
n n n . . . n2
n2 n2 n2 . . . n2
B 3 = B 2 .B =
...................
n2 n2 n2 . . . n2
Bng quy np, ta chng minh c
k1 k1 k1
n
n
n
. . . nk1
nk1 nk1 nk1 . . . nk1
Bk =
............................
nk1 nk1 nk1 . . . nk1

, vi mi s nguyn dng k

1. MA TRN

79

. T ta c
k

A = (2014B + In ) =

n
X

Cki 2014i B i Inki

n
X

Cki 2014i B i .

Hay ta c:
Ak = In + 2014Ck1 B + 20142 Ck2 B 2 + . . . + 2014k Ckk B k
T cc phn t ca Ak c xc nh nh sau:
Cc phn t trn ng cho chnh l: 1 + 2014Ck1 + 20142 Ck2 + . . . + 2014k Ckk .
Cc phn t cn li bng nhau v bng: 2014Ck1 + 20142 Ck2 + . . . + 2014k Ckk .
Bi 1.31 (H Qung Nam). T gi thit ta c
AB t = BAt , CDt = DC t , DAt CB t = I, ADt BC t = I.
Do
 


 t
  t
A B
D
B t
A B
D
B t
.
= I2n
= I2n
.
C t At
C D
C t At
C D

T Dt A B t C = I.
Bi 1.32 (H Quy Nhn). a. Ta d dng chng minh c rng
 det (A + tI)

t = .
lim tr(A + tI) = +, lim
t+
t+
tr(A + tI)
T vi t ln ta c
 det (A + tI)

1
2
A = (A + tI) tI =
(A + tI) +
t I
tr(A + tI)
tr(A tI)
s

2 
1
det (A + tI) 2
= p
(A + tI) +
(t
) (I)
tr(A + tI)
tr(A + tI)
s
!2
!2 

1
det (A + tI)
0 1
(A + tI) +
(t
)
.
= p
1
0
tr(A + tI)
tr(A + tI)
b. Gi s tn ti hai ma trn B, C M2 (R) tha mn iu kin bi ton. V
BC = CB nn
det (B 2 + C 2 ) = det (B + IC)(B iC) = | det (B + iC)| 0.
iu ny mu thun vi
2

det (B + C ) = det
Vy khng tn ti B, C M2 (R).

0 1
1 0


= 1 < 0.

80
Bi 1.33 (H Quy Nhn). T gi thit ca bi ton ta c.
(BA)3 = BABABA = B(ABAB)A = 0.
Suy ra BA ly linh. T y suy ra
BABA = (BA)2 = 0.
Khng nh trn ni chung khng ng vi A, B
ln hn 2. Tht vy, ta xt hai ma trn sau

0 0 1
0

0 0 0 ,B =
1
A=
0 1 0
0

l cc ma trn vung c cp

0 1
0 0 .
0 0

Khi ta c:

0 0 0
0 0 0
0 0 0
ABAB = 0 0 0 0 0 0 = 0 0 0 .
1 0 0
1 0 0
0 0 0
Nhng

0 1 0
0 1 0
0 0 0
BABA = 0 0 1 0 0 1 = 0 0 1 .
1 0 0
1 0 0
0 0 0

Bi 1.34 (H Sao ). t un = (xn , yn , zn ); un+1

1 3 3
= Autn ; A = 3 5 3 .
6 6 4

Xt |A tI| = 0 (t + 2)2
(t 4) = 0.
2
0 0

0 2 0 v ma trn chuyn c s P =
Ma trn cho ha B =
0 4
0

1
1 1
1 0 1
0 1 1 P 1 = 1 1
3 1 .
2
1 1
1
1 1 2

A = P BP 1 A2014 = P B 2014 P 1 , vi B 2014

Tm c A2014 suy ra x2014 = 22014 + 42014 .

22014
0
0
22014
0 .
= 0
0
0
42014

1. MA TRN

81

Bi 1.35 (H Sao ). Ta c

n 

cos x sin x
cos nx sin nx
=
sin x cos x
sin nx cos nx

sin
cos
10 + 2 5 p 5 1
18
18

= 4
A=


1 5
10 + 2 5
sin
cos
18
18

n
n
cos
sin
18
18 .
An = 4n
n
n
cos
sin
18
18
Bi 1.36 (H Sao ). Xt cc phng trinh Ax = 0 (1) v (A + A2 + +
An )X = 0 (2) .
Gi X0 l nghim ca phng trnh (1) suy ra X0 cng l nghim ca phng
trnh (2).
Gi s x0 l nghim ca (2) ta c Ax0 = A2 x0 An x0 = A2 Bx0 .
Vi B = E A A2 cldots An2 ; ta c AB = BA.
Ax0 = A2 Bx0 = ABA2 Bx0 = = B m Am Ax0 = 0.
Do phng trnh (1) v (2) cng c cng tp nghim. Suy ra:
rank(A) = rank(A + A2 + . . . + An ).
Bi 1.37 (H S phm H Ni 2). T gi thit ta c X, Y M2 (R) v X = Y ,
X 3 = I2 . T X 3 = I2 ta c cc gi tr ring ca X ch c th l nghim ca
a thc t3 1 = (t 1)(t2 + t + 1). a thc c trng Px (t) ca X l a thc
bc hai nn nu c nghim phc th c c hai nghim phc, do ch c th
xy ra cc kh nng:


Px (t) = (t 1)2
Px (t) = t2 + t + 1.
TH 1. Nu Px (t) = (t 1)2 ta c X 2 = 2X I2 nn thay vo X 3 = I2 ta c
2X 2 X I2 = 0. Do ta c X = I2 . Th li d thy X = I2 tha mn.
TH 2. Nu Px (t) = (
t2 + t + 1 th det X = 1, Tr(X) = 1. Do nu ma trn


a + d = 1
a b
X=
th
c d
ad bc = 1.
(


a + d = 1
a b
Ngc li nu X =
tha mn
th a thc c trng
c d
ad bc = 1
ca X l Px (t) = t2 +t+1. Theo nh l Cayley-Hamilton ta c X 2 +X +I2 = 0.
Do X 3 = I2 .

82
Bi 1.38 (H S phm H Ni 2). Ta chng minh det A 6= 0. Tht vy, gi

c1 , . . . ,
cn tng ng l cc vc t dng ca ma trn A = [aij ]. Khi :

c1 = (a11 , a21 , . . . , an1 ),


c2 = (a12 , a22 , . . . , an2 ), . . . . . .
cn = (a1n , a2n , . . . , ann )

Ta chng minh h vc t
c1 , . . . ,
cn l h c lp tuyn tnh. Tht vy, gi s:

1
c1 + 2
c2 + n
cn = 0

()

t |k | = max {|i | : i = 1, 2, . . . , n}. Nu k 6= 0 th t (*) xt ti ta th


k ta c:
1 a1k + 2 a2k + . . . + k akk + . . . + n ank = 0
Suy ra:
n
X
|i |
|i |
(
) = 2014
|aik |) 2014 <
2014 < |akk |
(
k
|k |
i=1,i6=k
i=1,i6=k
n
X

iu v l ny chng t k = 0, do t tnh cht cc i ca |k | suy ra


1 = 2 = . . . = n = 0. Vy det A 6= 0
Bi 1.39 (H S phm Thi Nguyn).

2014
0
0
0
0
2014

0
1
0
0
1
0

0 0 0 2014
0 0 1
0
1 1 0
0
1 1 0
0
0 0 1
0
0 0 0 2014

2.20142
0
0
0
0
2.20142

0
2
0
0
2
0

0 0 0 2.20142

0 0 2
0

0
0
2
.
B

0
0

0 0 2
0
2
0 0 0 2.2014

Tng t ta c

2014
0
0
0
0
2014

0
1
0
0
1
0

0 0 0 2014
0 0 1
0
1 1 0
0
1 1 0
0
0 0 1
0
0 0 0 2014

22013 .20142014
0
0
22013
0
0
0
0
0
22013
22013 .20142014
0

0
0

0
0

B 2014
0
0

0
0

0
22013
0
0
22013
0

2014

22013 .20142014
0
0
0
0
2013
2 .20142014

1. MA TRN

83

Trong

B=
4

Ta c B = 4I. Do B

2014


=

1 1
1 1

0
2107
2107 0

.

.

Bi 1.40 (H S phm Thi Nguyn). (i) Theo nh ngha.


(ii) Ta c tr(AB) = tr(BA) = tr((AB)T ). Theo (i), ta c
1
1
tr( (A B)(A B)T ) = tr(AAT AB T BAT + BB T )
2
2
1
= tr(AAT 2AB T + BB T ) 0.
2
1
1
tr( (B C)(B C)T ) = tr(BB T BC T CB T + CC T )
2
2
1
= tr(BB T 2BC T + CC T ) 0.
2
1
1
tr( (C A)(C A)T ) = tr(AAT AB T BAT + BB T )
2
2
1
= tr(AAT 2AB T + BB T ) 0.
2
Bi 1.41 (C S phm Vnh Phc). Ta c
(I A)(I + A + + A2013 ) = I A2014 = I
nn det(I A) 6= 0. Hn na,
(I A A1007 )(I A + A1007 ) = (I A)2 A2014 = (I A)2 ,
do det(I A A1007 ) 6= 0. Mt khc,
(I A)B = I A A1007 + A2014 = I A A1007 .
Do , bng cch ly nh thc hai v ta c det(B) 6= 0, ngha l B kh
nghch.
Bi 1.42 (H Tn Tro). Nu rank(A) = 0 th kt qu l hin nhin. Ta xt
trng hp rank(A) = 1. Khi cc dng l t l vi nhau theo mt t s no
. Khng gim tng qut, ta c


u1 v1 u1 v2 u1 v3 ... u1 vn
u1
u2 v1 u2 v2 u2 v3 ... u2 vn u2



= u3 v1 v2 v3 ... vn
u
v
u
v
u
v
...
u
v
A=
3
1
3
2
3
3
3
n


.
.
.
...
. ...
un v1 un v2 un v3 ... un vn
un

84


u1
v1
u2
v2


, V = v3 , suy ra A = U V T .
u
t U =
3


...
...
un
vn
P
Theo nh ngha vt ca ma trn, ta c: trace(A) = ni=1 ui vi = V T U , Do
A2 = (U V T )2 = U V T U V T = U (V T U )V T = trace(A).A.
Bi 1.43 (H Tn Tro). t A = (a b)I + bU , trong U = (1)kk . Ta c:
U n = k n1 U
Theo nh thc Newton ta suy ra kt qu:
n

A =

n
X

Cni (a b)ni bi U i

i=0
n

= (a b) I +

n
X

Cni (a b)ni bi U i

i=1

= (a b)n I +

n
X
i=1

1
Cni (a b)ni (kb)i U
k
n

1 X i
1
Cn (a b)ni (kb)i
= (a b) I (a b)n U + U
k
k i=0
n

= (a b)n I

1
[(a + (k 1)b)n (a b)n ] U
k

NH THC


Bi 2.1 (H An Giang). Gi s A =




m n
q n

. Ta c A =
v
p q
p m

d = mq np. Suy ra
det(A + dA ) = d[(d 1)2 + (m + q)2 ].
M det(A + dA ) = 0 v d 6= 0, nn d = 1 v q = m. Do


2m 2n

det(A dA ) = det
= det(2A) = 4d = 4.
2p 2q
Ch . Trong bi ton ny, biu din c nh thc di dng tng bnh
phng th i hi sinh vin c k thut v suy lun tt.

2. NH THC

85

Bi 2.2 (HV An ninh Nhn dn).

a1
a2

a3

A = ..
.

an1
an

Cho ma trn A Mn R nh sau

1 0... 0 0
0 1
... 0 0

0 0
... 0 0

.
..
..
. . .. ..
. . .
.
.

0 0
... 0 1
0 0
... 0 0

a. Hy tm mt vc t x Rn sao cho h vc t {x, Ax, A2 x, . . . , An1 x} c


lp tuyn tnh.
b. Gi s ma trn A ng dng vi ma trn ng cho diag(b1 , b2 , . . . , bn ).
Chng t rng cc s b1 , . . . , bn khc nhau tng i mt.
Bi 2.3 (H Bc Liu). Sau khi thay Fk (cos i ) bi cc gi tr ca n ri thc
hin cc bc sau:
bin i d2 d2 a11 d1 , ta c dng th hai c dng
a01 (cos 1 , cos 2 , , cos n );
ly a01 dng hai ra ngoi du nh thc lm nhn t chung,
bin i d3 d3 a12 d2 a22 d1 , ta c dng th hai c dng
a02 (cos2 1 , cos2 2 , , cos2 n );
.......................................
C tip tc qu trnh trn, ta c nh thc




1
1

1


cos 1

cos

cos

2
n
.
Dn = a01 a02 . . . a0,n1

.
.
.
.
.
.
.
.
.
.
.
.
.
.
.
.
.
.
.
.
.
.
.
.
.
.
.
.
.
.
.
.
.
.
.
n1

n1
n1
cos
1 cos
2 cos
n
y l nh thc Vandermonde, v ta c
Y
Dn = a01 a02 . . . a0,n1 (cos i cos j ).
i>j

Nhn xt. C th thay cos i bi xi , i = 1, . . . , n.


Bi 2.4 (HV Bu chnh Vin thng). Trng hp ma trn B kh nghch:
det (A + B) = det ((AB 1 + I)B) = det B. det (AB 1 + I)
B giao hon vi A dp B 1 cng giao hon vi A. A ly linh suy ra
X = AB 1 cng ly linh, do a thc c trng
PX () = det(X I) = ()n det (AB 1 + I) = PX (1) = 1

86
Trng hp B khng kh nghch, ta c det B = 0.
Vi mi s t nhin k 6= 0, xt ma trn Bk = B k1 I, Bk giao hon vi A
v tn ti N sao cho Bk kh nghch vi mi k N , do det (A + Bk ) =
det (Bk ), k N .
S dng tnh lin tc ca nh thc ta c:
det (A + B) = lim det (A + Bk ) = lim det Bk = det B
k

Bi 2.5 (HV Bu chnh Vin thng). a thc c trng ca ma trn A c th


biu din di dng:
PA () = det (A I) = ()n + C1 ()n1 + C2 ()n2 + . . . + Cn
Trong Ck l tng tt c cc nh thc con chnh bc k ca A.
c bit, Cn = det A, C1 = Tr(A) = 2013. V r(A) = 1 nn Ck = 0, vi mi
k 0. Vy
PA () = det (A I) = ()n + 2014()n1 ,
hay
det (A + I) = PA (1) = (1)n + 2014(1)n = 2015
Bi 2.6 (H Cng ngh thc phm Tp. HCM). Ta c
A3 + B 3 = (A + B)3 3AB(A + B)
= (C + I)3 3AB(A + B)
= [C 3 + I + 3C(C + I)] 3AB(A + B).
Suy ra
A3 + B 3 + C 3 + I = 3AB(A + B) 3C(C + I)
= 3(C + I)(AB C)
= 3(C + I)(AB + A + B + I)
= 3(A + I)(B + I)(C + I).
Do ,
A3 + B 3 + C 3 3(A + I)(B + I)(C + I) = I.
Nh vy
det[A3 + B 3 + C 3 3(A I)(B I)(C I)] = 1.
Bi 2.7 (H Cng ngh thc phm Tp. HCM). Xt nh thc cp n + 1 c
dng


n
1 x1 x21 . . . xn1
x
1
1


1 x2 x22 . . . xn1
xn2
2

(z) = . . . . . . . . . . . . . . . . . . . . . . . . .
(1).
n
1 xn x2n . . . xn1
x
n
n

1 z z 2 . . . z n1 z n

2. NH THC

87

Bng cch khai trin nh thc theo dng th n + 1 ta thy (z) l mt a


thc bc n c dng
(z) = an zn + an1 z n1 + . . . + a1 z + a0 .
Hn na, t (1) ta d dng thy rng x1 , x2 , ..., xn l cc nghim ca (z).
Do (z) ch c cc nghim l x1 , x2 , ..., xn . Theo nh l Viete, ta c
x1 + x2 + . . . + xn =

an1
,
an

ngha l
an1 = (x1 + x2 + . . . + xn )an .
D dng thy


1 x1 x21 . . . xn2
xn1
1

1 x2 x22 . . . xn2
xn2
2
an1 =
. . . . . . .
. . . . . . . . .2. . . . . . . . .n2
1 xn x . . . x
xnn
n
n


1 x1 x21 . . . x1n2 x1n1


1 x2 x22 . . . x2n2 x2n1
.

an =

.
.
.
.
.
.
.
.
.
.
.
.
.
.
.
.
.
.
.
.
.
.
.
.
.
.
.


2
n2
n1
1 xn x . . . x
x
n

Do ta c kt qu cn chng minh.
Bi 2.8 (D b).

1. iu kin Xi Xj = k ni rng
X

asi ajs = k.

T suy ra

d1
k

M = t AA = ..
.
k

k
d2

..
.

k
k
..
.

dm

2. K hiu e1 , . . . , em c s chnh tc ca Rm v C l vc t ct (phn t


ca Rm ) vi tt c cc h s bng k. Th th
det M = det (C + (d1 k)e1 ), . . . , C + (dm k)em ) .

88
T y, bng cch khai trin nh thc v s dng tnh tuyn tnh, cng
vi nhn xt rng cc nh thc ca cc ma trn m ct C xut
hin 2 ln bng 0, ta suy ra
n
Y
det M =
(di k)
i=1
n
X
+
det ((d1 k)e1 , . . . , (di1 k)ei1 , C, (di+1 k)ei+1 , . . . , (dn k)en ) .
i=1

T suy ra
Qm
i=1 (di k)
P
m
+
(di1Q k)k(di+1 k) (dn k)
Qm i=1 (d1 k) P
m
=
i=1
j6=i (dj k).
i=1 (di k) + k

det M =

3. Ch rng k dj vi mi i (v k = |Xi Xj | |Xi ) v ng thc


k = dj ch xy ra vi khng qu 1 ch s j. Ta suy ra det M l mt s
thc dng. Ni ring det M 6= 0, hay M c hng bng m. Nhng do
A Mn,m (R), t A Mm,n (R), ta suy ra rk t AA rkA n. T m n.
Bi 2.9 (H ng Thp). V A lun l nghim ca a thc c trng nn
A2 5A + 6I = 0. Vy

  2014
 

2 1
2
0
1 -1
2014
g(A) = A
=
.
.
.
1 1
0
32012
-1 2
Suy ra det(g(A)) = 62014 .
V f (A) = A nn
2014

[f (A)]

2014

=A


=


=

2
1

1
1

  2014
2
.
0

22015 32014
22014 32015

0
32012

2.32014 22015
2.32014 22014

 

1 -1
.
.
-1 2

.

Bi 2.10 (H ng Thp). Tnh nh thc ca tch


a1 a2
a3
a4
a1 a2
a3
a2 a1 a4 a3 a2 a1 a4


a3 a4 a1 a2 . a3 a4 a1
a4 a3 a2 a1
a4 a3 a2

T
a4
a3

a2
a1

Suy ra D(a1 , a2 , a3 , a4 ) = a21 + a22 + a23 + a24 .


Tng t i vi nh thc tch D(a1 , a2 , a3 , a4 )D(b1 , b2 , b3 , b4 ).

2. NH THC

89

Bi 2.11 (H Hng Vng, Ph Th). T gi thit suy ra aii chn v aij l


(i 6= j). Xt theo modulo 2, ta c



0 1 . . . 1 42013 42013 . . . 42013



1 0 ... 1 1

0
.
.
.
1



det A .. .. . . ..

..
..
..
.
.
. .

. .
.
.
.
.



1 1 ... 0 1
1
...
0





42013



Dn n






det A 42013


1
1
..
.

1 ...
0 ...
.. . .
.
.
1 1 ...

1 1... 1
0 1 . . . 0
..
..
. . . ..
.
.
.
0 0
. . . 1









0
1
1
..
.






42013 1.


Vy det A 6= 0.
Bi 2.12 (C Ng Gia T). Gi s det(B) 6= 0, ngha l B kh nghch. T gi
thit ta suy ra A + I = B 1 AB. Do trace(A) + trace(I) = trace(A + I) =
trace(B 1 AB) = trace(A), ngha l trace(I) = 0. y l iu mu thun. Vy
det(B) = 0.
Bi 2.13 (H Ngoi Thng - H Ni). Bt u t dng th nht n dng
th n 1, cng vo n tng ca tt c cc dng ng sau n, ta c.

x + n 1 x + n 1 x + n 1 x + n 1 ... x + n 1 x + n 1

n1
x + n 2 x + n 1 x + n 1 ... x + n 1 x + n 1


0
n2
x + n 3 x + n 1 ... x + n 1 x + n 1
Dn (x) =

.
.
.
.
.
.
...
...
...
...
...


0
0
0
0
...
1
x
Tip theo, vi j ln lt bng n 1, n 2, . . . , 1,
cng vo ct th j + 1, ta c

x+n1
0
0
0

n1
x1
1
0


0
n2 x1 2
Dn (x) =

.
.
.
...
... ...


0
0
0
0

nhn ct th j vi 1 ri

... 0
0
... 0
0
... 0
0
... ... ...
... 1 x 1

90
Sau , khai trin nh thc trn theo dng 1, ta c
= (x + n 1)Dn1 (x 1)
= (x + n 1)(x + n 3)Dn2 (x 2)
= ...
n
Y
=
(x + n 2i + 1).
i=1

Bi 2.14 (HV Phng khng Khng quan). Trc ht ta chng minh b


sau:
Cho P (x) = x2 +ax+b l tam thc bc hai vi h s thc v P (x) 0, x R.
A l ma trn vung cp n bt k. Chng minh rng det(P (A)) 0.
Tht vy, t gi thit suy ra P (x) c hai nghim phc lin hp , . hay ta c:
P (x) = (x )(x )
T ta suy ra:
P (A) = (A In )(A In ) = (A )(A )
Suy ra det(P (x)) = |(A In )|2 0. B c chng minh.
Xt a thc q(x) = x4 + 4 = (x2 + 2)2 4x2 = (x2 2x + 2)(x2 + 2x + 2) =
q1 (x)q2 (x)
Ta thy q1 (x) 0, x R, q2 (x) 0, x R (q1 (x) c hai nghim phc lin
hp l 1 i v q2 (x) c hai nghim phc hp 1 i). Theo b ta c :
det(q1 (A)) 0; det(q2 (A)) 0. T suy ra det(A4 + 4In ) = det(P (A)) =
det(q1 (A)) det(q2 (A)) 0. PCM
Du bng xy ra khi v ch khi hoc det(q1 (A)) = 0 hoc det(q2 (A)) = 0. Hay
a thc c trng ca A hoc nhn 1 i hoc nhn 1 i lm nghim.
Bi 2.15 (HV Phng khng Khng qun). Ta c


1

2
3
.
.
.
2013
2014


2
1
2
. . . 2012 2013

2
1
. . . 2011 2012
det A = D = 3
...
...
... ... ...
. . .

2013 2012 2011 . . .
2
1
Cng ct u v ct cui ta c

1
2
3

2
1
2


2
1
D= 3
...
.
.
.
.
..

2013 2012 2011


. . . 2013 2015
. . . 2012 2015
. . . 2011 2015
... ...
. . .
...
2
2015

2. NH THC

91

1
2
3

2
1
2


2
1
= 2015 3
...
.
.
.
.
..

2013 2012 2011

. . . 2013 1
. . . 2012 1
. . . 2011 1
... ... ...
...
2
1

Nhn hng th 2013 vi 1 ri cng vo hng cui, nhn hng 2012 vi 1


ri cng vo hng 2013, nhn hng th 2011 vi 1 ri cng vo hng th
2012,..., nhn hng u vi 1 ri cng vo hng th hai ta c:


1
2
3 . . . 2013 1

1 1 1 . . . 1
0

1 1 . . . 1
0
D = 1
... ... ... ... ... ...


1
1
1 ...
1
0
Khai trin nh thc theo ct cui cng, ta c:



1 1 1 . . . 1




1

1 1 . . . 1


2015


1
1
1
.
.
.
1
D = (1)
=
2015



... ... ... ... ...




1


1
1 ... 1

1 1 1 . . .
1
1 1 . . .
1
1
1 ...
... ... ... ...
1
1
1 ...

Cng hng cui vo hng cn li ta c





1 1 1 . . . 1




1


1
1
.
.
.
1



1
1 . . . 1 = 2015
D = 2015 1
... ... ... ... ...




1

1
1 ... 1

2 0 0
2 2 0
2 2 2
... ... ...
1 1 1

1
1
1
...
1

... 0
... 0
... 0
... ...
... 1

= 2015.22012
Bi 2.16 (H Qung Nam). T gi thit suy ra detA = 1, detB = 1. Do
detA 6= detB nn detA + detB = 0. Ta c
detB.det(A + B) = detB.det(A + B)t = det(BAt + I)
= det(AB t + AAt ) = detA.det(A + B)
= detB.det(A + B).
T suy ra detB.det(A + B) = 0. M detB = 1. Vy det(A + B) = 0.

92
Bi 2.17 (H Qung Nam). Ta chng minh A, B c chung mt vect ring.
Tht vy, gi s U l mt khng gian con ring ca A ng vi gi tr ring 1 .
Khi vi mi x U ta c:
A(Bx) = B(Ax) = 1 (Bx) Bx U
Ta nhn thy B nh x t U vo U . Do B c mt vect ring l phn t
ca U . V vy A, B c chung vect ring.
Bng cch chng minh tng t ta chng minh c A, B, C c chung mt
vect ring y 6= 0 ng vi cc gi tr ring 1 , 2 , 3 tng ng ca A, B, C.
Hin nhin rng 1 , 2 , 2 ph thuc tuyn tnh trong khng gian s phc. Do
tn ti cc s thc a, b, c khng ng thi bng 0 sao cho a1 +b2 +c3 = 0.
Khi (aA + bB + cC)y = (a1 + b2 + c2 )y = 0.
Vy det(aA + bB + cC) = 0.
Bi 2.18 (H Quy Nhn). V ai+6,j = (i + 6)j + j i+6 ij + j i = aij (mod3)
nn det A = 0, vi n 7.
Vi n = 6 tnh ton trc tip ta nhn c det A = 0
Vi n = 5 tnh ton trc tip ta nhn c det A = 12
Vy n = 5.
Bi 2.19 (H Sao ). Ta t

A=

1
1
..
.

Cn1 a1
Cn1 a2
..
.

Cn2 a21
Cn2 a22
..
.

...
...
..
.

Cnn an1
Cnn an2
..
.

1 Cn1 an+1 Cn2 a2n+1 . . . Cnn ann+1

bn1

bn1

bn3

bn1 bn1 bn1


2
3
1
B = ..
..
..
.
.
.
1
1
1

. . . bnn+1
. . . bn1
n+1
..
..
.
.
... 1

(a1 + b2 )n
(a1 + b2 )n
(a2 + b1 )n
(a2 + b2 )n
AB =

...
...
n
(an+1 + b1 ) (an+1 + b2 )n
|AB| = |A||B| = Cn1 Cn2 . . . Cnn

. . . (a1 + bn+1 )n+1


. . . (a2 + bn+1 )n

...
...
n
. . . (an+1 + bn+1 )

Y
ni>j

(ai aj )(bi bj ).

2. NH THC

93

Thay n = 2014 v ak = k; bk = k. Suy ra




0
1
22014 32014


1
0
1
22014

2014
2
1
0
1


.
.
.
.
.
.
.
.
.
.
..

20142014 20132014 . . . 22014

. . . |20142014
. . . 20132014
. . . 20122014
...
...
1
0

Bi 2.20 (H S phm H Ni 2). V B = AAt + I2014 l ma trn i xng


nn B c tt c 2014 gi tr ring u l s thc.

Gi l mt gi tr ring ca ma trn B ng vi vc t ring l


x 6= 0 . Ta
c:
Bx = x (AAt + 4I2014 )x = x AAt x = ( 4)x
Nhn c hai v bn tri vi xt ta c:
xt AAt x = ( 4)xt x (At c)t (At x) = ( 4)xt x 0.

T
x 6= 0 nn ta c 4 0 4.
Mt khc nh thc ca ma trn B l tch ca tt c cc gi tr ring ca n
k c bi nn nh thc ca B tha mn:
det B = det (AAt + 4I2014 ) 24028 .
Bi 2.21 (H S phm Thi Nguyn). (i) Theo nh l Caley-Hamilton A2
tr(A)A + det(A) = 0.
Ly vt hai v ta c iu phi chng minh.
(ii) Gi cc gi tr ring ca B l 1 , 2 , 3 , ta c: tr(B) = 1 + 2 + 3 ;
( + + )2 (2 +2 +2 )
tr(B 2 ) = 21 +22 +23 . T ta c 1 2 +1 3 +2 3 = 1 2 3 2 1 2 3 =
(tr(B))2 tr(B 2 )
.
2
a thc c trng ca B l P (x) = (1)3 (x 1 )(x 2 )(x 3 ) = (1)(x3
tr(A)x2 + (1 2 + 1 3 + 2 3 )x det(B)). Theo nh l Caley-Mamilton, ta
c
B 3 tr(B)B 2 + (1 2 + 1 3 + 2 3 )B det(B) = 0.
hay
(tr(B))2 tr(B 2 )
)B det(B) = 0.
2
Ly vt hai v ta c iu phi chng minh.
B 3 tr(B)B 2 + (

Bi 2.22 (H S phm Thi Nguyn). Ta c:




a1 1
a1 1 ... 1
1


1 a2 ... 1
1 1 a2

... = ... ...
D2014 = ... ... ... ...
1 1 ... ...
1 1 1

1 1 ... 1 a2014 1 1



... 1
1


... 1
1


... ...
...


... ...
1

... 1 a2014 1 + 1

94


a1 1 ... 1
a1 1 ... 1
0


1 a2 ... 1
1 a2 ... 1
0


... + ... ... ... ...
= ... ... ... ...
1 1 ... ...
1 1 ... ...
0


1 1 ... 1 a2014 1 1 1 ... 1


1
1
...
1
1

Khai trin nh thc u theo ct cui, cng ct cui nh thc th hai vi


cc ct trc n, ta c: D2014 = (a2014 1)D2013 + (a1 + 1)...(a2013 + 1) Mt
khc



a1 1 ... 1

1 a1 1 ... 1
1


1 a2 ... 1



1
1
a
...
1
1
2




... = ... ... ... ...
...
D2014 = ... ... ... ...

1 1 ... ...



1 1 1 ... ...
1


1 1 ... 1 a2014 1 1 ... 1 a2014 + 1 1

a1 1 ...

1 a2 ...

= ... ... ...
1 1 ...

0
0 ...



a1 1 ... 1

1
1
1


1 a2 ... 1

1
1
1




...
... + ... ... ... ... ...
1 1 ... ... 1
...
1


0 a2014 + 1 1 1 ... 1 1

Khai trin nh thc u theo dng cui, cng dng cui nh thc th hai
vi cc dng trc n, ta c: D2014 = (a2014 + 1)D2013 (a1 1)...(a2013 1).
Gi h trn, ta c
D2014 =

(a1 + 1)...(a2014 + 1) + (a1 1)...(a2014 1)


.
2

Bi 2.23 (C S phm Vnh Phc). Gi s p, q l cc s nguyn t ln hn


3. Suy ra p, q l cc s l v khng chia ht cho 3. Do p2 q 2 chia ht cho
8 v p2 , q 2 cng chia 3 d 1, ngha l p2 q 2 chia ht cho 24. Nh vy, bng
cch ly t dng th 2 n dng th n tr cho dng th nht ca det(A), ta
c tt c cc phn t t dng th 2 n dng th n ca det(A) u chia
ht cho 24. Do det(A) chia ht cho 24n1 .
Bi 2.24 (H Tn Tro). Lp nh thc D (z) c dng nh sau:

a1 + z x + z x + z

y + z a2 + z x + z

D (z) = y + z y + z a3 + z
.
.
.

y +z y +z y +z


... x + z
... x + z
... x + z .
...
.
... an + z

3. H PHNG TRNH TUYN TNH


95
P
v t B = ni,j=1 Aij , trong Aij l phn b i s ca cc phn t trong
nh thc D. S dng khai trin a tuyn tnh i vi D (z) ta c:

a1

X .

y
D (z) = D +

i .

y

...
...
...
...
...

z
.
z
.
z


... x
n
... .
X

... x = D + z
Aij = D + zB
i,j=1
... .
... an

Do ta c:
(

D (x) = D xB = (a1 x)(a2 x)...(a3 x)


D (y) = D yB = (a1 y)(a2 y)...(a3 y)

Suy ra
D=

x(a1 y)(a2 y)...(a3 y) y(a1 x)(a2 x)...(a3 x)


xy

H PHNG TRNH TUYN TNH

Bi 3.1 (H An Giang). H cho tng ng vi

(a11 + 1)x1 + a12 x2 + + a1n xn = 0

a12 x1 + (a22 + 1)x2 + + a2n xn = 0


........................................

an1 x1 + an2 x2 + + (ann + 1)xn = 0.


Ma trn h s ca h phng trnh c dng M = A + I. Ta c
M (A 2I) = (A + In )(A 2In ) = A2 A 2In = 2In .
Suy ra det(M ) 6= 0, ngha l M kh nghch. Do h cho ch c nghim
tm thng.
Ch . Bi ton ny c xy dng da trn tng phn tch a thc t2
t 2 = (t + 1)(t 2). Da trn tng ny, ta c th thay i h s 1 v
phi bi h s khc 0 v khc 1, ri p dng s phn tch
(t )(t + 1) = t2 t ( 1)
ta cng c kt qu tng t.

96
Bi 3.2 (H Bc Liu). Do k 6= 1 v k 6= 3 nn ma trn h s ca h phng
trnh c dng chnh tc theo dng l

2k + 2
1 0 0 0
k3

1+k

0 1 0 0

k 3 .

0 0 1 0 2(k 1)

k3

(k 1)2
0 0 0 1
k3
T y ta c kt qu cn chng minh.
Bi 3.3 (H Bch Khoa H Ni). t A = (aij ) Mn (Z). H cho tng

1 
ng vi A I X = 0. Ta c A l ma trn vi h s nguyn nn a
n
thc c trng p(t) ca A l a thc n khi (h s cao nht bng 1) vi
h s nguyn. Do p(t) khng th c nghim hu t khng nguyn. Suy ra

1
1 
6= 0, ngha l det A I 6= 0. Nh vy, h phng trnh trn ch c
p
n
n
nghim tm thng.
Ch . y l li gii khc vi li gii do tc gi xut, s dng tnh cht
ca a thc c trng v tnh cht nghim ca a thc.
Bi 3.4 (H Hng Vng, Ph Th). Bng cch t yi = xi 671(i =
1, 2, . . . , 2014), h cho thnh:

y1 + y2 + y3 = 0

y2 + y3 + y4 = 0

y3 + y4 + y5 = 0

y 4 + y 5 + y 6 = 0
.........

y2011 + y2012 + y2013 = 0

y2012 + y2013 + y2014 = 0

y2013 + y2014 + y1 = 0

y
2014 + y1 + y2 = 0
T phng trnh th nht v th hai suy ra y1 = y4 . T phng trnh th
t v th nm suy ra y4 = y7 . Tip tc qu trnh ny cui cng ta c
y1 = y4 = y7 = . . . = y2014 . Bng lp lun tng t, ta c y2 = y5 = y8 =
. . . = y2012 , y3 = y6 = y9 = . . . = y2013 . T hai phng trnh cui cng ta thu
c y2 = y2013 . T phng trnh u tin v phng trnh cui cng ta thu

3. H PHNG TRNH TUYN TNH

97

c y3 = y2014 . Kt hp cc khng nh trn suy ra y1 = y2 = y3 = . . . = y2014 .


Thay vo h ta c y1 = y2 = y3 = . . . = y2014 = 0. Vy nghim ca h
cho l x1 = x2 = x3 = . . . = x2014 = 671.
Bi 3.5 (C Ng Gia T). Cng v theo v tt c cc phng trnh ca h, ta
c
x1 + x2 + + x2014 = 1.
Tr phng trnh th k cho phng trnh th k + 1 (k < 2014), ta c
(x1 + x2 + + x2014 ) 2014xk = k (k + 1) = 1.
Suy ra xk =

P
2012
2
v x2014 = 1
xk =
.
2014
2014
k<2014

Ch . Ta c th thay s 2014 bi s t nhin n bt k.


Bi 3.6 (H Phm Vn ng). t

x1
x2

X=
...
xn
H cho tr thnh (A + 2014E)X = 0. Ly chuyn v hai v ta c X t (A +
2014X)t = 0. Dn n X t (A + 2014E) = 0, hay X t A + 2014X t E = 0. T
y suy ra X t AX + 2014X t EX = 0.
Ta c AX + 2014X = 0 AX = 2014X. Kt hp li, ta c
X t (2014X) + 2014X t X = 0 X t X + X t X = 0 X t X = 0.
Nh vy,

x1 x2

x1
 x2
2
2

... x2014
... = 0 x1 + ...x2014 = 0 x1 = ... = x2014 = 0
x2014

Bi 3.7 (H Sao ). Ta c h phng trnh tng ng

(1 + a1 b1 )x1 + a1 b1 x2 + . . . + a1 bn xn = c1

a b x + (1 + a b )x + . . . + a b x = c
2 1 1
2 2 2
2 n n
2

.........

an b1 x1 + an b2 x2 + . . . + (1 + an bn )xn = cn

98
Xt nh thc

1 + a1 b 1
a1 b 2

a2 b 1
1
+
a2 b 2
Dn =
...
...

an b 1
an b 2

...
a 1 bn
...
a 2 bn
...
...
. . . 1 + an b n





= Dn1 + an bn


D n = 1 + a1 b 1 + a2 b 2 + . . . + an b n = 1
j
Dn
= Dni
Theo Cramer h lun lun c nghim xi = D
n
Dni l nh thc D b di ct i thay bng (c1 , c2 , . . . , cn )T .
Do vy h lun c nghim nguyn (pcm).
Bi 3.8 (H S phm H Ni 2). Vit h phng trnh cho di dng
Ax =

1
1
x (A
I2014 )x = 0
2014
2014

1
Gi s det (A 2014
I2014 ) = 0. iu ny chng t =
ca A. V vy a thc c trng ca A l

1
2014

l mt gi tr ring

PA (t) = (1)n tn + (1)n1 Tr(A)tn1 + . . . + det A Z[t]


1
l mt nghim . T suy ra 2014 l c ca
a thc trn nhn = 2014
1
n
I2014 ) 6= 0, v vy h phng trnh
(1) . V l ny chng t det(A 2014
0
0

cho c nghim duy nht x =


. . . .
0

Bi 3.9 (H S phm Thi Nguyn). Tch ma trn h s di dng

a1
a2 a3 ... a2013 a2014
a2014 a1 a2 ... a2012 a2013

a2013 a2014 a1 ... a2011 a2012

..
.. ..
..
..

.
. .
.
.

a3
a4 a5 ... a1
a2
a2
a3 a4 ... a2014 a1

1 0 0 0 0 0
0 1 0 0 0 0
0 1 0 0 0 0
0 0 1 0 0 0

0 0 1 0 0 0
0 0 0 1 0 0

= a1
0 0 0 1 0 0 + a2 0 0 0 0 1 0

0 0 0 0 1 0
0 0 0 0 0 1
0 0 0 0 0 1
1 0 0 0 0 0

3. H PHNG TRNH TUYN TNH

0
1

0
+.... + a2014
0

0
0

99
0
0
1
0
0
0

0
0
0
1
0
0

0
0
0
0
1
0

0
0
0
0
0
1

0
1
0
0
0
0

0
0
1
0
0
0

0
0
0
1
0
0

0
0

1
0

1
0

0
0

0
0

0
B=
0

0
1

1
0
0
0
0
0

Theo trn, ta c A = a1 I + a2 B + ... + a2014 B 2013 . a thc c trng ca B


l P (x) = x2014 1. a thc ny c 2014 nghim trong trng s phc l
cc cn bc 2014 ca 1. Ta k hiu cc cn l 1 , 2 , ..., 2014 . Khi cc gi
tr ring ca A l P (1 ), P (2 ), ..., P (2014 ). Cc gi tr ny khc khng v P (x)
c cc nghim 2, 3, ..., 2014. T trn ta c det(A) = P (1 )P (2 )...P (2014 ) 6= 0.
Vy A ch c nghim tm thng x1 = x2 = ... = x2014 = 0.
Bi 3.10 (H Tn Tro). K hiu S l tp hp nghim ca h phng trnh.
Ln lt tnh cc n theo x1 , tch cc trng hp ch s chn, l ca n.
a. Vi n = 4 h phng trnh v nghim, nu a4 a3 + a2 a1 6= 0.
Nu a4 a3 + a2 a1 = 0, th h phng trnh c v s nghim. Khng gim
tng qut,
x1
x2
x3
x4
x5

R
= 2a1 x1
= 2a2 2a1 + x1
= 2a3 2a2 + 2a1 x1
= 2a4 2a3 + 2a2 2a1 + x1

Vi n = 5, ta c nghim ca h l:
x1
x2
x3
x4
x5

= a5 a4 + a3 a2 + a1
= a5 + a4 a3 + a2 a1
= a5 a4 + a3 a2 + a1
= a5 + a4 a3 + a2 a1
= a5 a4 + a3 a2 + a1

b.Trng hp tng qut:


(1). Nu n chn, n = 2k, k N . th:

100
S = , nu a2k a2k1 + + a2 a1 6= 0;
S = {x1 , 2a1 x1 , 2(a2 a1 ) + x1 , ..., 2(a2k1 a2k2 + + a1 ) x1 }, x1 R,
nu a2k a2k1 + + a2 a1 = 0.
(2). Nu n l, n = 2k + 1, k N , th:
x1 = a2k+1 a2k + + a3 a2 + a1
x2i+1 = a2k+1 a2k + a2i+2 + a2i+1 a2i + + a3 a2 + a1
x2i = a2k+1 + a2k + a2i a2i1 + a2i2 a3 + a2 a1
i {1, 2, ..., k}

KHNG GIAN VC T V NH X TUYN TNH

Bi 4.1 (HV An ninh nhn dn). a. C th ly x = (0, 0, . . . , 0, 1)t v d dng


kim tra n tha mn yu cu
b. Do A diag(b1 , b2 , . . . , bn ), tc l A cho ha c, nn tn ti mt c s ca
Rn gm n vc t ring c lp tuyn tnh ca A l {e1 , e2 , . . . , en }.
Vi x chn nh trn ta c
x = 1 e1 + 2 e2 + . . . + n en
Ax = 1 b1 e1 + 2 b2 e2 + . . . + n bn en
... = ...
An1 x = 1 bn1
e1 + 2 b2n1 e2 + . . . + n bn1
1
n en
V h vc t {x, Ax, A2 x, . . . , An1 x} c lp tuyn tnh, nn

1
2
...
n
1 b1
2 b2 . . . n bn

det
= D 6= 0
..
..
.
.
..
..

.
.
n1
n1
n1
1 b1
2 b2
. . . n bn
S dng nh thc Vandermonde ta c D = 1 2 . . . n

(bi bj ) 6= 0.

i>j

Suy ra cc s b1 , . . . , bn khc nhau tng i mt.


Bi 4.2 (H Bch Khoa H Ni). T gi thit ta c [dim(V1 + V2 ) dim V1 ] +
[dim V1 dim(V1 V2 )] = 1. M dim(V1 + V2 ) dim V1 dim(V1 V2 ). Do
[dim(V1 + V2 ) dim V1 ] = 0 hoc [dim V1 dim(V1 V2 )] = 0. Suy ra V2 V1
hoc V1 V2 , ngha l V1 V2 = V1 hoc V1 V2 = V2 . T ta c kt qu
cn chng minh.

4. KHNG GIAN VC T V NH X TUYN TNH

101

1 0 0
Bi 4.3 (H Bch Khoa Tp. H Ch Minh). t P = 0 1 0, Q =
1 1 1

1 0 1
a b s
0 1 1 v khng gian con W = c d s . W l khng gian con 5

0 0 1
s s 3s
chiu ca M3 (R) vi mt c s gm cc ma trn

1 0 0
0 1 0
0 0 0
A1 = 0 0 0 ; A2 = 0 0 0 ; A3 = 1 0 0 ;
0 0 0
0 0 0
0 0 0

0 0 0
0 0 1

A4 = 0 1 0 ; A5 = 0 0 1 .
0 0 0
1 1 3
Xt nh x f : M3 (R) M3 (R) tha f (A) = P AQ. V P, Q kh nghch nn f
l mt sng nh. chng minh dim(H) = 5, ta cn
chng t f (H)
= W.
a11 a12 s
Tht vy, xt A = (aij )3 H : f (A) = P AQ = a21 a22 s W, s =
s
s 3s
a11 + a12 + a13 .

1 0 1
0 1 1
0 H,
Thm na, f 1 (A1 ) = 0 0 0 H, f 1 (A2 ) = 0 0
1 0 1
0 1 1

0 0 0
0 0
0
1
1

1 0 1 H, f (A4 ) = 0 1 1 H, f 1 (A1 ) =
f (A3 ) =
0 1 1

1 0 1
0 0 1
0 0 1 H.
1 1 1
Nh vy f (H) = W , ta c iu phi chng minh.
Bi 4.4 (H Bch Khoa Tp. H Ch Minh). D thy A3 Xi = Xi , i = 1, 2, 3. V
X1 , X2 , X3 c lp nn A3 = I.
A l ma trn thc cp 3 (l) nn chc chn c 1 tr ring bng 1.
Hn na, (A2 + A + I)(X1 X2 ) = A2 (X1 X2 ) + A(X1 X2 ) + X1 X2 =
X3 X1 + X2 X3 + X1 X2 = 0 do det(A2 + A + I) = 0 tc A phi c
3 tr ring phn bit

1 = 1, 2 = 1 i 3, 3 = 1 + i 3.
t Yk = X1 + 2k X2 + 3 X3 . Ta c
AYk = X2 + 2k X3 + 3 X1 = k (2k X2 + k X3 + X1 ) = k Yk .

102
iu ny chng t Yk l VTR ca A ng vi TR k , k = 1, 2, 3.
Ta suy ra A cho ha c, tc A = P DP 1 vi D = diag(k ) v P =
[Y1 Y2 Y3 ].
kt thc, ta ch

1 2 3
P = [X1 X2 X3 ] 21 22 23
1 1 1
hay

A = [X1 X2 X3 ]P DP 1 [X1 X2 X3 ]1

0 0 1
= [X1 X2 X3 ] 1 0 0 [X1 X2 X3 ]1 .
0 1 0

Bi 4.5 (HV Bu chnh Vin thng). Trng hp tn ti Pj (x) 0 th mnh


ng
Gi s Pj (x) khng ng nht bng 0 i vi mi j.
Nu cc bc n1 , n2 , . . . , nr khc nhau, gi s n1 < . . . < nr th
ni i 1 n1 + . . . + nr

r(r 1)
v l.
2

vy tn ti i 6= j sao cho ni = nj = n.Pj (x) = axn + . . . , Pi (x) = bxn + . . .


t Qi = Pj ab Pi : deg (Qi ) < n, lc h {P1 (x), . . . , Pr (x)} ph
thuc h {P1 (x), . . . , Pi1 (x), Qi , Pi+1 (x), . . . , Pj (x), Pr (x)} ph thuc.
Tng ca bc ca h {P1 (x), . . . , Pi1 (x), Qi , Pi+1 (x), . . . , Pj (x), Pr (x)} nh
hn r(r1)
1. Tip tc qu trnh ny cui cng c h mi c a thc
2
ng nht bng 0, do ph thuc tuyn tnh.
Bi 4.6 (H Cng ngh thc phm Tp. HCM). t A = diag(a1 , a2 , . . . , an ), B =
(bij ) Mn (R). Do AB = BA nn (ai aj )bij = 0, i, j. Suy ra, nu ai 6= aj th
bij = 0. Do dim V chnh bng s cc cp ch s (i, j) sao cho ai = aj . Vi
2
mi cm , s cp ch s ai = aj = cm l rm
. Do dim V = r12 + r22 + + rk2 .
Bi 4.7 (D b). K hiu V l khng gian vc t gm cc a thc thun nht
theo X1 , . . . , Xn c bc d vi h s thc. Khi d thy V sinh bi cc n
thc c dng X1r1 X2r2 . . . Xnrn vi r1 , . . . , rn 0 v r1 + . . . + rn = d. H sinh
ny l mt h c s ca V . Nh vy, tnh chiu ca V ta ch cn m
s n thc nh trn, ni cch khc ta i m cc b s nguyn khng m
(r1 , . . . , rn ) l nghim ca phng trnh r1 + . . . + rn = d. Tnh ton ta s thy
chiu ca V l


n+d1
.
d

4. KHNG GIAN VC T V NH X TUYN TNH

103

Bi 4.8 (D b).
1. Hin nhin ta c ker f k ker f k+1 . T suy ra dy
cc khng gian con ker f k , k 0, l tng (th t bao hm). T suy
ra cc dy dim Im f k , k 0, v dim ker f k , k 0 tng ng l dy
tng. y l mt dy cc s t nhin, b chn bi n, chiu ca V. T
suy ra tn ti mt s nguyn m nh nht sao cho ker f m = ker f m+1 ,
ni ring ker f 0 ( ker f 1 ( ( ker f m . Ta s ch ra rng ker f k =
ker f k+1 vi mi m k. Gi s k m v x ker f k+1 . Th th do
0 = f k+1 (x) = f m+1 (f km (x)) ta suy ra f km (x) ker f m+1 . Gi thit
ker f m = ker f m+1 dn n f km (x) ker f m v do f k (x) = 0, hay
x ker f k . T ker f k+1 ker f k v nh vy ker f k = ker f k+1 . Nh
vy, dy ker f k , k 0, l mt dy tng thc s ti k = m v khng i
k t . Ni ring, ta phi c m n. Mt khc, ta nhc li nh l v
hng
dim ker f k + dim Im f k = dim V = n.
T ta cng d dng suy ra dy Im f k l gim thc s ti k = m ri
khng i k t . Khng nh c chng minh.
2. Ta c, vi mi k 0, f (ker f k+1 ) ker f k : nu x ker f k+1 th
f k+1 (x) = 0, ngha l f k (f (x)) = 0, hay f (x) ker f k . Tng t
f (ker f k+2 ) ker f k+1 . iu ny cho thy nh x f gi ker f k+2 ln
ker f k+1 ng thi gi khng gian con ker f k+1 ln ker f k . T suy ra
f cm sinh mt nh x tuyn tnh
f : ker f k+2 / ker f k+1 ker f k+1 / ker f k .
nh x f l mt n nh. Tht vy, gi s x ker f k+2 / ker f k+1 sao
cho f(
x) = 0. iu ny c ngha l f (x) ker f k (vi mi x ker f k+2
vi nh x trong ker f k+2 / ker f k+1 ). Ni cch khc f k+1 (x) = 0, hay
x ker f k+1 , v nh vy x = 0.
T suy ra dim(ker f k+2 / ker f k+1 ) dim(ker f k+1 / ker f k ). T suy
ra dim ker f k+1 dim ker f k dim ker f k+2 dim ker f k+1 v khng nh
c chng minh.
3. Ta c dim ker f m + dim Im f m . Do vy, ta ch cn chng minh tng
ker f m + Im f m l trc tip. Trng hp m = 0 l tm thng, ta gi
s m 1. Gi s x ker f m Im f m . Gi s y V sao cho x = f m (y).
Th th f m (y) = f 2m (x) = 0. Ta suy ra x ker f 2m v do x ker f m ,
hay f m (x) = 0. Nh vy y = 0 v tng cn chng minh l trc tip.
4. Gi s A Mn (R). Theo cc kt qu trn, p dng cho t ng cu
ca Rn tng ng vi ma trn A trong c s chnh tc, R = =Am
ker Am . Ch rng cc khng gian V1 = ker Am , V2 = Im Am ng nh
bi A v: A(ker Am ) ker Am1 kerAm , A(Im Am ) = Am+1 Am .

104
By gi, xt mt c s ca Rn c to thnh t mt c s ca V1 =
ker Am v mt c s ca V2 = Im Am . Th th ma trn ca A (tt hn
l ca t ng
 cu xy
 dng t A nh ni trn) trong c s nh
N 0
vy c dng
. kt thc, ta ch rng hn ch ca A xung
0 C
V1 l lu linh (x ker Am = Am (x) = 0) v hn ch ca A xung V2
l kh nghch v l mt ton cu (A(Im Am ) = Am+1 = Am .)
Bi 4.9 (D b). Mt mt, nu tn ti cc phn t x1 , . . . , xn sao cho ma trn
(fi (xj )) l kh nghch th f1 , . . . , fn hin nhin c lp tuyn tnh: nu khng,
mi quan h ph thuc tuyn tnh ca chng s cm sinh mt quan h ph
thuc tuyn tnh trn cc vector hng ca ma trn (fi (xj )).
o li, ta gi s f1 , . . . , fn l mt h c lp tuyn tnh. Ta s ch ra s tn
ti ca cc phn t x1 , . . . , xn vi tnh cht yu cu. Ta thit lp khng nh
ny bng qui np theo n. Trng hp n = 1 l r rng: ni rng h mt hm
s f l c lp ngha l ni f khng tm thng, ni cch khc, tn ti x
f (x) = 0. Gi s khng nh l ng cho n 1 v f1 , . . . , fn l mt h c
lp n cc hm s trn X. Ni ring, h f1 , . . . , fn1 cng c lp. Theo gi
thit qui np, tn ti cc phn t x1 , . . . , xn1 det(fi (xj ))1i,jn1 6= 0. Vi
x X, xt nh thc


f1 (x1 ) f1 (xn1 )
f1 (x)


..
..

.
. .
d(x) =
fn1 (x1 ) fn1 (xn1 ) fn1 (x)


fn (x1 ) fn (xn1 )
fn (x)
Bng cch khai trin theo ct cui cng ta thu c mt biu thc
d(x) = d1 f1 (x) + + dn1 fn1 (x) + dn fn (x),
trong di l i nh thc tng ng vi h s fi (x). Mt khc dn =
det(fi (xj ))1i,jn1 6= 0 v cc fi l c lp d(x) khng th ng nht bng
0. Gi s d(xn ) 6= 0. Th th b x1 , . . . , xn tho mn yu cu bi ra vi h
f1 , . . . , f n .
Bi 4.10 (D b).
1. Nu f cho ho c, mi c s cho ho f cng
cho ho f 2 v ker f = ker f 2 = khng gian ring ca f ng vi gi
tr ring 0. Tht vy, hin nhin ta c ker f ker f 2 v dim ker f =
dim ker f v c f v f 2 u c hng bng s cc gi tr ring 6= 0 ca f .
o li, gi s f 2 cho ho c v ker f = ker f 2 . Gi 1 , 2 , . . . , k l
cc gi tr ring 6= 0 ca f 2 . Vi mi 1 i k gi i l mt cn bc
2 ca i . Do X 2 i = (X i )(X + i ) v X i , X + i nguyn t
cng nhau (do iu kin i 6= 0) nn cng thc phn tch hch cho ta

4. KHNG GIAN VC T V NH X TUYN TNH

105

ker(f 2 i Id) = ker(f i Id) ker(f + i Id).


iu ny cho thy
Cn = ker f (ki=1 ker f i Id)
= ker f (ki=1 ker(f i Id) ker(f + i Id)).
ng thc trn cho ta phn tch ca Cn thnh tng trc tip cc khng
gian ring ca f v do f cho ho c.
2. K hiu A l ma trn cho trong bi v (e1 , . . . , en ) c s chnh tc
ca Cn . Th th A2 ei = ai an+1i ei vi mi i. Ni ring A2 l cho ho
c. Hn na, dim ker A = #{i; ai = 0} v dim ker A2 = #{i; ai an+1i =
0}. T suy ra dim ker A = dim ker A2 (ai = 0 an+1i = 0, i).
Bi 4.11 (H ng Thp). Chng minh dim(F1 + F2 + ... + Fn ) dim F1 +
dim F2 + ... + dim Fn bng quy np theo n.
Bi 4.12 (H Ngoi Thng - H Ni). Xt nh x tuyn tnh f : V1 V2
V3 V 2 vi f (x1 , x2 , x3 ) = (x1 x3 , x2 x3 )
f (x1 , x2 , x3 ) = 0 (x1 , x2 , x3 ) = (, , ) vi mi V1 V2 V3 .
Do dim (V1 V2 V3 ) = dim ker f
4019 dim V1 + dim V2 + dim V3 = dim V1 V2 V3 = dim ker f + dim Imf
Ta c
4029 dim ker f + dim V 2 = dim ker f + 4028
Nn dim V1 V2 V3 = dim ker f 1 hay V1 V2 V3 6= 0.
Bi 4.13 (H Nng nghip H Ni). Theo nh ngha ca bc ly linh, ta
c up1 6= 0. Hn na, uk 6= 0 vi mi 1 k p 1, v nu uk = 0 th
up1 = uk .up1k = 0, suy ra v l.
V up1 6= 0 nn tn ti x E sao cho up1 (x) 6= 0. Ta s chng minh h vct
{x, u(x), . . . , up1 (x)} c lp tuyn
Pp1tnh.k (2 im)
p
Ly (0 , 1 , . . . , p1 ) R m k=0 k u (x) = 0. Gi s
c t nht mt s k
khc khng. t i = min k {0, 1, . . . , p 1} : k 6= 0 . Ta c:
p1
X

k u (x) = 0

p1
X

k=0

k uk (x) = 0

k=i

Suy ra
p1i

p1
X
k=i

k u (x) = 0

p1
X
k=i

k up1i+k (x) = 0 (2 im)

106
Suy ra i up1 (x) = 0 (v vi k i + 1 th p 1 i + k p nn up1i+k =
0. Do i = 0 v up1 (x) 6= 0. Suy ra v l. Suy ra k = 0, k. Vy h
{x, u(x), . . . , up1 (x)} c lp tuyn tnh (1 im).
Bi 4.14 (H S phm H Ni 2). Gi s a0 id + a1 f + . . . + ap1 f p1 = 0, ai
R, i = 0, 1, . . . , p 1. Nhn f p2 vo hai v cu ng thc ta c a0 f p2 = 0.
T f p2 6= 0 ta suy ra a0 = 0. Do ta c:
a1 f + a2 f 2 + . . . + ap1 f p1 , ai R, i = 1, . . . , p 1
Nhn hai v vi f p3 vao hai v ca ng thc ta c a1 f p2 = 0. Do
a1 = 0. Thc hin tng t ta c a3 = a4 = . . . = ap1 = 0. Chng t h
cc t ng cu {idE , f, . . . , f p1 } l c lp tuyn tnh.
 Rb

Bi 4.15 (C S phm Vnh Phc). t A =
fi (x)fj (x)dx . Do det A = 0
a

nn h cc vct dng ca A ph thuc tuyn tnh, ngha l tn ti 1 , 2 , . . . , n


khng ng thi bng 0 sao cho vi mi j = 1, 2, ..., n ta c
n
X

 Zb

i=1


fi (x)fj (x)dx = 0.

Mt khc
n
X

i fi (x)

2

i=1

n
X
j=1

n
X


i fi (x)fj (x) ,

i=1

nn bng cch ly tch phn hai v ca ng thc trn ta suy ra


Zb h X
n
a

Do

n
P

i2

i fi (x) dx = 0.

i=1

i fi = 0, ngha l {f1 , f2 , . . . , fn } ph thuc tuyn tnh.

i=1

GI TR RING V VC T RING

Bi 5.1 (H Bch Khoa H Ni). Gi s l mt tr ring ca A. Khi tn


ti u 6= 0 sao cho Au = u. Do B ly linh nn tn ti s nguyn k 0 sao
cho v := B k u 6= 0 v Bv = B k+1 u = 0. Kt hp gi thit AB = BA, ta c
(A + B)v = Av = A(B k u) = B k (Au) = B k (u) = v, ngha l l tr ring
ca A + B.
Ngc li, nu l tr ring ca A + B th p dng chng minh trn vi hai
ma trn A + B v B ta c l tr ring ca (A + B) + (B) = A.

5. GI TR RING V VC T RING

107

Bi 5.2 (H Bch Khoa H Ni). Ta c

0
b
b2 . . . bn1
b1
0
b
. . . bn2

A=
. . . . . . . . . . . . . . . . . . . . . . . . . . ,
b1n b2n b3n . . . 0
nn



x
b
b2 . . . bn1
1
b
x
b
. . . bn2
.
det(A + xI) =
. . . . . . . . . . . . . . . . . . . . . . . . .
. 1n
b
b2n b3n . . . x

Bng cch nhn dng th i cho bi ri ly dng th i tr dng th nht ta


c


x

b
b2 . . .
bn1




0
1 2
1n 1 x b(x 1) 0 . . .

det(A + xI) = b .b . . . b
. . . . . . . . . . . . . . . . . . . . . . . . . . . . . . . . . . . .


1 x
0
0 . . . bn1 (x 1)


2
n
x
b
b
.
.
.
b
1


1 0 . . . 0
n1 b
= (x 1)
. . . . . . . . . . . . . . . . . . . . .
1n

b
0 0 ... 1
Tip tc ly ct th nht cng b1i ct th i ta c det(A + xI) = (x
1)n1 (x + n 1). Bng cch thay x bi t ta c a thc c trng ca A l
p(t) = (t + 1)n1 (t n + 1). Suy ra det(A) = (1)n p(0) = (1)n (1 n) v A
c 2 tr ring l 1 = n 1 (n) v 2 = 1 (bi n 1).
Bi 5.3 (C Ng Gia T). Gi thit ca bi ton tng ng vi
(A + B I)2 = (A I)(B I).
Do det(A + B I) = 0 khi v ch khi det(A I) = 0 hay det(B I) = 0.
Ngha l ta c iu phi chng minh.
Bi 5.4 (H Sao ). a. Nu A kh nghch AB v A1 ABA c cng a thc
c trng hay AB v BA c cng gi tr ring. Nu A khng kh nghch
det (A) = 0, tn ti m ln Ak = A Ak khng suy bin (k>m). Do vy
Ak B v BAk c cng gi tr ring.
|Ak B E| = |BAk E| Cho k pcm
b. Gi s A ly linh cp m, B l ly linh cp n ta c Am = 0; B n = 0. Xt
m+n
P k
(A + B)m+n =
Cm+n Ak B m+nk = 0
k=0

Suy ra A+B ly linh (pcm)

108
Bi 5.5 (H Tn Tro). Theo gi thit ta c AA = A A, A = A, ta c:
(Ax) Ax = x A Ax = x AA x = (A x) A x.
Mt khc, v l gi tr ring ca A, nn Ax = x, hay Ax x = 0. K hiu
l s phc lin hp ca s phc . Khi :
0 0 = (Ax x) (Ax x) = ((Ax) x )(Ax x)
= (x A x )(Ax x)
= x A Ax x A x x Ax + x x
= x AA x x A x x Ax + x x
= x A(A x x) x (A x x)
= (x A x )(A x x)
= [(x A x )] (A x x)
= (A x x) (A x x)
A x = x.
Do x l vc t ring ca ma trn A ng vi gi tr ring .

A THC

Bi 6.1 (H Bch Khoa H Ni). t g(x) =

n
P

f (k) (x). Ta c g(x) = f (x) +

i=1

g 0 (x) v deg g(x) = deg f (x) = n. Do f (x) 0, x R nn n l s chn. Do


g(x) t gi tr nh nht trn R. Gi s g(x) t gi tr nh nht ti x0 ,
khi g 0 (x0 ) = 0, nn g(x0 ) = f (x0 ) 0. Do g(x) g(x0 ) 0, x R.
Bi 6.2 (HV Bu chnh Vin thng). Ta s chng minh vi a thc bc n:
P (x) = xn + an1 xn1 + . . . + a1 x + a0 c y nghim thc th P 0 (a)
n
P
0 (x)
n1
1
=
n(P (a)) n 0 vi a > max {x1 , . . . , xn }. S dng ng thc: PP (x)
xxi
v phn tch: P (x) =
c

P 0 (a)
P (a)

n
P
i=1

cho n s dng

n
Q

i=1

(x xi ). Khi thay a vo cc ng thc trn ta

i=1
1
axi
1
axi

v P (a) =

n
Q

(a xi ). p dng bt ng thc Cauchy

i=1

> 0(i = 1, . . . , n) ta c:
n
X
i=1

1
n
rn
Q
a xi
n
(a xi )
i=1

6. A THC
Hay

P 0 (a)
P (a)

109

n

n

P (a)

(P 0 (a))n nn (P (a))n1 (v P 0 (a), P (a) > 0), thay

n = 2014 v ta c pcm.
Du bng xy ra khi v ch khi cc nghim bng nhau.
Bi 6.3 (HV Bu chnh Vin thng). Ta c f 0 (x) = 3x2 3 cho nn cc im
ti hn l x = 1, x = 1 v do cc gi tr ti hn l f (1) = 3, f (1) = 1.
Li c f (2) = 1, f (0) = 1, f (2) = 3 cho nn cc khng im ca f (x) nm
trong cc khong (2; 1), (0; 1), (1; 2). Nh vy ta ch cn m s ln f i
qua mi khong l s c s nghim ca f (f (x)) = 0. Tht vy, f i qua
(2; 1) ng mt ln (khi x < 2), i qua (0; 1) vi 3 ln (khi 2 < x <
1, 0 < x < 1, 1 < x < 2 ) v cui cng f i qua (1; 2) ng 3 ln. V vy tng
cng f (f (x)) = 0 c ng 7 nghim phn bit.
Bi 6.4 (H Cng ngh thc phm Tp. HCM). Gi s P (x) = an xn +an1 xn1 +
+ a1 x + a0 , vi an 6= 0. Suy ra P 0 (x) = nan xn1 + (n 1)an1 xn2 + + a1 .
- Nu n 2 th tha s bc cao nht ca P (P 0 (x)) l
n(n1)
an (nan xn1 )n = nn an+1
;
n x

tha s bc cao nht ca P 0 (P (x)) l


nan (an xn )n1 = nann xn(n1) .
Do , t P (P 0 (x)) = P 0 (P (x)) ta suy ra nn an = n, ngha l n = 1 v an = 1
(do an nguyn) mu thun.
- Nu n = 0 th t P (P 0 (x)) = P 0 (P (x)) suy ra P (x) = 0, x R.
- Nu n = 1 th P (x) = ax + b, nn t P (P 0 (x)) = P 0 (P (x)) suy ra b = a a2 .
Do P (x) = ax + a a2 , vi a nguyn.
Vy cc a thc tha mn yu cu ca bi ton l P (x) 0 v P (x) =
ax + a a2 , vi a nguyn.
Bi 6.5 (H Hng Vng, Ph Th). Nu P (x) = 0 l mt hng s th ta
c a2 = a a = 0 hoc a = 1. Vy P (x) = 0 hoc P (x) = 1.
Gi s deg P (x) > 0. D thy h s cao nht ca P (x) phi bng 1. Gi x0 C
l mt nghim ca P (x). Khi x0 1 cng l mt nghim ca P (x + 1). Suy
ra
x20 + x0 + 1; (x0 1)2 + (x0 1) + 1 = x20 x0 + 1
u l nghim ca P (x). Gi s x0 l nghim c mun ln nht trong s tt
c cc nghim ca P (x). Khi ta c |x0 | |x20 + x0 + 1|, |x0 | |x20 x0 + 1|.
Suy ra
2|x0 | |x20 + x0 + 1| + |x20 x0 + 1| |(x20 + x0 + 1) (x20 x0 + 1)| = 2|x0 |.
iu ny dn n |x0 | = |x20 + x0 + 1| = |x20 x0 + 1|, x20 + x0 + 1 = (x20
x0 + 1), R, 0. Nu = 0 th x0 = 0. T trn suy ra x2 + x0 + 1 = 0 v

110
l. Do > 0 v d dng suy ra = 1. Vy x20 + 1 = 0. V P (x) R[x] nn
tn ti Q(x) R[x], Q(x) khng chia ht cho (x2 + 1) v s nguyn dng n
P (x) = (x2 + 1)n Q(x). D kim tra c a thc Q(x) tha mn
Q(x)Q(x + 1) = Q(x2 + x + 1).
Lp lun tng t nh trn suy ra Q(x) = 0 hoc Q(x) = 1.
Vy c p s P (x) = 0, P (x) = 1 hoc P (x) = (x2 + 1)n , n N .
Bi 6.6 (H Hng Vng, Ph Th). Nu P (x) = 0 l mt hng s th ta
c a2 = a a = 0 hoc a = 1. Vy P (x) = 0 hoc P (x) = 1.
Gi s deg P (x) > 0. D thy h s cao nht ca P (x) phi bng 1. Gi x0 C
l mt nghim ca P (x). Suy ra x20 cng l mt nghim ca P (x). Do ta
c dy s x0 , x20 , x40 , . . . , x2n
0 , . . . gm nhng nghim ca P (x).
2
4
2
4
Nu |x0 | < 1 th x0 , x0 , x0 , . . . , x2n
0 , . . . phn bit v |x0 | > |x0 | > |x0 | > . . . >
2n
|x0 | > . . . . iu ny v l v P (x) khng th c v s nghim.
4
2
Nu |x0 | > 1 th x0 , x20 , x40 , . . . , x2n
0 , . . . phn bit v |x0 | < |x0 | < |x0 | < . . . <
2n
|x0 | < . . . . iu ny v l v P (x) khng th c v s nghim. Do mi
nghim x0 ca P (x) u tha mn |x0 | = 1. Gi n = deg P (x), a1 , . . . , an l
nghim ca P (x) : P (x) = (x a1 )(x a2 ) . . . (x an ).
Suy ra P (x2 ) = (x2 a1 )(x2 a2 ) . . . (x2 an ) v P (x 1) = (x (a1 +
1))(x (a2 + 1)) . . . (x (an + 1)). T h thc P (x)P (x 1) = P (x2 ) suy ra
a1 + 1, a2 + 1, . . . , an + 1 cng l nghim ca P (x2 ). Nh vy cc phn t ny
thuc tp hp

{ a1 , a2 , . . . , an , a1 , a2 , . . . , an } ,
v do cng c mun bng 1. Vit ak = cos k + i sin k . Khi
1
1 = |ak + 1| = | cos k + i sin k + 1| cos k = .
2
Suy ra ak = 21 +

3
i
2

hoc ak = 12

3
i.
2

Do P (x) ch gm cc nhn t

1
3
1
3
(x + +
i)(x +
i) = x2 + x + 1.
2
2
2
2
Vy P (x) = (x2 + x + 1)m , m N
p n: P (x) = 0 hoc P (x) = (x2 + x + 1)m , m N .
Bi 6.7 (C Ng Gia T). Nu ngi i trc vit s 1 vo h s ca x th
f (x) = x(x2 1) + x2 + . Do chin thut lun thng ca ngi i trc
l: Nu ngi chi sau vit s a vo mt du th ngi i trc vit s a
vo du cn li.

6. A THC

111

Bi 6.8 (H Nng nghip H Ni). Ta c P (x) = a0 khng tha mn vy


P (x) c dng
P (x) = an xn + an1 xn1 + . . . + a1 x + a0 ,
vi an 6= 0. Khi khai trin 2 v ca (1) th s hng c bc ln nht ca
P (P (x)) + 1 l
n2
an (an xn )n = an+1
n x

2
cn s hng c bc ln nht ca P 2 (x) + 2P (x) + (x2 + 3x + 1)2 l

nu n < 2
x
2
8
(an + 1)x nu n = 2

4 4n
an x
nu n 2.
Nu n 2 th trc ht ta phi c
2

xn = x8 n2 = 8
iu trn v l v n l s t nhin. Vy n > 2 v
2

n
an+1
= a4n x4n
n x

Suy ra
(
an+1
= a4n
n
n2 = 4n

(
n=4

an = 1

Do vy ta cn tm a thc dng
P (x) = x4 + a3 x3 + a2 x2 + a1 x + a0 .
Ta t G(x) = P (x) (x2 + 3x + 1)2 + 1. Vy (1) tng ng vi

2
P (P (x)) + 1 = P 2 (x) + 3P (x) + 1 G(x)

2


P (P (x)) P 2 (x) + 3P (x) + 1 + 1 = G(x) G(x) 2(P 2 (x) + 3P (x) + 1)


G(P (x)) = G(x) G(x) 2(P 2 (x) + 3P (x) + 1)
Nu G(x) khc 0, ta t k = degG, (k 3) nn ta c 4k = k + 8. Suy ra v l
v k l s t nhin, vy G(x) = 0, x. T ta c
P (x) = (x2 + 3x + 1)2 1 = x(x + 1)(x + 2)(x + 3).

112
Bi 6.9 (H Phm Vn ng). Do x1 , ..., xn l nghim nn P (x) =
xi ). Ta c:
n Y
n
X
0
P (x) =
(x xi )

Qn

i=1 (x

j=1 i=1

Q
x xj
xn + ni=1 (x xi )
xi xj
Ta c f (x1 ) = ... = f (xn ) = 0 v bc f (x) n 1. Do f (x) = 0
P
xni
a thc f (x) c h t cao nht l ni=1 Qn
(x1 + ... + xn ) = 0
j=1 (xi xj )
P
xni
Theo Viet x1 + ... + xn = 1. Vy ni=1 Qn
= 1.
j=1 (xi xj )
Xt a thc: f (x) =

Pn

i=1

xni

Qn

j=1

Bi 6.10 (HV Phng khng Khng qun). Gi x1 , x2 , . . . , xm l cc nghim


ca Q(x) (k c bi). Gi s mun ca chng khng vt qu 2014. Theo
nh l Vit ta c:

|b1 | = |(x1 + x2 + . . . + xm )| 2014.m

.........



P
k
k
|bk | = (1)
x1 x2 . . . xk 2014k .Cm
(1)

i1 <i2 <...<ik

.........

m
|bm | = |(1) x1 x2 . . . xm | 2014m
k
. Nh vy a thc Q(x)
R rng (1) mu thun vi gi thit |bk | > 2014k .Cm
c t nht mt nghim y tha mn bt ng thc:

|y| > 2014.

(2)

Do a thc P (x) chia ht cho a thc Q(x) nn ta c P (y) = 0. Gi s


|ai | 2013, i = 1, n.

(3)

Khi ta c


0 = |P (y)| = y n + a1 y n1 + a2 y n2 + . . . + an1 y + an


|y n | a1 y n1 . . . | |an1 y| |an |.
(4)
T (2) v (3) ta c:
|ai | 2013 < |y| 1; i = 1, n.

(5)

T (4) v (5) ta nhn c:




0 = |P (y)| |y n | (|y| 1)( y n1 + . . . + |y| + 1) = 1
Vy tn ti i (i = 1, 2, . . . , n) sao cho |ai | > 2013.

(mu thun)

6. A THC

113

Bi 6.11 (H Qung Nam). Gi s a1 , a2 , ..., a33 l 33 nghim nguyn ca a


thc P (x) 2014.
Do P (x) chia ht cho Q(x) trn vnh Z[x] nn Q(a1 ), Q(a2 ), ..., Q(a33 ) chia
ht cho 2014 hay Q(ai )inA = {1, 1, 2, 2, 1007, 1007, 2014, 2014}. (A c
8 phn t.). Theo nguyn l Dirichlet, tn ti mt gi tr a A sao cho
Q(ai1 ) = Q(ai2 ) = Q(aia = Q(ai5 ) = a 6= 0.
Vy deg Q(x) 5.
Bi 6.12 (H S phm H Ni 2). Khng tn ti a thc P (x), Q(x) R[x]
sao cho:
P (x)
= x2 + 4x + 2014.
Q(x)
Tht vy, gi s tn ti a thc P (x), Q(x) R[x] sao cho
P (x)
= x2 + 4x + 2014.
Q(x)
Th th Q(x) 6= 0, x R v deg P = deg Q + 1. T Q(x) 6= 0, x R suy
ra deg Q = 2k, k N v khi a thc P (x) c bc l s l. V vy tn ti
P (x0 )
s x0 R sao cho P (x0 ) = 0, tc l Q(x
= 0 x20 + 4x0 + 2014 = 0
0)
(x0 + 2)2 + 2010 = 0. iu ny v l chng t khng tn ti cc a thc
P (x), Q(x) R[x].
Bi 6.13 (H S phm Thi Nguyn). Gi s P (x) l a thc bc n c n
nghim a1 , ...., an . Nu x = ai l nghim ca P (x) th ta c iu phi chng
minh. Gi s x khng l nghim ca P (x). Khi
n

P 0 (x) X 1
=
.
P (x)
x

a
i
i=1
Ly o hm hai v, ta c
n

X 1
P 0 (x)P (x) (P 0 (x))2
=

< 0.
P (x)2
x ai
i=1
T ta c iu phi chng minh.
Bi 6.14 (C S phm Vnh Phc). f (x) c bc 2014 nn F (x) c bc 2015,
m F (x) c t nht 2014 nghim thc nn F (x) c ng 2015 nghim thc
(k c s bi). t G(x) = F (x) + f (x), suy ra G(x) l a thc c bc 2015.
Nu = 0 th hin nhin G(x) c ng 2015 nghim thc.

114
Nu 6= 0 th ta t H(x) = ex/ F (x). Khi H 0 (x) = ex/ G(x). M F (x)
c ng 2015 nghim thc nn H(x) c 2015 nghim thc, suy ra H 0 (x) c
t nht 2014 nghim thc, ngha l G(x) c t nht 2014 nghim thc. M
G(x) l a thc c bc 2015, nn G(x) c ng 2015 nghim thc.
Ghi ch. Bi ton ny s dng hai nh l c in ca ton hc:
nh l 1. Mi a thc bc n(n 1) vi h s thc c ti a n nghim thc.
nh l 2. Nu hm thc f kh vi trn R v phng trnh f (x) = 0 c t nht
n nghim thc th phng trnh f 0 (x) = 0 c t nht n 1 nghim thc (k c
s bi).
Bi 6.15 (H Tn Tro). Ta xt cc trng hp sau
1. a = b = 0 th P (x) l ty ;
2. a = 0, b 6= 0 th P (x) = 0 vi mi x;
3. a 6= 0, b = 0 th P (x) = c, c R l mt hng s ty ;
4. a 6= 0, b 6= 0;
/ N, thay x = b vo gi thit, ta c x = b a. Thay x = b a
- Nu ab
vo gi thit, ta li c x = b 2a l nghim... C tip tc nh vy, suy
ra P (x) = x, vi mi x R;
- Nu ab N th a thc P (x) c x = a, x = 2a, ..., x = (n 1)a l cc
nghim. Khi ta c:
P (x) = (x a)(x 2a)...[x (n 1)a]Q(x).
Thay P (x) vo gi thit ta c Q(x a) = Q(x), vi mi x R, tc l
Q(x) = k, vi k R l mt hng s ty . Do :
P (x) = (x a)(x 2a)...[x (n 1)a].

CC BI XUT: GII TCH

DY S

Bi 1.1 (HV Bu chnh Vin thng). Ta c


xn+1 xn =
Do vy, xn+1 > xn

n+1
> 0.
(n + 2)!

n = 1, 2, , 2013. Nh vy
0 < x1 < x2 < < x2014 .

Khi
x2014 <

p
1
n
xn1 + xn2 + + xn2014 < 2014 n x2014 .

Cho n +. Ta c I = x2014 .
Mt khc, t
k
1
1
=

(k + 1)!
k! (k + 1)!
suy ra
xk = (1

1
1
1
1
1
1
) + ( ) + + (
)=1
.
2!
2! 3!
k! (k + 1)!
(k + 1)!

Do vy
x2014 = 1

1
.
2015!

Suy ra
I =1

1
.
2015!

Bi 1.2 (H Cng ngh thc phm Tp. HCM). .


a) Xt hm s



2
f (x) = ln x + 1 + x
l l v lin tc trn on [1, 1] nn

R1

f (x)dx = 0.

Chia on [1, 1] thnh 2n on c di bng nhau bi cc im chia

116
xi = 1 + ni . Trn mi on con [xi1 , xi ], ly im i = xi , i = 0, 2n. Khi ,
tng tch phn Riemann ca hm s f (x) l

s

2
2n
2n
X
X
1
i
i
Sn =
f (i )xi =
ln 1 + + 1 + 1 +
n i=1
n
n
i=1
Do ,
Z1
lim Sn =

f (x)dx =0.

Mt khc, ta c

s

2
h
1
1
1n
Sn =
ln 1 + + 1 +
+
n
n
n

s

2 i
2n n
2n
+ 1+
+ ln 1 +
n
n
q
q

2 + (1 n)2
2 + (2n n)2
1

n
+
n
2n

n
+
n
1

= ln
+ + ln
n
n
n
q

n1
2n
Y
i n + n2 + (i n)2

.
= ln
2n
n
i=1
T suy ra
q

n1
2n
Y
i n + n2 + (i n)2

= 1.
lim
2n
n
n
i=1
b) T gi thit ta c
bn+1 =

an + 3bn
4

1
(an bn ). Suy ra
4
 n
1
an+1 bn+1 =
(a b) 0, n
4

suy ra an+1 bn+1 =

Ta cn chng minh dy {an } hi t.


Gi s a b, ta c a2 b2 , ...., an bn b, suy ra
an+1 =

an b n
an an
+

+
= an .
2
2
2
2

(1)

1. DY S

117

Nh vy, dy {an } l n iu tng, b chn trn bi b. Do , tn ti gii


hn lim an .
n

T (1), ta c
lim an = lim bn .

Trng hp a > b, chng minh tng t.


Bi 1.3 (D b). Xt dy thun nht xn+1 = 2xn , suy ra xn = 2n1 c.
By gi xt dy cho bi xn+1 = 2xn + 3n (). Ta tm nghim di dng
xn = An + B. Thay vo (*) ta c A = B = 3.
Xt dy cho bi xn+1 = 2xn + 2n (). Ta tm nghim di dng xn = An2n .
1
Thay vo (**) ta c A = .
2
1
Suy ra nghim ca bi ton c dng xn = c2n1 3n 3 + n2n . Thay n = 1,
2
suy ra c = 2. Vy xn = (n 2)2n1 3n 3. Thay n = 2013 ta c kt qu
cn tm.
Bi 1.4 (D b). 1. Ta c
an
an+1
= 2
1.
an
an an + 1
Do dy s l gim. Mt khc d chng minh an 1 nu a1 1 v
an < 1 nu a1 < 1 T tnh c gii hn limn an = 1 nu a1 1 v
limn an = 0 nu a1 < 1.
2. Do
an 1
a2n
1= 2
.
an+1 1 = 2
an an + 1
an an + 1
Nn an =

1
an+1 1

1
.
an 1

Ch ra gii hn limn
1
nu a1 < 1.
1a1

Pn

n=1

ai = nu a1 1 v limn

Bi 1.5 (H Hng Vng, Ph Th). Xt hm s


f (x) = 2013 ln(x2 + 20142 ) 20142 ,
vi x R. D thy f lin tc trn R v xn+1 = f (xn ).
f 0 (x) = 2013

2x
2013
0

|f
(x)|

.
x2 + 20142
2014

Xt hm s
g(x) = x f (x),

x R.

Pn

n=1

ai = 1 +

118
Ta c g 0 (x) = 1 f 0 (x) > 0, vi mi x R, dn n hm g(x) ng bin trn
R v g(x) lin tc trn R. Ta c
g(0) = f (0) = 2013 ln(20142 ) + 20142 > 0,
g(20142 ) = 2013 ln(20142 + 20144 ) < 0.
Do g(0).g(20142 ) < 0 v tn ti c (20142 ; 0) tha mn g(c) = 0, hay
f (c) = c. Do g ng bin nn c l duy nht.
Li c xn+1 = f (xn ) nn vi mi n tn ti qn nm gia c v xn tha mn
f (xn ) f (c) = f 0 (qn ).(xn c).
Suy ra
|xn+1 c| = |f 0 (qn )|.|xn c|

2013
.|xn c|.
2014

Vy
2013
2013 n1
|x1 c|.
.|xn1 c|
2014
2014
Cho n , ta c |xn c| 0 v
|xn c|

lim xn = c.

Bi 1.6 (C Ng Gia T). + Tm un + 1 v un 1.


+ Tnh c
 u 1 2n1  2013 2n1
un 1  un1 1 2
1
=
==
=
un + 1
un1 + 1
u1 + 1
2015
Cho nn

n1

2.20152
un =
1.
(2015)2n1 20132n1
Vy
lim un = 1.
Bi 1.7 (H Qung Bnh). Ta c bt ng thc ln(1 + x) < x vi x > 0 do
ln(1 + n1 ) < n1 vi n N .
Do
1
> ln(1 + n) ln n, n N .
n
Suy ra
1 1
1
1+ + +...+ (ln 2ln 1)+(ln 3ln 2)+...+(ln(n+1)ln n) = ln(n+1),
2 3
n

1. DY S

119

v
0<

1
1
2

1
3

1 + + + ... +

1
n

1
.
ln(n + 1)

1
=0
n ln(n + 1)
lim

suy ra
lim

1
1
2

1
3

1 + + + ... +

1
n

= 0.

Bi 1.8 (H Qung Nam). T gi thit ta c


(n = 1)un+1 un = 2(nun un1 ).
t vn = (n = 1)un+1 un , suy ra
vn = 2vn1 = ... = 2n1 v1 = 2n1 .
Khi ,
(n + 1)un+1 un = 2n1 ,
suy ra
(n + 1)!un+1 n!un = 2n1 n!.
t wn = n!un , ta c
wn =

n2
X

2i (i + 1)!.

i=0

Vy
n2

1 X i
2 (i + 1)!.
un =
n! i=0
Bi 1.9 (H Quy Nhn). V ng thc

n
n
n
1 + 2 + ... + 2014 = n a1 + n a2 + ... + n am .

(2)

xy ra vi mi n N nn khi n , s dng gii hn c bn lim

a=1

vi a > 0, ta c m = 2014. Suy ra

n
n
n
n( 1 1) + n( 2 1) + ... + n( 2014 1)

= n( n a1 1) + n( n a2 1) + ... + n( n am 1) (3)
Ch rng

ax 1
= ln a, a > 0.
x0
x
lim

120
Do ,

lim n( n a 1) = ln a, a > 0.

Trong (3), cho n ta c


ln 1 + ln 2 + ... + ln 2014 = ln a1 + ln a2 + ... + ln am .
Vy P = a1 a2 ...am = 2014!
Bi 1.10 (H Sao ). Chng minh rng c vi mi x > 1 ta c:

x
x
< 1+x1< .
2+x
2
p dng cho x =

k
,
n2

ta c:

k
k

2
n + 2n
k + 2n2
Do

n
X
k=1

r
1+

k
k
1 2
2
n
2n

k, n 1.

r
n
n
X
X
k
k
k
( 1 + 2 1)
xn =
.
2
2
n + 2n
n
2n
k=1
k=1

Hay
n(n + 1)
n(n + 1)
xn
.
2
2(2n + n)
4n2
Suy ra
1
lim xn = .
n
4
Bi 1.11 (H S phm H Ni 2). Bng quy np, d thy xn l mt dy s
dng. Ta c
1
1
ln xn+2 = ln xn+1 + ln xn n N .
2
2

t un = ln xn n N . Ta c:
u1 = ln x1 , u2 = ln x2 , un+2 =

un+1 + un
2

n N .

T cng thc xc nh dy un ta c
1
1
1
x2
un un1 = (un1 un2 ) = = ( )n2 (u2 u1 ) = ( )n2 ln .
2
2
2
x1
Do
1
x2
1
1
x2
un = ( )n2 ln + un1 = . . . = [( )n2 + ( )n3 + + 1] ln + ln x1
2
x1
2
2
x1

1. DY S

121
2 2 1
x2
= [ ( )n1 ] ln
+ ln x1 .
3 3 2
x1

Suy ra
lim un =

2 ln x2 + ln x1
.
3

Vy dy xn hi t v

lim xn = x13 x23 .

Bi 1.12 (C S phm Nam nh). D thy un > 0 vi mi n. Ta chng


minh un 2 vi mi n 3. Tht vy, d thy u3 = 2. Gi s uk 2 vi mi
k = 3, .., n, ta chng minh un+1 2.
Tht vy
!
1
2
> u2n 4 > 2
un+1 = un . 1 +
un1
!
1
un = u2n1 . 1 +
> u2n1 2un1
un2
vi mi n 4 Vy un > 2n3 .u3 do
lim un = +

n+

un+1
1
= un . 1 +
un
un1

!
> un .

T 2 iu trn dn n
un+1
= +
n+ un
lim

un
= 0.
n+ un+1

lim
T gi thuyt ta c:

un+1 un1 u2n = u2n .un1


Chia c 2 v cho un .un1 ta c
un+1
un

= un .
un
un1
p dng lin tip cng thc trn ta c:
un
un1

= un1
un1 un2

122
....
u3 u2

= u2 .
u2 u1
Cng v cc ng thc trn ta c
u2 + u3 + + un =

un+1 u2
.
un
u1

Suy ra
u1 + u2 + + un =

un+1
,
un

v v vy
lim

n+

(u1 + u2 + + un ) .un
un+1
1
= lim
= .
n+ 2un+1 + 1
2un+1 + 1
2

Bi 1.13 (C S phm Qung Ninh). Ta c xn =


n
X
k=1

xk =

1
1

, cho nn
n! (n + 3)!

1
1
1
1
1
1
+ +

.
1! 2! 3! (n + 1)! (n + 2)! (n + 3)!

Do
lim

n
X

5
xk = .
3
k=1

Bi 1.14 (H S phm Thi Nguyn). T gi thit ta c


an an1 =

n2
(an1 an2 ), n 3.
n

t vn = an an1 . Ta thu c dy {vn } tho mn


v2 = a2 a1 , vn =

n2
vn1 ,
n

vi mi n > 2. T ta thu c
1 2 3 4
n4 n3 n2
2(a2 a1 )
vn = . . . . . .
.
.
v2 =
.
3 4 5 6
n2 n1 n
n(n 1)
Vy
an an1 =

2(a2 a1 )
.
n(n 1)

1. DY S

123

Ta d dng thu c
an = an an1 + an1 an2 + + a3 a2 + a2
1
1
1
= a2 + 2(a2 a1 )(
+
+ +
)
3.2 4.3
n(n 1)
a2 a1
= 2a2 a1 2
.
n
Bi 1.15 (C S phm Vnh Phc). Xt hm s
f (x) =

1
arctan x
2014 2

l hm kh vi trn R v
|f 0 (x)| = |

1
1
| .
2
2(1 + x )
2

t g(x) = x f (x) khi g l hm kh vi trn R v


g 0 (x) = 1 +

1
> 0.
2(1 + x2 )

Hn na,

< 0, g(
)>0
2014
2014

cho nn tn ti duy nht l (0;


) sao cho g(l) = 0 hay f (l) = l.
2014
Mt khc, theo nh l Lagrange tn ti c R sao cho
g(0) =

1
|un+1 l| = |f (un ) f (l)| |un l|.
2
Do ,
1
|un l| ( )n1 |u1 l|.
2
Suy ra lim un = l.
Bi 1.16 (H Tn Tro). Cho dy s {an } vi
an+1

3
2

< an <

n1
n2
n
1
= 1 n+1 2n+1 + (1 + n )an a2n
2
2
2
2

Tnh
lim an .

5
2

v
n N.

124
Gii. Ta c
an+1 = 1 + an

n
1
n+1
n
(an n )2 + n+1 .
n
2
2
2
2

Suy ra
an+1

n
n+1
n
1
= 1 + an n (an n )2 .
n+1
2
2
2
2

t
un = an

n
,
2n

n N.

Suy ra un+1 = 1 + un 21 u2n v 1 < u1 < 2. Bng phng php quy np ta


chng minh c

|un 2| < 2n , n N.
Suy ra
lim un =

Hay
lim an =

2.

2.

Bi 1.17 (H Tn Tro). Cho dy s (un ) vi mi n = 1, 2, ...xc nh nh


sau:
(
u1 = 1
un+1 = 1 + u1 u2 . . . un vi mi n = 1, 2, . . . .
t Sn =

n
P
k=1

1
.
uk

Tm
lim Sn .

n+

Gii. Ta c
un+1 = 1 + u1 u2 . . . un

n 1

un = 1 + u1 u2 . . . un1

n 2,

v
suy ra:
un+1 1
= un
un 1
Do do

1
un+1 1

1
un (un 1)

n 2.

1
1
.
un 1 un

Hay
1
1
1
=

un
un 1 un+1 1

n 2.

2. HM S

125

Suy ra
n

X 1
X 1
1
1
1
1
1
1
+
=
+

=
+

.
Sn =
u1 k=1 uk
u1 k=1 uk 1 uk+1 1
u1 u2 1 un+1 1
M
u1 = 1, u2 = 1 + u1 = 2.
Nn
Sn = 2

1
un+1 1

Ta c
un+1 un = 1 + un (u1 u2 . . . un1 1) > 0.
Hay (un ) tng Nn n 2 ta c un+1 1 = u1 u2 . . . un > u1 (1 + u1 )n1 = 2n1 .
Suy ra
lim (un+1 1) = +.
n+

Suy ra
lim Sn = 2

n+

1
un+1 1

= 2.

HM S

Bi 2.1 (H Bch khoa H Ni). Gi s phn chng: hm s f (x) = sin(x2014 )


tun hon trn R. Tn ti T > 0 sao cho
f (x + T ) = f (x), x R.

2014
Phng trnh sin(x2014 ) = 0 c nghim dng xk =
k, k R. V


 2014
2014
lim (xk+1 xk ) = lim 2014
k+1
k =0
k+

k+

nn s khng im ca f (x) trong [0; T ] l v hn. T y dn ti mu thun.


Bi 2.2 (H Bch khoa H Ni). a) Vi > 0 cho trc, tn ti x0 > 0 sao
cho vi x x0 th

|f (x)| .
2
Tn ti s T > 0 sao cho g(x + T ) = g(x), x R. Ly x1 , x2 R, tn ti
n1 , n2 N sao cho x1 + n1 T > x2 + n2 T > x0 . Khi ta c
f (x1 + n1 T ) + g(x1 + n1 T ) > f (x2 + n2 T ) + g(x2 + n2 T )

126
Suy ra
g(x1 ) g(x2 ) > f (x2 + n2 T ) f (x1 + n1 T )
i vai tr x1 , x2 , ta suy ra |g(x1 ) g(x2 )| , vy g(x) l hng s.
b) Xt hm s f (x) = ex , ta c


1
x1
x
1 ex
f (x + 1) f (x) = e
e =
e
Vy mt hm s cn tm l
0 (x) =

e x
e .
1e

Vi l hm s tha mn bi th = 0 c tnh cht


(x + 1) = (x) x R.
p dng cu a) cho hm s = + 0 tng, tha man
lim 0 (x) = 0,

x+

suy ra l hng s.
Vy tt c cc hm cn tm c dng =

e x
e + C.
1e

Bi 2.3 (H Cng ngh thc phm Tp. HCM). Ta c


 
x


2
x ln f 2x
lim f
= lim e
.
x+
x+
x
p dng quy tc LHospital, ta thu c



2
ln f (2 t)
2f 0 (2 t)

lim x ln f
= lim+
= lim+
x+
t0
t0 2 tf (2 t)
t
x

2f 00 (2 t)

= 2.
= lim+
t0 2f (2 t) + 4 tf 0 (2 t)
Vy

 
x
2
lim f
= e2 .
x+
x

Bi 2.4 (H Hng Vng, Ph Th). 1) Ta c


xf (x)
0 (x) =

Rx

Rx
f (t)dt f (x) tf (t)dt

Rx
( f (t)dt)2
0

Rx
f (x) (x t)f (t)dt
=

0
Rx

> 0,
f (t)dt)2

x > 0

2. HM S

127

Vy (x) tng trn khong (0, +).


Theo Lpitan, ta c
lim+ (x) = lim+

x0

x0

xf (x)
= 0.
f (x)

2) Xt hm g(x) xc nh trn [0; +) tha mn g(x) = (x) nu x > 0;


g(0) = 0. D thy g(x) lin tc trn [0; 2015] nn g(x) lin tc u trn
[0; 2015].
Suy ra g(x) lin tc u trn (0; 2015). Vy (x) lin tc u trn (0; 2015).
Do ,
lim

n+

2014n + i 1 
2014n + i 

= 0 i 2015.
n
n

Vy
lim

n+

2015 
X

i=1

2014n + i 
2014n + i 1 

= 0.
n
n

Bi 2.5 (H Qung Nam). Gi s tn ti a, b R, a 6= b sao cho f (a) 6= f (b).


Theo gi thit ta c g(a) = g(b) = c.
Ln lt cho y = a, y = b ta c
(f (x) f (a))(g(x) c)) = 0,

x R

(4)

(f (x) f (b))(g(x) c)) = 0,

x R

(5)

T (4) v (5) suy ra


(f (b) f (a))(g(x) c) = 0,
Vy g(x) = c,

x R.

x R.

Bi 2.6 (C S phm Nam nh).


f 0 (x) = ln(1 + x) +
f 00 (x) =

x
1+x

1
1
+
> 0 vi mi x > 0
1 + x (x + 1)2

(1)

p dng nh l Lagrange vi hm f (x) trn khong (0, x) ta c tn ti c(x)


(0, x) sao cho:
f 0 (c(x)) =

f (x) f (0) x ln(1 + x)


=
= ln(1 + x)
x0
x

128
S duy nht: Gi s tn ti d(x) (0, x), d(x) 6= c(x) m
f 0 (d(x)) = ln(1 + x) = f 0 (c(x)).
Do theo nh l Rolle vi hm f 0 (x) tn ti a gia c(x), d(x) sao cho
f 00 (a) = 0.
iu ny mu thun vi (1).
Bi 2.7 (C S phm Qung Ninh). S dng quy tc LHpital ta c

R x2012
1006
tdt
tan
2012x2011 tan(x2 )
1006
.
lim+ 0
=
lim
=
2014
2013
x0+
x0
x
2014x
1007
Bi 2.8 (C S phm Qung Ninh). V f (1) = 2 cho nn lim f (x) = 2 v
x1+

ax2 x nhn x = 1 l nghim cho nn a = 1; b = 1.


Bi 2.9 (H S phm Thi Nguyn). Trc ht ta chng minh tn ti a, b
(0, 1] sao cho
|f (b) f (a)| > |b a|2014 .
Tht vy gi s ngc li |f (b) f (a)| |b a|2014 vi mi a, b (0, 1]. Khi
ta c
f (b) f (a)


|b a|2013 vi mi a 6= b (0, 1].

ba
Cho b a, ta c f 0 (a) = 0, vi mi a (0, 1]. Do f l hm hng trn
(0, 1]. V f lin tc nn f l hm hng trn [0, 1]. Mu thun vi gi thit f
khc hng. Tip theo ta c th gi s |f (1) f (0)| 0. Xt hm
g(t) = |f ((1 t)a + t) f ((1 t)b| |((1 t)a + t) (1 t)b|2014 ,
khi g lin tc trn [0, 1] v g(0)g(1) < 0. Theo nh l gi tr trung gian
tn ti t0 (0, 1) sao cho g(t0 ) = 0. Nh vy ta c
|f ((1 t0 )a + t0 ) f ((1 t0 )b| = |((1 t0 )a + t0 ) (1 t0 )b|2014 ,
t x1 = (1 t0 )a + t0 , x2 = (1 t0 )b ta c iu phi chng minh.
Bi 2.10 (H Tn Tro). Cho f (x) l hm s lin tc v xc nh trn khong
[0, 1] tha mn:
0 f (x) 1,
Cho
Z
c=

1
x [0; 1] vf (0) = f (1) = .
2

Z
f (x)dx, F (x) =

f (t)dt.
0

2. HM S

129

Chng minh rng:


a.

(
F (x) x
F (x) c

b.

khi 0 x c
khi c x 1.

F (x)dx < c
0

c2
.
2

Gii. a). Ta c: 0 < f (x) 1 v f (1) = 21 . Suy ra


1

1dx = 1.

f (x)dx <

0<

Do do
c < 1.
Ta c:
Z
F (x) =

f (t)dt
0

vi F (0) = 0, F (1) = c, F (x) = f (x) x [0, 1]. Suy ra F (x) ng bin trn
[0, 1]. Hay
F (x) F (1) = c.
(6)
Mt khc, t gi thit:
0 f (x) 1 x [0, 1]
hay
0 f (t) 1 t [0, x].
Suy ra
Z

Z
f (t)dt <

F (x) =

(7)

1dt = x.
0

T (6) v (7) suy ra:


F (x) min{x, c}.
Ta suy ra
(
F (x) x
F (x) c
b). Theo chng minh a). Ta c:
Z 1
Z c
Z
F (x)dx =
F (x)dx +
0

khi 0 x c
khi c x 1.

Z
F (x)dx

cdx = c

xdx +
c

c2
.
2

130
Du ng thc xy ra khi:
(
F (x) = x
F (x) = c

khi 0 x c
khi c x 1.

Hay
( 0
F (c ) = 1
0
F (c+ ) = 0.
iu ny mu thun vi gi thit:
0

F (x) = f (x)

x [0, 1].

Vy du ng thc khng xy ra. Vy


Z

F (x)dx < c
0

c2
.
2

PHP TNH VI PHN

Bi 3.1 (HV An ninh nhn dn). Xt phng trnh tm n l hm y(x) vi


x [0, 1] v k (0, 1) sau
y 2 y + kx(1 x) = 0.

Ta thy = 4kx2 4kx + 1 = (2 kx 1)2 + 4 kx(1 k) 0.


Nn c cc nghim

1
1
y1 (x) = (1 4kx2 4kx + 1), y2 (x) = (1 + 4kx2 4kx + 1).
2
2
Ta thy y1 (x) v y2 (x) l cc hm lin tc, ng thi

1
1
y1 (0) = y1 (1) = 0, y1 ( ) = (1 1 k)
2
2

1
1
y2 (0) = y2 (1) = 1, y2 ( ) = (1 + 1 k)
2
2
iu kin ca bi dn ti mt trong hai kh nng
a.

1
1
f ( ) (1 1 k)
2
2

3. PHP TNH VI PHN

131

b.

1
1
f ( ) (1 + 1 k).
2
2
Vi kh nng a) ta xt hm g(x) = f (x) y1 (x) lin tc trn [0, 1] v c

1
1
1
1
1
g( ) = f ( ) y1 ( ) = f ( ) (1 1 k) 0
2
2
2
2
2
g(1) = f (1) y1 (1) = 1 0 = 1 > 0.
Nn phng trnh g(x) = 0 c nghim c [ 12 , 1), tc l f (c) = y1 (c). S dng
nh l Lagrang cho hm f (x) trn cc on [0, c] v [c, 1] th tn ti cc im
x1 (0, c), x2 (c, 1), x1 6= x2 sao cho
f (x1 ) =

f (c) f (0)
y1 (c)
f (1) f (c)
1 y1 (c)
=
, f (x2 ) =
=
.
c0
c
1c
1c

Khi
f (x1 )f (x2 ) =

y1 (c) 1 y1 (c)
= k.
c
1c

Vi kh nng b) ta xt hm h(x) = f (x) y1 (x) lin tc trn [0, 1] v c

1
1
1
1
1
h( ) = f ( ) y2 ( ) = f ( ) (1 + 1 k) 0
2
2
2
2
2
h(0) = f (0) y2 (0) = 0 1 = 1 < 0.
Nn Nn phng trnh h(x) = 0 c nghim d (0, 12 ], tc l f (d) = y2 (d). S
dng nh l Lagrang cho hm f (x) trn cc on [0, d] v [d, 1] th tn ti
cc im x1 (0, d), x2 (d, 1), x1 6= x2 sao cho
f (x1 ) =

f (d) f (0)
y2 (d)
f (1) f (d)
1 y2 (d)
=
, f (x2 ) =
=
.
d0
d
1d
1d

Khi
f (x1 )f (x2 ) =

y2 (d) 1 y2 (d)
= k.
d
1d

Bi 3.2 (H Bch khoa H Ni). Xt hm s




x
g(x) = f
, x [0; 1).
1x
Do lim g(x) = lim f (x) = f (0) = g(0) nn hm g(x) thc trin lin tc ln
x1

x+

[0; 1] v g(0) = g(1). p dng nh l Rolle cho hm g, ta suy ra iu phi


chng minh.

132
0

Bi 3.3 (HV Bu chnh Vin thng). Theo nh ngha ca lim f (x) = 0, ta


n+

c
0

 > 0, xn0 x xn0 | lim f (x)| <


n+


2

(1)

Vi mi dy (xn ) sao cho lim xn = +, p dng nh l Lagrange cho hm


n+

f (x) trn on [xn0 , xn ], tn ti n (xn0 , xn ) sao cho


|

f (xn ) f (xn0 )
| = |f (n )|
xn xn0

(2)

Kt hp (1) v (2), ta nhn c


|

f (xn ) f (xn0 )

| = |f (n )| .
xn xn0
2

Khi
f (xn0 ) 
f (xn0 )
f (xn )
f (xn0 ) 
xn
(1
)<
<
+ (1 0 ).
xn
2
xn
xn
xn
2
xn
Vi n ln, ta c

f (xn0 )

<
< .
2
xn
2
M 2 (1

x n0
)
xn

< 2 . Khi , ta c

 <

f (xn )
f (xn )
f (x)
<  lim
= 0 lim
=0
n+ xn
n+ x
xn

Bi 3.4 (H Cng ngh thc phm Tp. HCM).


Bi 3.5 (H Cng ngh thc phm Tp. HCM). t h(x) = f 2 (x) + (f 0 (x))2 .
Khi , h(x) l hm kh vi trn [1, 1], p dng nh l Lagrange trn cc
on [1, 0], [0, 1], tn ti a (1, 0), b (0, 1) sao cho
f 0 (a) =

f (0) f (1)
f (1) f (0)
v f 0 (b) =
.
2
2

Suy ra
|f 0 (a)| =

|f (0)| + |f (1)|
1+1

= 1 dn ti h(a) 1 + 1 = 2,
2
2

v
|f 0 (b)| =

|f (2)| + |f (0)|
1+1

= 1 dn ti h(b) 1 + 1 = 2.
2
2

3. PHP TNH VI PHN

133

M h(0) = f 2 (0) + f 02 (0) = 3 > 2. Do h(x) khng t gi tr ln nht ti


a, b. V h(x) kh vi trn [a, b] nn tn ti c [a, b] sao cho h(c) = max h(x).
[a,b]

Khi , h0 (c) = 0. R rng c (a, b).


Ta c h(c) h(0) = 3, f 2 (c) 1 suy ra (f 0 (c))2 2, dn ti f 0 (c) 6= 0.
Mt khc, h0 (c) = 2f 0 (c) (f (c) + f 00 (c)). Suy ra f (c) + f 00 (c) = 0.
Bi 3.6 (H Hng Vng, Ph Th). Xt hm s
g(x) = x2014 .ef (x) .
Do f (x) l hm lin tc trn [0; 1], kh vi trn (0; 1) nn hm g(x) cng lin
tc trn [0; 1], kh vi trn (0; 1) v g(0) = 0, g(1) = ef (1) = 1
g 0 (x) = x2013 ef (x) (2014 + xf 0 (x)).
p dng nh l Lagrange ta c x0 (0; 1) sao cho
g 0 (x0 ) = g(1) g(0) = 1.
x2013
ef (x0 ) (2014 + x0 f 0 (x0 )) = 1.
0
x0 f 0 (x0 ) + 2014 =

1
x2013
ef (x0 )
0

Bi 3.7 (H Hng Vng, Ph Th). Ta c : x (0; 2) th:


(
f (x) = f (0) + f 0 (1 ).x , 1 (0; x)
f (x) = f (2) + f 0 (2 ).(x 2) , 2 (x; 2)
Suy ra
(
f (x) 1 2x, x (0; 2)
f (x) 2x 1
T ta c

Z1

Z1
f (x)dx

(1 2x)dx = 0

Z2

Z2
f (x)dx

Z2

Z1
f (x)dx =

(2x 1)dx = 2
Z2
f (x)dx 2.

f (x)dx +
0

134
Mt khc

R2

f (x)dx = 2 khi v ch khi :

(
f (x) = 1 2x x [0; 1]
f (x) = 2x 1 x [1; 2]
iu ny mu thun vi tnh kh vi lin tc ca f .
Vy
Z2
f (x)dx > 2.
0

3
Bi 3.8 (H Ngoi thng H Ni). Gi c [0, 1] sao cho f (c) = min f (x) = .
x[0;1]
4
0
D thy c (0, 1) v f (c) = 0.
Khai trin Taylor ca f (x) trong ln cn im c ta c
f (x) = f (c) + f 0 (c)(x c) +

f 00 [c + (x c)]
(x c)2 ; 0 < < 1
2

f 00 [c + 1 (c)]
Ti x = 0, ta c f (0) = f (c) + f 0 (c)(c) +
(c)2 ; 0 < 1 < 1 suy
2
1007
ra f 00 [c + 1 (c)] = 2 .
2c
f 00 [c + 2 (1 c)]
Ti x = 1, f (1) = f (c) + f 0 (c)(1 c) +
(1 c)2 ; 0 < 2 < 1 suy
2
1007
ra f 00 [c + 2 (1 c)] =
.
2(1 c)2
Suy ra
10072
f 00 [c + 1 (c)].f 00 [c + 2 (1 c)] =
.
4c2 (1 c)2

2
c+1c
1
M c(1 c)
= nn
2
4
f 00 [c + 1 (c)].f 00 [c + 2 (1 c)] =

10072
10072
2

 = 1007 .4
1 2
4c2 (1 c)2
4 4

Mt khc, do c + 1 (c), c + 2 (1 c) [0; 1] nn


2
1007 .4 f [c + 1 (c)].f [c + 2 (1 c)] max f (x) .
2

00

00

00

x[0;1]

Do max f 00 (x) 2014.


x[0;1]

3. PHP TNH VI PHN

135

a+b
lin tc trn [a; b]
2
v kh vi trn (a; b). Hn na, g(a).g(b) < 0 do tn ti x0 sao cho g(x0 ) = 0
a+b
hay f (x0 ) =
x0 .
2
p dng nh l Lagrange cho hm f trn cc khong (a; x0 ) v (x0 ; b) th tn
ti c1 (a; x0 ) v c2 (x0 ; b) sao cho
Bi 3.9 (C Ng Gia T). Xt hm g(x) = f (x) + x

b x0 0
f (b) f (x0 )
x0 a
f (x0 ) f (a)
=
; f (c2 ) =
=
.
x0 a
x0 a
b x0
b x0

f 0 (c1 ) =

Hn na, f l hm lin tc trn [a; b] v kh vi trn (a; b) cho nn theo nh


l Lagrange tn ti c3 (a; b) sao cho
f 0 (c3 ) =

f (b) f (a)
= 1.
ba

Vy f 0 (c1 ).f 0 (c2 ).f 0 (c3 ) = 1.


Bi 3.10 (H Nng nghip). Nu mi mt trong cc hm f (x), f 0 (x), f 00 (x), f 000 (x)
i du, chng hn f 00 (x) i du th do n lin tc nn tn ti a f 00 (a) = 0,
tc ta c du ng thc.
Gi s c 4 hm trn u khng i du. Ta chng minh f (x) v f 00 (x) cng
du. Khai trin Taylor f (x) ti 0 n cp 2 ta c:
f (x) = f (0) + f 0 (0)x +

f 00 (x) 2
x , 0 < < 1 (2 im)
2

T nu f 00 (x) > 0 th f (x) > f (0) + f 0 (0)x.


Nu f 0 (0) > 0 th f (0) + f 0 (0)x > 0 vi x ln.
Nu f 0 (0) < 0 th f (0) + f 0 (0)x > 0 vi x nh.
T nu f 00 (x) > 0 th f (x) > 0 (2 im).
Tng t nu f 00 (x) < 0 th f (x) < 0, tc f (x) v f 00 (x) cng du. Tng
t f 0 (x) v f 000 (x) cng du. Do vy tn ti a f (a)f 0 (a)f 00 (a)f 000 (a) 0 (1
im).
Bi 3.11 (HV Phng khng Khng qun). t g(x) =
0

1
f (x)

, ta c:

00

f (x) 00
2[f (x)]2 f (x)f (x)
g (x) = 2 ; g (x) =
.
f (x)
f 3 (x)
0

Do f (x) > 0; f (x) > 0, nn


0

g(x) > 0; g (x) < 0.

(1)

136
Xt
00

00

00

f 4 (x)
g (x)g(x)
2[f (x)]2 f (x)f (x) 1
f (x)f (x)
.
.
=
=
2

0
0
0
[g (x)]2
f 3 (x)
f (x) [f (x)]2
[f 0 (x)]2
00

(Do

f (x)f (x)
[f 0 (x)]2

2, theo gi thit). T cc kt qu trn suy ra


00

g (x) 0, x 0

(2)

T (1) v (2) suy ra hm g (x) n iu khng gim v b chn trn bi 0,


nn tn ti gii hn
0
lim g (x) = c 0
x

Gi s c < 0. Do

lim g (x) = c 0

nn

g (x) c, x 0.
Suy ra
Z

g (t)dt
0

cdt g(x) g(0) cx.


0

Hay
g(x) cx + g(0), x 0.
iu ny khng th xy ra v g(x) 0, x 0. (V khi x ln th cx + g(0) <
0

f (x)
2
x+ [f (x)]

0). Vy c = 0 khi v ch khi lim g (x) = 0 khi v ch khi lim


x

= 0.

Bi 3.12 (H Qung Bnh). Gi s


P (x) = an xn + an1 xn1 + + a0

(an 6= 0).

V P (x) 0 vi mi x R nn n chn v an > 0.


Xt hm
F (x) = P (x) + P 0 (x) + + P (n) (x).
V F cng l a thc bc n vi h s ca xn l an nn lim F (x) = +.
x

Do tn ti xo R sao cho
F (xo ) = min F (x).
xR

Theo nh l Fermat
F 0 (xo ) = F (xo ) P (xo ) = 0.
Nh vy F (xo ) = min F (x) = P (xo ) 0, v F (x) 0, x R.
xR

3. PHP TNH VI PHN

137

Bi 3.13 (H Qung Nam). Xt tch phn


Z 2
f (x)(1 + cos x)dx = (a0 + 1).
0

Do tn ti x1 [0; 2] sao cho f (x1 ) > 0.


Xt tch phn
Z
2

f (x)(1 cos x)dx = (a0 1).


0

Do tn ti x2 [0; 2] sao cho f (x2 ) < 0.


T y ta c iu phi chng minh.
Bi 3.14 (H Qung Nam). Xt hm s
1
(x) =
2

Z

2
f (t)dt 0,

x R.

Ta c
0 (x) = g(x),

x R.

Vi x > 0 v y < 0, theo nh l Lagrange,


0

(x)
(x) (0)
(y)
=
= g(x ) g(0) = 0 g(y ) =
0.
x
x
y

Do (x) = 0 vi mi x R, dn n f (x) = 0 vi mi x R.
Bi 3.15 (H Quy Nhn). t g(x) = f (x) sin x, x [0; /2]. Suy ra g(x)
/2
R
lin tc trn [0; /2] tho mn
g(x)dx = 0. Do x ra hai trng hp.
0

Nu g(x) l hm ng nht khng trn [0; /2] th f (x) = sin x. S dng bt


ng thc
x3
x
sin x x, x [0; /2]
6
ta suy ra iu phi chng minh.
Gi s g khng ng nht bng khng trn [0; /2]. V g lin tc nn va
nhn gi tr m, va nhn gi tr dng trn [0; /2]. Theo nh l gi tr
trung gian, tn ti x0 (0; /2) sao cho g(x0 ) = 0. Do f (x0 ) = sin x0 v ta
cng c iu phi chng minh.
Bi 3.16 (H Sao ). T |f (x)|

1
ex2 1

x R , suy ra

lim f (x) = 0.

138
Do
lim (f (x)

e x2

1
) = 0.
1

Xt hm
g(x) = f (x)

1
ex2 1

Hm g kh vi trn R v limx g(x) = 0.


Do , hm g phi t cc tr a phng ti 1 im no . Tht vy, gi s
tn ti x0 sao cho
g(x0 ) > 0.
V limx = 0 < g(x0 ) nn tn ti b nh
g(x) < g(x0 )x < a.
V limx+ = 0 < g(x0 ) nn tn ti a ln g(x) < g(x0 ) x > b.
Do hm lin tc trn on [a, b] nn tn ti c [a, b]
g(c) = max g(x) = max g(x).
x

[a,b]

Khi g t cc tr a phng ti c. Theo nh l Fermat g (c) = 0 hay


0

f (c) =

2c
ec2 1

Tng t cho trng hp tn ti x0 sao cho g(x0 ) < 0.


Bi 3.17 (H Sao ). Xt hm
Z x
00
g(x) =
(f (t) + f (t)) sin tdt.
0

Theo gi thit, hm g lin tc trn on [0, ], kh vi trn (0, ). Ta c g(0) = 0


v
Z
Z
00
g() =
f (t)dt +
f (t) sin tdt
0
Z0
Z
0
0

= f (t) sin t|0


f (t) cos tdt +
f (t) sin tdt
0
0
Z
Z
= 0 f (t) cos t|0
f (t) sin tdt +
f (t) sin tdt = 0.
0

0
0

p dng nh l Rolle cho hm g, tn ti c (0, ) : g (c) = 0 hay


00

(f (c) + f (c)) sin c = 0


hay

00

f (c) = f (c).

3. PHP TNH VI PHN

139

Bi 3.18 (H Sao ). Ta c
Z
Z 1
00
0
1
f (t)(1 t)dt = (1 t)f (t)|0 +

f (t)dt

0
0

= f (0) + f (1) f (0) = f (0) = 2014.


p dng bt ng thc Bunhiacopski ta c
Z 1
 Z 1 00
 21  Z 1
 21
00
2
2
f (t)(1 t)dt|
(f (t)) dt
(1 t) dt
2014 = |
0
0
0
Z
 12
1  1 00
(f (t))2 dt .

3 0
Suy ra
Z

00

(f (x))2 dx 3(2014)2 .

Du bng xy ra khi v ch khi f (x) = 1007x(x 1)(x 2).


Bi 3.19 h(H
i S phm
h i H Ni 2). Vi  > 0 cho trc bt k, t n =
1
2014(1 +  ) , vi 1 l phn nguyn ca 1 . Ta c
2014
1
=
n
1+

1



< min{, 1}.
1+

Xt hm s

2014  n .x
F (x) = f (x) x
.e 2014 ,
n

h 2014 i
x 0,
[0, 1].
n

h
i
Do hm f lin tc trn [0, 1] kh vi trn (0, 1) nn hm F lin tc trn 0, 2014
n
v kh vi trn (0, 2014
). Mt khc, ta c F (0) = F ( 2014
) = 0 nn theo nh l
n
n


Rolle, tn ti x 0, 2014
(0, ) sao cho: F (x ) = 0. Suy ra
n
f (x )(x

n .x
2014 n .x
2014 n
).e 2014 + f (x )[(x
).
+ 1].e 2014 = 0.
n
n 2014

Hay
f (x ) =
vi x (0, 2014
) (0, ).
n

n2 x f (x )
2014(2014 n x )

140
Bi 3.20 (C S phm Nam nh). t
Z1
(1 2x)f (x)dx.

I=
0


t

u = f (x)

dv = (1 2x)dx

1
I = (x x2 )f (x) 0

du = f 0 (x)dx
v = x x2
Z1

(x x2 )f 0 (x)dx =

Z1

(x2 x)f 0 (x)dx

Vy
Z1

Z1

(f (x) + (1 2x)f (x))dx = 5


0

(x2 x + 1)f 0 (x)dx = 5.

p dng nh l gi tr trung bnh th 2 ca tch phn vi hm f 0 (x) v hm


g(x) = x2 x + 1. Hm g(x) = x2 x + 1 > 0 vi mi x. Ta c tn ti c (0, 1)
sao cho:
!
Z1
2
3
1 5
x
x
+ x 0 = f 0 (c).
5 = f 0 (c). (x2 x + 1)dx =
3
2
6
0

Suy ra
f 0 (c) = 6.
Bi 3.21 (C S phm Nam nh). Xt hm
2

g(x) = sin f (x).ex .


Ta c

g 0 (x) = cos f (x).f 0 (x).ex 2x. sin f (x)ex


2

= ex .(cos f (x).f 0 (x) 2x sin f (x)),


v g(0) = g(1) = 0. Nh vy hm g(x) tha mn cc iu kin ca nh l
Rolle. Do tn ti c (0, 1) sao cho g 0 (c) = 0, suy ra
cos f (c).f 0 (c) 2c sin f (c) = 0.
V f (c) [1, 1] nn cos f (c) > 0. Vy
f 0 (c) = 2c tan f (c).

3. PHP TNH VI PHN

141
Z

f (t)dt
2013
x2014

Bi 3.22 (C S phm Qung Ninh). Xt hm s g(x) =


Z c
Khi s dng nh l Rolle ta c c 2014
f (x)dx = cf (c).
2013

Bi 3.23 (H S phm Thi Nguyn). t


x

f (t)dt.

F (x) =
0

Theo gi thit ta c
Z

xf (x)dx =
0

Z
=

1 Z

xdF (x) = xF (x)

Z
f (x)dx

F (x)dx =

F (x)dx
0

f (x)dx
0

Vy
Z

F (x)dx = 0.
0

Xt hm
g(x) = e

2014x

F (t)dt.
0

Do g kh vi lin tc trn [0, 1]. Ta c g(0) = g(1) = 0, nn theo nh l


Rolle, tn ti x0 (0, 1) sao cho g 0 (x0 ) = 0. Ta c
0

g (x) = e

2014x

Z
(F (x) 2014

F (t)dt).
0

Do
Z

x0

F (x0 ) = 2014

F (x)dx.
0

Xt hm
Z
h(x) = F (x) 2014

F (t)dt,
0

ta c h(0) = h(x0 ) = 0, h tha mn nh l Rolle, do tn ti c (0, x0 ) sao


cho h0 (c) = 0. Do
Z c
f (c) = 2014
f (x)dx.
0

142
Bi 3.24 (C S phm Vnh Phc). t
2014x
Z x
1

f (t)dt.
g(x) = e 2013 2013
x2012 a
Khi g l hm lin tc trn [a; b] v kh vi trn (a; b). Hn na,
g(a) = g(b) = 0.
Do , p dng nh l Rolle cho hm g trn [a; b] th tn ti c sao cho
g 0 (c) = 0. Hay ta c c
Z c
Z c
f (x)dx = 0.
f (x)dx 2013cf (c) + 2012
2014c
a

Bi 3.25 (C S phm Vnh Phc). T gi thit ta c


xf 2014 (x) 2014f (x) = 2013x x(f 2014 (x) + 2013) = 2014f (x).
hay
x=

2014f (x)
.
+ 2013

f 2014 (x)

2014f (x)
suy ra g l hm lin tc trn [1; 1].
+ 2013
Xt hm s h(x) = g(x) x khi h(1).h(1) 0 nn tn ti x0 [1; 1]
sao cho h(x0 ) = 0 hay g(x0 ) = x0 .
t g(x) =

f 2014 (x)

Bi 3.26 (H Tn Tro). Ta c g(x) = x (x) l hm s lin tc trn on


[1, 2] v (x) c o hm trn on [1, 2]. Theo nh l gi tr trung bnh ,tn
ti s c (1, 2) sao cho
Z 2
0
g(x)dx = g(c)(2 1) = c (c).
1

Mt khc
Z
1

2 Z 2

g(x)dx =
xd(x)x = x(x)
(x)dx
1
1
1
Z 2
= 2(2) (1)
(x)dx = 1
Z

1
0

Suy ra c (c) = 1. Xt hm s
h(x) = x0 (x) 1.
Hm h(x) lin tc trn on [1, 2] v kh vi trn khong (1, 2). Ta c
00

h(1) = h(c) = 0, h0 (x) = x (x) + 0 (x).


heo nh l Rolle, tn ti s thc a (1, c) (1, 2) sao cho h0 (a) = 0. Vy
00
phng trnh x (x) + x0 (x) = 0 lun c nghim trong khong (1, 2).

4. PHP TNH TCH PHN

143

PHP TNH TCH PHN

Bi 4.1 (HV An ninh nhn dn). Ta thy


Z 1
Z 1
Z
0=
xf (x)dx = a
xf (x)dx =
0

axf (x)dx

, a R.

Khi , k hiu M = max |f (x)| th a R ta c


x[0,1]

Z 1
Z 1
Z 1




2
2
x f (x)dx =
x f (x)dx
axf (x)dx =
(x ax)f (x)dx
0
0
Z 10
Z 1

|(x2 ax)||f (x)|dx


|(x2 ax)| max |f (x)|dx
x[0,1]
0
0
Z 1
Z 1
2
M
|(x ax)|dx = M I(a) , vi I(a) =
|(x2 ax)|dx
2

Suy ra
Z
|

x2 f (x)dx| M min I(a) M min I(a).


aR

a[0,1]

Mt khc, vi a [0, 1] ta c
Z 1
Z a
Z 1
2
|x(x a)|dx
|x(x a)|dx +
|(x ax)|dx =
I(a) =
a
0
0
Z 1
Z a
2
(x2 ax)dx
(ax x )dx +
=
0

ax2 x3 a
x3 ax2 1 1 3
=(
)|0 + (
)| = (2a 3a + 2).
2
3
3
2 a 6
Bng cch kho st hm s g(a) = 2a3 3a + 2 trn on a [0, 1] ta c

1
min g(a) = g( ) = 2 2
[0,1]
2
v

1
2 2
min I(a) = I( ) =
.
[0,1]
6
2

Vy ta c bt ng thc cn chng minh.


Bi 4.2 (H Bch khoa H Ni). Ta c
Z 3
Z
Z 3
2
2
2
dx
dx
dx
=
+
= I1 + I2 .
I=

2 + cos x
2
+
cos
x
0
0 2 + cos x
2

144

3
x
.
i bin t = tan trong I1 , ta c kt qu I1 =
2
9
x
4 3
i bin t = cot trong I2 , ta c kt qu I2 =
.
2
9

5 3
Vy I =
.
9
Bi 4.3 (HV Bu chnh Vin thng). p dng bt ng thc Cauchy cho tch
phn, ta c
Z

Z
f (x)dx.

Mt khc
sZ

Z
f (x)dx

Xt hm g(t) =

t
M

m
t


1
dx
f (x)

sZ

2
p
1
dx = 1.
f (x). p
f (x)

r
Z 1
f (x)
m
m
dx.
dx.
M
M
0
0 f (x)
r Z 1
Z 1

1
m
f (x)
m
.
dx +
dx
2 M
M
0 f (x)
r Z 1 0
f (x)
m 
1 m
+
dx.
=
2 M 0
M
f (x)

1
dx =
f (x)

trn on [m, M ]. Ta c
0

g (t) =

1
m
2.
M
t

Hm s t gi tr ln nht bng gLN = 1 +


Nh vy, bt ng thc c chng minh.

m
M

khi v ch khi t =

mM .

Bi 4.4 (HV Bu chnh Vin thng). Do f (x) lin tc trn [0; 1], nn tn ti
cc s x1 , x2 [0; 1] sao cho
f (x1 ) = min f (x) : x [0; 1], f (x2 ) = max f (x) : x [0; 1].
Vi g(x) > 0 ta c
g(x)f (x1 ) g(x)f (x) g(x)f (x2 )
v do
Z

Z
g(x)f (x1 )dx

Z
g(x)f (x)dx

g(x)f (x2 )dx.


0

4. PHP TNH TCH PHN

145

Khi
1
f (x1 )
m

f (x)g(x)dx f (x2 ).
0

Do f (x) lin tc trn [0; 1], nn tn ti s c [0; 1] sao cho


1
m

f (x)g(x)dx = f (c).
0

Bi 4.5 (HV Bu chnh Vin thng). T


2

ex ex
ex ex
=
x = x
x
x2
suy ra
Z +  Z
I=
0

tx2

Z

dt dx =

Z

tx2

xe

etx dt.

1
dx dt =
2


1
ln tdt = ln .
2

Bi 4.6 (H Cng ngh thc phm Tp. HCM). V hm s f (x) lin tc trn
[0, 1] nn tn ti
M = max (|f (x)| + 1) < +
x[0,1]

Khi , vi > 0, ta chn sao cho 0 < < min

1

Z
Z1


xn f (x)dx
xn |f (x)| dx M



(1)

1

Z
Z1


(1 )n+1
n
xn f (x)dx
x
|f
(x)|
dx

M
.


n+1

(2)

n o
, 1 . Ta c
M

T (1) v (2), ta suy ra


1

Z



(1 )n+1
n

0 x f (x)dx + M
.
n+1


0

Trong (3), cho n , ta c


1

Z



n
0 lim x f (x)dx .
n


0

(3)

146
V > 0 l ty nn dn n
Z1
lim

xn f (x)dx = 0.

sin x
sin x
l hm gim trn [0, ] v lim+
= 1. Vi x
x0
x
x
dng nh, xt t [x, 2013x]. Ta c

Bi 4.7 (D b). Hm

Z
 sin 2013x 2012 2013x
1
2013x

2013x
Z

dt

 sin x 2012
(sin t)2012
dt

t2013
x

2013x
Z

1
dt.
t

Cho x 0+ ta c
2013x
Z

lim+

x0

(sin t)2012
dt = ln 2013.
t2013

Rx2

Bi 4.8 (D b). Xt hm s F (x) =

R1

f (t)dt +

f (t)dt. Ta c F (0) = 0,

1x2

R1
R1
R1
F (1) = 2 f (t)dt. V F (x) lin tc v 0 f (t)dt 2 f (t)dt nn tn
0

ti x() [0, 1] sao cho F (x()) =

R1

f (t)dt. Mt khc, F 0 (x) = 2x(f (x2 ) +

f (1 x2 )) 0. Do , hm F (x) l n iu tng, dn n x() l duy nht.


p dng nh l gi tr trung bnh ta c
Z1

f (t)dt = x2 ()(f (1 ()) + f (2 ())),

vi 0 xi1 () x2 (), 1 x2 () xi1 () 1. Vy,


R1

R1

f (t)dt

f (t)dt
x2 ()
0
0
=

f (1 ()) + f (2 ()) 0 f (0) + f (1)


Bi 4.9 (H Hng Vng, Ph Th). Ta c :
Z+
Z0
Z+
1
1
1
f x dx =
f x dx +
f x dx
x
x
x

= I1 + I2

4. PHP TNH TCH PHN

147

1
Thay t = x , ta c:
x
dx =


1
t
1+
dt vi x > 0.
2
t2 + 4

dx =


1
t
1
dt vi x < 0.
2
t2 + 4
Z+

t
dt
f (t) 1
t2 + 4

1
I1 =
2

1
I2 =
2

Z+

t
f (t) 1 +
dt.
t2 + 4

Do

+
R

f (x)dx hi t nn

Z+
x
f (x).
dx
2
x +4

hi t ( do tiu chun Aben)


Z+
Z+
1
f (t)dt = J.

f x dx =
x

Bi 4.10 (H Ngoi thng H Ni). t


Z1
u=

Z1
f (x)dx v F (x) =

xf (x)dx, v =
0

Zx

f (t)dt.
0

Do tnh cht lm, ta c


f (t)

t
xt
t
xt
f (x) +
f (0) = f (x) +
.
x
x
x
x

Suy ra
Zx 
F (x)
0

t
xt
f (x) +
x
x

x
f (x)t2
t2
xf (x) x
dt =
+t =
+ .
2x
2x 0
2
2

148
Do ,
Z1
u=

Z1
xf (x)dx =

xdF (x) = xF (x)|10

Z1
=v
=v

F (x)dx
0

Z1

F (x)dx v
0

Z1


xf (x) x
+
dx
2
2

u 1

2 4

3u
1
v v2.
2
4
Vi f (x) = x + 1 tha mn bi v bt ng thc xy ra du bng.
Hay

Bi 4.11 (H Ngoi thng H Ni). a) Xt hm s A : [0, a] R xc nh


bi
f (x)
Zx
Z
A(x) = f (t)dt +
f 1 (t)dt xf (x).
0

Hm A tha mn cc tnh cht


+ A thuc lp C 1 trn [0; a];
+ Vi mi x [0; a], A0 (x) = f (x) + f 1 (f (x)) f 0 (x) (f (x) + xf 0 (x)) = 0.
Do A khng i, hn na A(0) = 0. Vy A = 0.
b) Vi x [0; a], xt hm s Bx : [0; f (a)] R cho bi
Zy

Zx
Bx (y) =

f (t)dt +
0

f 1 (t)dt xy.

Ta c Bx thuc lp C 1 trn [0; f (a)] v


y [0; f(a)], (Bx ) 0 (y) =f 1 (y) x
T ta c

(Bx ) 0 (y) = 0 y = f(x)


(Bx ) 0 (y) > 0 y [f(x); f(a)]
(Bx ) 0 (y) < 0 y [0; f(x)]

Dn n y [0; f(a)];Bx (y) Bx (f(x)) = 0 . Vy ta c iu chng minh.


Bi 4.12 (C Ng Gia T). p dng bt ng thc Cauchy cho cc s dng
8
ta c
2a 2b; b + 1; b + 1;
(a b)(b + 1)2
(2a 2b) + (b + 1) + (b + 1) +

8
8.
(a b)(b + 1)2

4. PHP TNH TCH PHN

149

Hay
a+

4
3
(a b)(b + 1)2

cho nn
f (x) = min{a +

4
} = 3.
(a b)(b + 1)2

ng thc xy ra nu v ch nu a = 2; b = 1.
Z 2014/3
Z 2014/3
f (x)dx =
3dx = 2014.
V vy,
0

Bi 4.13 (C Ng Gia T). p dng nh l Lagrange th tn ti c (t2 ; t)


sao cho t (0; 1).
Do f 00 > 0 cho nn f 0 tng trn [0; 1]. V vy, f 0 (c) > f 0 (t2 ).
Do ,
f (t) f (t2 )
f 0 (c) =
> f 0 (t2 )
t t2
hay
f (t) f (t2 ) > f 0 (t2 )(t t2 ).
T suy ra
Z

Z
f (t)dx

f (t )dt >
0

Z
>
0

1
(tt )f (t )dt =
2
2

1
f (t )dt + (1 t)f (t2 )|10 +
2
2

(1t)df (t )+
0

f (t)dt >
0

f (t2 )dt.

V vy,
Z

f (t)dt > 3

2
0

f (t2 )dt f (0).

Bi 4.14 (H Nng nghip). Gi thi gian t lc bt u tho nc n khi


cn b l x (pht). T gi thit ta c phng trnh
Z x
2400 =
(100 2t)dt x2 100x + 2400 = 0.
0

Dn n
x = 40 hoc x = 60.
Gi tr tm c x = 60 b loi v x 50.

150
Bi 4.15 (H Nng Nghip). Ta c
Z 2
2 Z 1
Z
f (x)dx =
f (x)dx +
0

2

f (x)dx

2

Z
2

f (x)dx

2 !

Z
+

f (x)dx

Suy ra
3
2

2

Z

Z
3

f (x)dx
0

2 Z
f (x)dx +

2 !
f (x)dx
.

Li c
Z

2
0

1
0

[f 0 (x)]2 dx

[f (x)] dx +

[f (x)] dx =

nn chng minh bi ton ta ch vic chng minh


2 Z 1
Z 1
[f 0 (x)]2 dx (1)
f (x)dx
3
0

2

Z

[f 0 (x)]2 dx

f (x)dx

(2)

Ta c theo cng thc tch phn tng phn:


Z 1
Z 1
Z
1
0
f (x)dx = xf (x) |0
xf (x)dx =
0

xf 0 (x)dx

T
Z

2

f (x)dx

Z
=

1
0

xf (x)dx

2

x dx
0

1
3

[f 0 (x)]2 dx

[f 0 (x)]2 dx.

Nh vy (1) c chng minh. Ta ch cn chng minh (2). Xt tch phn:


Z 2
Z 2
0
2
(2 x)f (x)dx = (2 x)f (x) |1 +
f (x)dx (do f (1) = 0).
1

T
Z
1

2 Z 2
2 Z 2
Z 2
0
2
f (x)dx =
(2 x)f (x)dx
(2 x) dx
[f 0 (x)]2 dx
1
1
1
Z
1 2 0
[f (x)]2 dx.
=
3 1

l iu phi chng minh.

4. PHP TNH TCH PHN

151

Bi 4.16 (HV Phng khng Khng qun). t

f (x) = x2 + 1 x.
Khi

f (x) =

1
1
=
.
x2 + 1 + x =
f (x)
x2 + 1 x

t t = x, ta c
Z

cos2014 x
dx =
1 + f (x)

I=
2

Z
=

cos2014 (t)

dt
1 + t + t2 + 1

cos2014 t
1 dt =
1 + f (t)

f (x) cos2014 x
dx.
1 + f (x)

T suy ra
Z

2I =
2

cos2014 x
dx +
1 + f (x)
2014

cos

Z
xdx = 2

f (x) cos2014 x
dx
1 + f (x)
cos2014 xdx

do cos2014 x l hm chn. Vy ta c
Z
I=

cos2014 xdx.

t In =

0 cosn xdx vi n N. t
(
u = cosn1 x,
dv = cos xdx,

ta suy ra
(
du = (n 1) cosn2 x( sin x)dx,
v = sin xdx
T ta c
n1

In = cos

x sin x|0 + (n 1)

cosn2 x(1 cos x)dx = (n 1)(In2 In .

Suy ra
In =

n1
In2
n

n 2.

Do vy, ta nhn c
I = I2014

1.3. . . .2013
1.3. . . .2013
=
I0 =
2.4. . . .2014
2.4. . . .2014

dx =
0

1.3. . . 2013
.
2.4. . . 2014 2

152
Bi 4.17 (HV Phng khng Khng qun). Ta c
Z
Z
2014
2014
1
1
dx
I=
dx
=
2
2
1 + ecos 2014x
1 + ecos 1007xsin 1007x
0
0
Z
Z
2
2014
2014
1
esin 1007x
=
dx =
dx.
2
2
1 + ecos2 1007xsin 1007x
ecos2 1007x+esin 1007x
0
0
t t =

2014

(1)
(2)

x, ta c:

esin 1007( 2014 t)


2
+ esin 1007( 2014 t) dt
2

cos
e 1007( 2014 t)

I=
Z

2014

2014

esin

2 ( 1007t)
2

2 ( 1007t)
2

ecos

2014

I=

+ esin
2

ecos

2 ( 1007t)
2

1007x

ecos2 1007x+esin

T (1) v (3) suy ra

2014

Z
2I =

dx =
0

Vy
I=

dt.

2 1007x

dx

(3)

.
2014

.
4028

Bi 4.18 (HV Phng khng Khng qun). Ta c


Z 2014
f (x)
I=
f (x + 2014
)dx
0
n
Z 2014. 1
Z 2014. 2
n
n
f (x)
f (x)
=
+
+
2014
1
f (x + n )dx
f (x + 2014
)dx
0
2014. n
n
Z 2014. n1
Z 2014
n
f (x)
f (x)
+
+
.
2014
f (x + n )dx
f (x + 2014
)dx
2014. n2
2014. n1
n
n
n
Xt tch phn
Z

2014. k+1
n

Ik =
k
2014. n

f (x)
,0 k n 1
f (x + 2014
)dx
n

t x = t + 2014. nk , ta c:
Z
Ik =
0

1
2014. n

f (t + 2014. nk )
=
f (t + 2014 k+1
)dt
n

Z
0

1
2014. n

f (x + 2014. nk )
, 0 k n 1.
f (x + 2014 k+1
)dx
n

4. PHP TNH TCH PHN

153

T ta c
2014

Z
I=
0

f (x)
f (x + 2014
)dx
n

1
2014. n

f (x)
+
f (x + 2014. n1 )dx

=
0
1
2014. n

f (x +
f (x +

+
0

2014. n2
)
n
n1 dx
2014. n )

1
2014. n

Z
0

f (x + 2014. n1 )
dx +
f (x + 2014. n2 )

1
2014. n

+
0

f (x + 2014. n1
)
n
dx.
n
f (x + 2014. n )

Suy ra
2014

Z
I=
0

f (x)
f (x + 2014
)dx
n

1
2014 n

Z
=
0

Z
+
0

1
2014 n

f (x)
+
f (x + 2014 n1 )dx

1
2014 n

Z
0

f (x + 2014 n2
)
n
n1 dx +
f (x + 2014 n )

Z
0

f (x + 2014 n1 )
dx +
f (x + 2014 n2 )

1
2014 n

f (x + 2014 n1
)
n
dx.
f (x)

Do f (x + 2014) = f (x), v f (x) l hm tun hon vi chu k T = 2014. T


ta c
Z 2014 1 h
n
f (x + 2014 n1 )
f (x)
I=
+
+
f (x + 2014 n1 ) f (x + 2014 n2 )
0
f (x + 2014 n2
) f (x + 2014 n1
)i
n
n
+
+
dx
f (x)
f (x + 2014 n1
)
n
p dng bt ng thc Cauchy cho n s dng, ta c:
f (x + 2014 n1 )
f (x)
+
n2
)
f (x + 2014 n1 ) f (x + 2014 2 ) + + f (x+2014 n1
n
+
n
f (x+2014.
)
n

T ta c:
Z
I n.

f (x+2014 n1
)
n
f (x)

n.

1
2014. n

dx = 2014.
0

Bi 4.19 (H Qung Bnh). Ta c


Z
2I =
0

ex .[(sin x + cos x) (cos x sin x)]dx


(sin x + cos x)2
Z
Z x
2
2 e .(cos x sin x)dx
ex dx
=

.
(sin x + cos x)2
0 sin x + cos x
0

154
Z x
2 e .(cos x sin x)dx
ex .(cos x sin x)dx
=
Xt H =
(sin x + cos x)2
(sin x + cos x)2
0
0
(cos x sin x)dx
1
t u = ex , dv =
ta c du = ex dx, chn v = sin x+cos
,
x
(sin x + cos x)2
Z

2
ex dx
ex
2
H=
|0 +
.
sin x + cos x
0 sin x + cos x
Do

ex
1
2I =
|02 = e 2 1 I = (e 2 1)
sin x + cos x
2
Z

Bi 4.20 (H Qung Bnh). a) Ta c


Z1

Z1
xf (x)dx +

Z1
F (x)dx =

Z1

x.F (x)dx +
0

Z1
=

F (x)dx
Z1

[xF (x) + F (x)]dx =


0

Do F (1) =

R1

xf (x)dx +

R1

[xF (x)]0 dx = xF (x)|10 .

F (x)dx.

b)
Z1
f (t)dt = F (1) F (x),
x

Theo gi thit ta c:
Z1
f (t)dt

1 x2
1 x2
F (1) F (x) =
, x [0, 1].
2
2

Z1

Z1
F (1)dx

Z1
F (x)dx

1 x2
dx,
2

Z1
F (1)

1
F (x)dx , ()
3

Z1
T (*) v cu a) ta c

1
xf (x)dx .
3

p dng bt ng thc Bunhiacpski cho hai hm s f (x) v g(x) = x ta


c:

4. PHP TNH TCH PHN


Z1

Z1

Z1

f (x)dx

x dx.
0

155

xf (x)dx

1
3

Z1

 2
1
f (x)dx
3
2

Bi 4.21 (H Qung Nam). S dng phng php i bin s, ta chng


minh c
Z /4
Z /4 

ln(cos x)dx =
ln cos(x ) dx.
4
0
0
T suy ra

Z /4 
cos(x 4
ln
) dx = 0.
cos x
0
Vy I =

ln 2.

Bi 4.22 (C S phm Qung Ninh). a) Ta c


Z
Z
1 x
1 x
f (t)dt 2014|
|f (t) 2014|dt
|
x 0
x 0
Do hm

|f (t) 2014|dt

g(x) =
0

l hm s tng khng m.
Nu g(x) + th
lim |f (x) 2014| = 0

x+

cho nn

1
lim
x+ x

f (t)dt = 2014.
0

M
cho nn
x

Nu g(x) < M th |f (x) 2014| <


1
lim
x+ x
b) Nu

Rb
a

|f (x)|dx =

Rb
a

f (t)dt = 2014.
0

f (x)dx th
Z b
(|f (x)| f (x))dx = 0.
a

t g(x) = |f (x)| f (x) lin tc, khng m v


Z b
g(x)dx = 0
a

nn g(x) = 0 hay |f (x)| = f (x).

156
Bi 4.23 (H Quy Nhn). t g(x) = f (x) sin x, x [0; /2]. Suy ra g(x)
/2
R
lin tc trn [0; /2] tho mn
g(x)dx = 0. Do xy ra hai trng hp.
0

a. Hm g(x) ng nht bng 0 trn [0; /2]: Ta c f (x) = sin x. S dng bt


ng thc
x3
sin x x, x [0; /2]
x
6
ta suy ra iu phi chng minh.
b. Hm g(x) khng ng nht bng khng trn [0; /2]: V g lin tc nn va
nhn gi tr m, va nhn gi tr dng trn [0; /2]. Theo nh l gi tr
trung gian, tn ti x0 (0; /2) sao cho g(x0 ) = 0. Do f (x0 ) = sin x0
v ta cng c iu phi chng minh.
Bi 4.24 (H S phm H Ni 2). t
p
Z 4
22
ln(9 x)
p
p
dx.
22
ln(9 x) + 22 ln(x + 3)
2
i bin x = 6 y, ta c:
Z

p
p
Z 4
22
ln(y + 3)
ln(x + 3)
p
p
p
p
dy =
dx.
22
22
22
ln(y + 3) +
ln(9 y)
ln(x + 3) + 22 ln(9 x)
2
22

Do , ta c:
Z
2I =

dx = 2.
2

Vy I = 1.
Bi 4.25 (H S phm H Ni 2). Ta c
Z x
Z x

|f (x)| =
f (t)dt
|f (t)|dt x [a, b].
a

p dng Bt ng thc Bunhiacopski ta c


Z x
 Z x  21  Z

|f (t)|dt
dt
a

x a.(

 12
|f (t)|2 dt

a
1

[f (t)]2 dt) 2

x [a, b].

Suy ra
|f (x)|

Z
x a(
a

[f (t)]2 dt) 2

x [a, b].

4. PHP TNH TCH PHN

157

Hay l
Z

[f (t)]2 dt x [a, b].

[f (t)] dt (x a).

f (x) (x a).
a

V vy
Z

b
2

f (x)dx

[f (t)] dt.

x a)

Rb

a
b

Z b

[f (t)]2 dtdx

(b a)2
(x a)dx =
2

[f (x)]2 dx.

Bi 4.26 (H S phm Thi Nguyn). Xt hm G(x) lin tc kh vi tng


khc tha mn

nu x [0, 1/3)
x
(4)
G(x) = 1 2x nu x [1/3, 2/3)

x 1 nu x [2/3, 1]
R 2/3
V 1/3 f (x)dx = 0 nn ta c
Z

1/3

Z
f (x)dx 2

f (x)dx =

Z
f (x)dx =

f (x)dx +

G0 (x)f (x)dx.

2/3

1/3

2/3

Tch phn tng phn ta nhn c


Z 1
Z 1
Z 1
1
f (x)dx =
f (x)dG(x) = [f (x)G(x)]|0
G(x)f 0 (x)dx
0
0
0
Z 1
=
G(x)f 0 (x)dx.
0

T p dng bt ng thc Cauchy-Schwarz ta c


Z 1
Z 1
2  Z 1
2 Z 1
0
2
f (x)dx =
G(x)f (x)dx
G (x)dx
(f 0 (x))2 dx
0
0
0
0
Z 1
1
=
(f 0 (x))2 dx.
27 0
T suy ra iu cn chng minh. Du bng xy ra khi f (x) = cG(x), c l
hng s.
Bi 4.27 (C S phm Vnh Phc). t hm h(x) = f (x) x. Khi h l
hm lin tc trn [0; 1] nn t gi tr ln nht ti x0 . Ta c
Z 1
Z x0
Z 1
Z 1
f (x)dx =
f (x)dx +
f (x)dx
f (x)dx.
0

x0

x0

158
Do f l hm n iu tng cho nn
Z 1
Z 1
f (x0 )dx = (1 x0 )f (x0 ).
f (x)dx
x0

x0

Suy ra
Z

f (x)dx (1 x0 )f (x0 ) = f (x0 ) x0 f (x0 )


0

= h(x0 ) + x0 (1 x0 )
Z 1
h(x0 )dx
h(x0 ) =
0
Z 1
h(x2014 )dx

0
Z 1
Z 2
2014
=
f (x )dx
x2014 dx
0
Z0 1
1
=
f (x2014 )dx
.
2015
0
Bi 4.28 (C S phm Vnh Phc). Ta c
Z 1/2
1 1
xf 0 (x)dx = f ( )
2 2
0
R 1/2
do 0 f (x)dx = 0. T suy ra
Z 1/2
Z 1
Z
2
VT =(
f (x)dx +
f (x)dx) = (
0

Z
=(

1/2

f (x)dx)2

1/2

1 1
(f (x) f (1/2)dx) + f ( ))2
2 2
1/2
Z 1
Z 1/2
xf 0 (x)dx)2
(1 x)f 0 (x)dx +
=(
1/2
1

Z
2(

1
12

0
0

1/2

(1 x)f (x)dx) + 2(
0

1/2
1

xf 0 (x)dx)2

(f 0 (x))2 dx.

CHUI S

Bi 5.1 (H Bch khoa H Ni). Khai trin hu hn hm


ln(1 + x) = x

x2
+ (x)
2

5. CHUI S

159

trong ln cn x = 0, vi (x) = o(x2 ). Suy ra




(1)n
(1)n
1
= +
ln 1 +
+ n ,
2n
n
n

 
1
n = o
.
n

+
X
(1)n

hi t theo tiu chun Leibniz.


Chui
n
n=2

+
+ 
X
X
1
1
1
1
V
+ n
khi n v
phn k nn
+ n cng phn
2n
2n
n
2n
n=2
n=2

k. Vy chui cho l phn k.


Bi 5.2 (HV Bu chnh Vin thng). Ta c
X

1
n(n + 1). . . (n + k)
n=1 k=1
=
=
=

1
n(n + 1). . . (n + k)
k=1 n=1

X
X
1
n(n + 1). . . (n + k 1)
1 1
k k!

k=1 n=1

X
k=1

1
1
(n + 1). . . (n + k) k

Mt khc
Z
0

ex 1
dx =
x

Z
0

X 1
1  X xk 
dx =
x k=1 k!
k!
k=1

Z
0

xk 
dx.
x

T nhng iu trn ta suy ra iu cn chng minh.


Bi 5.3 (D b). T gii thit ta c xn > 0 vi n N0 , N0 ln. Suy ra, vi
n N0 th
xn
xn+1 xn
xn+1 xn
1
=

.
xn+1
xn
aN0
V

X
xn+1 xn
a
)=
,
aN0
N0
n=N
0

nn chui cho hi t.
Ch : iu ny khng ng nu a < 0.

160
Bi 5.4 (H Hng Vng, Ph Th). Nhn xt an > 1n.
Ta c:
1
1
1
1
= 2
=
+
.
an+1 2
an 3an + 2
an 2 an 1
Suy ra
1
1
1

=
.
an 2 an+1 2
an 1

N
X
n=1

X

1
1
1
1
1
1
1
=

=
.
an 1 n=1 an 2 an+1 2
a1 2 aN +1 2
2 aN +1 2

Mt khc :
an+1 an = a2n 4an + 4 = (an 2)2 > 0 n.
{an } l dy tng.
Gi s
L = sup an L > 4.
n1

Nu L < + L = L2 3L + 4 L = 2 ( v l )
L = + lim an = +.
M
N
X
1
1
1
=
a 1
2 aN +1 2
n=1 n
nn cho N +

N 1
P
1
= .
2
n=1 an

Bi 5.5 (HV Phng khng Khng qun). Gi s


0

f (x) X
=
bn x n ,
g(x) =
f (x)
n=0
trong |bn | 2, n N theo gi thit. Mt khc ta c:
0

f (x)
f (x) = f (x)
.
f (x)
0

T gi thit ta c:
0

f (x) = a1 x + 2a2 x + 3a3 x2 + + nan xn1 +


Ta li c:
0

f (x)
f (x)
= (1+a1 x+a2 x2 + +an xn + )(b0 +b1 x+b2 x2 + +bn xn + )
f (x)

5. CHUI S

161

By gi, ta chng minh bng phn chng. Gi s |an | n + 1 khng ng


vi mi gi tr ca n. Gi k l s t nhin nh nht sao cho |ak | > k + 1. T
cc lp lun trn ta c
a1 x + 2a2 x + 3a3 x2 + + nan xn1 +
= (1+a1 x+a2 x2 + +an xn + )(b0 +b1 x+b2 x2 + +bn xn + )n N.
So snh h s ca xk1 hai v, ta c:
kak = b0 ak1 + b1 ak2 + + bk2 a1 + bk1

(1)

Xt hai v ca h thc (1) V tri c


kak > k(k + 1),
v k l s t nhin nh nht sao cho ak > k + 1. V phi c
|b0 ak1 + b1 ak2 + + bk2 a1 + bk1 | 2(|ak1 + + |a1 | + 1)
2(k + + 2 + 1) = k(k + 1).
Dn n mu thun vi (1). Vy:
|an | n + 1
Bi 5.6 (H Quy Nhn). a) V chui

n N.
an hi t nn theo tiu chun Cauchy

n=1

lim


a[ n2 ]+1 + a[ n2 ]+2 + ... + an = 0,

trong , [.] l k hiu hm phn nguyn. V dy {an } gim nn


n
a[ n2 ]+1 + a[ n2 ]+2 + ... + an [ ]an .
2
V vy,

P
n=1

[ n2 ]an = 0. Do ,

P
n=1

n
a
2 n

= 0 v ta cng c

nan = 0.

n=1

b) Ta c

3
3
sin n3 + n = (1)n sin n3 + n n
n

= (1)n sin p
3
(n3 + n)2 + n 3 n3 + n + n2
= (1)n an .
trong {an } l dy s dng gim dn v 0. Theo tiu chun Leibniz cho
chui an du, ta c chui cho hi t.

162
Bi 5.7 (H Sao ). t
cj = sup (n + 1)an

j 1.

n2j

Ta s ch ra
cj+1 4c2j .
Tht vy, vi mi n 2j+1 , tn ti s mt s nguyn k 2j sao cho n = 2k
hoc n = 2k + 1.
Nu n = 2k ta c
a2k a2k+1 a2k

4c2j
c2j
4c2j

(k + 1)2
2k + 1 2k + 2

Nu n = 2k + 1 ta c:
a2k+1 a2k+2 ak ak+1

c2j
4c2j
4c2j

.
(k + 1)(k + 2)
2k + 2 2k + 3

4c2

j
Do , dy (an n+1
)n2j+1 l mt dy tng cc s hng khng dng v hi
t n 0. V vy
4c2j
an
n 2j+1 .
n+1
Hay
c2j+1 4c2j .

Suy ra dy ((4cj )2t )j0 khng tng v v vy b chn trn bi mt s q < 1 .


V vy cj q 2t vi j ln. Vi bt k n trong khong 2j v 2j+1 ta c
an

cj

q 2t ( q)n .
n+1

V vy
lim

n
an < q < 1.

Theo tiu chun Cauchy, suy ra chui cho hi t.


Bi 5.8 (H S phm H Ni 2). Vi mi n N ta xt cc kh nng sau:
1
a) Nu an 2n+1
th ta c
n

ann+1 =

an
1

ann+1
b) Nu an <

1
2n+1

th

2an .

5. CHUI S

163
n

ann+1 < (

2n+1

) n+1 =

1
.
2n

V vy, ta lun c:
n

ann+1 2an +

Ta c chui

an hi t v chui

n=1

P
n=1

1
2n

1
2n

n N.

cng hi t.

Theo du hiu so snh cho chui s dng ta c chui

ann+1

n=1

cng hi t.
Bi 5.9 (C S phm Nam nh). un =

1
n2 + n + 1
2
n(n + 1)(n + 2)(n + 3) n

+
1
P
hi
t
nn
un hi t
2
n=1
n=1 n
1
1
Ta c n2 + n + 1 = .((n + 1)(n + 2)(n + 3) n(n + 1)(n + 2) .((n + 2)(n +
3
2
2
3) n(n + 1)) + ((n + 3) n)
3

Chui

+
P

1
un = .
3

1
1

n n+3

1
.
2

1
1

n(n + 1) (n + 2)(n + 3)

2
+ .
3
un = vn vn+1
!
1 1
1
1
1
vi vn = .
+

3 n n + 1n + 2
2
Sn = v1 vn+1
7
Do lim Sn = v1 =
n+
18

1
1

n(n + 1)(n + 2) (n + 1)(n + 2)(n + 4)

!
1
1
2
1
+
+ .
n(n + 1) (n + 1)(n + 2)
3 n(n + 1)(n + 2)

Bi 5.10 (H S phm Thi Nguyn). T gi thit ta c


a2n+1 = 2n+1 an+1 2n an ,
do

n
X
k=1

a2n = a2 2a + 2n an .

164
Tip theo ta s tnh
lim 2n an .

Tht vy gi 0

tha mn a = 2 sin2 hay


2
r
a
1
= arcsin(
) = arcos(1 a).
2
2

Bng chng minh quy np ta d dng chng minh c an = 2n sin2 (


Do vy ta c
lim 2n an = lim 22n sin2 (

Vy
lim

n
X
k=1

a2n

X
n=1

2n1

) = 42 = arcos2 (1 a).

a2n = a2 2a + arcos2 (1 a).

2n1

).

6. PHNG TRNH HM

165

PHNG TRNH HM

Bi 6.1 (HV Bu chnh Vin thng). Ly o hm 2 v ca


f (2015x + 2014) = 2015f (x)
Khi

x R.

2015f (2025x + 2014) = 2015f (x)


Thay x bi

x2014
2015

x R.

, ta c

x2014
2
2014
x 2014
0
0 x 2015 + 1
) = f ( 2015
)=f (
f (x) = f (
)
2015
2015
20152
n
0 x 2015 + 1
= = f (
).
2015n
0

Cho n , ta nhn c
0

f (x) = f (1).
Nh vy
f (x) = mx + k

x R

trong m = f (1). Thay x = 1 vo h thc ban u, ta c


f (1) = 2014f (1)
v do f (1) = 0. Bi vy k = m v do
f (x) = m(x + 1).
Th li: Tha mn. Nh vy
f (x) = m(x + 1)
trong m hng s.
Bi 6.2 (H Cng ngh thc phm Tp. HCM). a) T gi thit suy ra
f (f (f (x))) + f (f (x)) = 22 f (x)

n+2
n+1
f (x) + f (x) = 22 f n (x), n N
vi x = f 0 (x).
C nh x th (2) tr thnh phng trnh truy hi tuyn tnh cp 2
un+2 + un+1 = 22 un

(2)

166
Phng trnh c trng l 2 + 22 = 0 1 = 2 = 2. Suy ra
un = f n (x) = n A + (2)n B

(3)

Trong (3), thay n = 0, ta c x = u0 = A + B.


Trong (3), thay n = 1, ta c f (x) = A 2B.
V f n : [0, +) [0, +) nn ta c
f n (x)  n
=
A + (1)n B
0
(2)n
2
 n  1 n
V
0, khi n nn bt ng thc trn ng phi c
=
2
2
B = 0. Khi , f (x) c dng
f (x) = A = x.
Th li, ta thy f (x) = x tho mn phng trnh cho. Vy f (x) = x.
b) Phng trnh cho tng ng vi


x+y
x+y
xf (x) + yf (y)
f
, x, y R
(4)
=
2
2
2
t g(x) = xf (x). Khi , g(x) cng l hm lin tc trn R v t (4) suy ra


x+y
g(x) + g(y)
g
, x, y R.
(5)
=
2
2
Suy ra g(x) = ax + b, vi a, b l cc hng s cho trc tu .
T g(0) = 0 dn n b = 0. V g(x) = xf (x) nn f (x) = a, x 6= 0.
V f (x) lin tc ti im x = 0 nn a = 0.
Vy f (x) = 0 vi mi x R.
Bi 6.3 (H Hng Vng, Ph Th). Xt hm s
g(x) = ex/18 (f (x) 2014).
Suy ra g(0) = g(1) = 0. Hn na g(x) l hm lin tc trn [0; 1], c o hm
trn (0; 1). Ta c :
g 0 (x) =


ex/18
18f 0 (x) f (x) + 2014 0 x (0; 1)
18

g(x) l hm gim trn [0; 1].


M g(0) = g(1) = 0 g(0) 0 trn [0; 1]
f (x) 2014 trn [0; 1].
Th li ta thy f (x) 2014 trn [0; 1] tha mn.

6. PHNG TRNH HM

167

Bi 6.4 (C Ng Gia T). Gi s f l hm s cn tm. Khi


f (x + y) f (x).f (y) 2014x+y .
Thay x = y = 0 th f (0) f 2 (0) 1 nn f (0) = 1.
Thay y = x th f (0) f (x).f (x) 1.
V vy, f (x).f (x) = 1.
Thay y = 0 th f (x) 2014x . Suy ra f (x) 2014x . Cho nn
f (x).f (x) 2014x+(x) = 1.
V vy,
f (x) = 2014x .
Bi 6.5 (H Nng nghip). Khai trin Taylor ca hm f (x) ti cp 2:
f (x) = f (0) +

f 00 (cx ) 2
f 00 (cx ) 2
f 0 (0)
x+
x =1+x+
x , cx (0, x).
1!
2!
2

Suy ra
f (x) (1 + x) =

f 00 (cx ) 2
x.
2

Ly tch phn hai v ta c


Z

Z
f (x)dx

(1 + x)dx =
0

f 00 (cx ) 2
x dx 0.
2

T suy ra
Z

3
f (x)dx .
2

Du ng thc xy ra khi v ch khi f 00 (cx ) = 0, x, suy ra f 00 (x) = 0, x


(0, 1). Do ta c f (x) = 1 + x.
Bi 6.6 (H Qung Bnh). T gi thit ta c f (x) = 12 f ( x2 ) + x, x R
1 x
f
=
2
2


1
x
f 2 =
2
2
2
...

1
x
f
=
2n
2n

1 x x
.f 2 + 2
22
2
2
1 x x
f 3 + 4
23
2
2
1

 x 
x
f
+
2n+1
2n+1
22n

168
Cng cc ng thc trn v theo v ta c:


 x 
1
1
1
1
f (x) = n+1 f n+1 + x 1 + 2 + 4 + ... + 2n .()
2
2
2
2
2
Qua gii hn ng thc () khi n ta c:
lim f (x) = 0.f (x) + x.

1
1

1
4

4x
.
3

Bi 6.7 (H Sao ). Gi thit


f (y)

f (x + y) = f (x)e 2014 1

x, y R

(6)

Thay x = y = 0 vo (6) ta c:
f (0)

f (0) = f (0)e 2014 1

(7)

Nu f (0) = 0 th trong (6) thay x = 0 ta c f (y) = 0 y R.


Nu f (0) 6= 0 th t (7) suy ra f (0) = 2014.
Thay x = 0 trong (6) ta c
f (y)

f (y) = 2014e 2014 1

y R.

Do
f (y)
f (y)
= e 2014 1 > 0 y R.
2014

(8)

Xt hm s
g(u) = eu1 u, u > 0.
Ta c

g (u) = eu1 1
0

do g (u) = 0 hay u = 1. Do , phng trnh g(u) = u c nghim duy nht


u = 1.
V vy t (8) ta c
f (y) = 2014, y R.
Kt lun: f (x) 0 hoc f (x) 2014 trn R.
Bi 6.8 (H S phm H Ni 2). Gi s tn ti hm f tha mn yu cu ca
bi ton. Ta xt cc trng hp sau:
a) Nu f (f (x)) 0 x R th
f (x + y) f (x) + yf (f (x)) f (x) x R, y 0.

6. PHNG TRNH HM

169

V vy f l hm gim trn R v
f (0) > 0 f (f (x)) x R.
Suy ra f (x) > 0 x R (mu thun vi gi thit).
b) Nu tn ti z R sao cho f (f (z)) > 0 th
f (z + x) f (z) + xf (f (z)) x R.
V vy ta c
lim f (x) = + lim f (f (x)) = +.

x+

x+

Do tn ti x0 ln :
f (x0 ) 0, f (f (x0 )) > 1.
Cng do lim f (f (x)) = + ta c th chn y0
x+

x0 +1
f (f (x0 ))1

> 0 ln

f (f (x0 + y0 + 1)) 0.
T cc khng nh trn ta c:
f (x0 + y0 ) f (x0 ) + y0 f (f (x0 )) x0 + y0 + 1.
Suy ra
f (f (x0 + y0 )) = f [(x0 + y0 + 1) + (f (x0 + y0 ) x0 y0 1)]
f (x0 + y0 + 1) + [f (x0 + y0 ) x0 y0 1]f (f (x0 + y0 + 1))
f (x0 + y0 + 1) f (x0 + y0 ) + f (f (x0 + y0 ))
f (x0 ) + y0 f (f (x0 )) + f (f (x0 + y0 ))
> f (f (x0 + y0 ))
(mu thun) .
Bi 6.9 (C S phm Nam nh). Cho x = y = 0 ta c f (0) =
f (0) = 0. Cho x = 0 ta c f (y/3) = f (2y)/3. Vy
!
2x + y
1
f
= f (4x + 2y) .
3
3
Do
f (4x + 2y) = f (x) + f (2y).
3
Chn y sao cho 4x + 2y = x y = x. Do
2
f (x) = f (x) + f (3x)
Suy ra f (3x) = 0 f (x) = 0 vi mi x R

2
f (0)
3

170
Bi 6.10 (C S phm Nam nh). Ta c
Z1
I=
0

Z1
I=

!2
f 0 (x)
p
2 dx 0.
f (x)

(f 0 (x))2 dx
4
f (x)

Z1
0

f 0 (x)dx
p
+
f (x)

Z1
4dx
0

Z1

1
df (x)
p
+ 4x 0
f (x)
0
p
p
p
1
I 4 8 f (x) 0 + 4 = 8 8( f (1) f (0)) = 0.
I 44

p
p
f 0 (x)
= 2 2 f (x) = 2x + 2C. f (x) = x + C. Do
Vy I = 0 v p
f (x)
f (0) = 1 C = 1. Suy ra
f (x) = (x + 1)2
Bi 6.11 (C S phm Qung Ninh). Ta c
f (x) f (

x
x
x
x
x
)=
; , f (
) f(
)=
.
n1
n
2014
2014
2014
2014
2014n

f (x) f (

x
1
1
1
)
=
x(
+
+

+
).
2014n
2014 20142
2014n

Ta c
f (x) f (0) =
Vy
f (x) =

x
x/2014
=
.
1
2013
1
2014

x
+ 2014.
2013

Bi 6.12 (H S phm Thi Nguyn). Gi s tn ti hm f tha mn iu


kin bi ton. Ta thy (xy 2)2 + 2(x + y)2 = (x2 + 2)(y 2 + 2). t
g(x) = (x2 + 2)f (x),
theo gi thit ta thy hm g tha mn
g(x) x + 1, g(x + y) g(x)g(y)
vi mi x, y R. Tip theo ta s chng minh
g(x) = ex .

6. PHNG TRNH HM

171

Tht vy t iu g(x) x + 1 ta suy ra g(0) 1. T iu kin g(x + y)


g(x)g(y) ta suy ra g 2 (0) g(0), do vy g(0) 1, vy g(0) = 1.
T gi thit ta c g(x1 + x2 + + xn ) g(x1 )g(x2 ) . . . g(xn ), vi mi s t
nhin n. Do ta c
x
x
x
x
x
g(x) = g(n ) = g( + + ) g n ( ) (1 + )n .
n
n
n
n
n
nhng t

Bt ng thc ng vi mi n Z+ . Cho n , ta c
x
g(x) limn (1 + )n = ex . Hay g(x) ex vi mi x R. T gi thit ca g
n
1
cho y = x, ta c 1 = g(0) g(x)g(x), suy ra g(x)
, g(x) ex ,
g(x)
do ta thu c g(x) ex . Vy g(x) = ex . Hay
f (x) =

ex
.
x2 + 2

Th li ta thy hm f tha mn iu kin bi ton.


Bi 6.13 (C S phm Vnh Phc). Cho x = y = 0 th
f (0) (f (0))2 1
suy ra f (0) = 1. Cho x = y th
f (0) f (x).f (x) 1
nn
f (x) =

1
.
f (x)

Cho y = 0 th f (x) 2014x v f (x) 2014x . T suy ra


f (x) =

1
1

= 2014x .
f (x)
2014x

V vy,
f (x) = 2014x .
Kim tra c hm f (x) = 2014x tha mn yu cu bi ton.
Bi 6.14 (H Tn Tro). Tm hm s f : R R tha mn:
(x y)f (x + y) (x + y)f (x y) = 4xy(x2 + y 2 )

x, y R.

Gii. Ta c
(x y)f (x + y) (x + y)f (x y) = 4xy(x2 + y 2 ).

172
Suy ra
(x y)f (x + y) (x + y)f (x y)
1
1
= [(x+y)(xy)][(x+y)+(xy)][ [(x+y)+(xy)]2 [(x+y)(xy)]2 ]
4
4
t
(
u=xy
v =x+y
ta c
vf (u) uf (v) = (u + v)(u v)[(u + v)2 (u v)2 ].
Suy ra
vf (u) uf (v) = u3 v v 3 u
Hay
v[f (u) u3 ] = u[f (v) u3 ].
Vi uv 6= 0 ta c:
f (v) v 3
f (u) u3
=
u
v
Do

u, v R .

f (u) u3
=a
u

hay
f (u) = au + u3

u 6= 0.

Vi u = 0; v 6= 0 suy ra f (u) u3 = 0 hay f (u) = u3 do f (0) = 0. Hm


f (u) = au + u3 tha mn f (0) = 0. Vy
f (u) = au + u3
Hm s cn tm l f (x) = ax + x3 (a R).

u R.

You might also like